Thanks to visit codestin.com
Credit goes to www.scribd.com

0% found this document useful (0 votes)
91 views68 pages

Internal Medicine

A 35-year-old woman on hemodialysis 2-3 times daily develops tumoral calcinosis and markedly elevated PTH levels despite normal ionized calcium levels. This is most consistent with a diagnosis of secondary hyperparathyroidism, which occurs in patients with chronic kidney disease. Both MEN 2A and MEN 2B syndromes are associated with parathyroid hyperplasia and pheochromocytoma. A 55-year-old male presenting with cough, dyspnea and fever for 3 days shows right lower lobe infiltrates on chest x-ray. Based on exam and lab findings, he is likely in the consolidation/edema phase of community-acquired pneumonia.

Uploaded by

Go Ideas
Copyright
© © All Rights Reserved
We take content rights seriously. If you suspect this is your content, claim it here.
Available Formats
Download as DOCX, PDF, TXT or read online on Scribd
0% found this document useful (0 votes)
91 views68 pages

Internal Medicine

A 35-year-old woman on hemodialysis 2-3 times daily develops tumoral calcinosis and markedly elevated PTH levels despite normal ionized calcium levels. This is most consistent with a diagnosis of secondary hyperparathyroidism, which occurs in patients with chronic kidney disease. Both MEN 2A and MEN 2B syndromes are associated with parathyroid hyperplasia and pheochromocytoma. A 55-year-old male presenting with cough, dyspnea and fever for 3 days shows right lower lobe infiltrates on chest x-ray. Based on exam and lab findings, he is likely in the consolidation/edema phase of community-acquired pneumonia.

Uploaded by

Go Ideas
Copyright
© © All Rights Reserved
We take content rights seriously. If you suspect this is your content, claim it here.
Available Formats
Download as DOCX, PDF, TXT or read online on Scribd
You are on page 1/ 68

INTERNAL MEDICINE

101 35 yr old woman with chronic renal failure and undergoing hemodialysis 2-3x a day
develops tumoral calcinosis. Her PTH is markedly elevated while her ionized Ca is normal. The
most probable diagnosis is: A. Primary hyperparathyroidism B. Secondary hyperparathyroidism
C. Tertiary hyperparathyroidism D. Parathyroid carcinoma E. Pseudohyperparathyroidism Page
213 of IM Platinum. Elevated PTH occuring in CKD patients is secondary hyperparahyroidism.
ANGELA PAULINE P. CALIMAGLOYOLA (TOP 8 - FEB 2015 MED BOARDS;
TOPNOTCH MD FROM UST) DIAGNOSTIC EXAM - AUG 2015 102 Clinical features
present in both MEN 2A and MEN2B syndromes: A. Pituitary adenoma B. Parathyroid
hyperplasia C. Pheochromocytoma D. Marfanoid habitus E. Pancreatic islet cell hyperplasia
Page 208 of IM Platinum. Men 2A- Medullary thyroid carcinoma, pheochromocytoma,
parathyroid hyperplasia. Men 2B- Medullary thyroid carcinoma, Pheochromocytoma, mucosal
and gastrointestinal neuromas, marfanoid features. Men1- Parathyroid hyperplasia,pancreatic
islet cell hyperplasia/adenoma/carcinoma and pituitary hyperplasia/adenoma. ANGELA
PAULINE P. CALIMAGLOYOLA (TOP 8 - FEB 2015 MED BOARDS; TOPNOTCH MD
FROM UST) DIAGNOSTIC EXAM - AUG 2015 103 A 55 y/o male presents at the OPD with
cough, mild dyspnea, and fever of 37.8OC. Symptoms have been present for the past 3 days, and
when over-thecounter cold medications were having no effect, he decided to seek medical
attention. On PE, BP 120/70, PR 96 and regular, and RR 23. Lung examination revealed the
presence of right lower lung crackles with decreased breath sounds in the RLLF. Laboratory
studies revealed WBC of 15.6 with a left shift, sodium of 140, potassium of 4.5, BUN of 22 and
creatinine of 1.0. Chest x-ray revealed a right lower lobe infiltrate. The patient is in which phase
of CAP if there is predominance of neurophils, abundant fibrin deposition and cessation of
erythrocyte extravasation? A. Edema B. Red hepatization C. Gray hepatization D. Resolution E.
Consolidation Page 112 of IM Platinum. Pneumonia has four stages, namely consolidation, red
hepatization, grey hepatization and resolution. Consolidation/Edema-Occurs in the first 24 hours;
Cellular exudates containing neutrophils, lymphocytes and fibrin replaces the alveolar air.
Capillaries in the surrounding alveolar walls become congested. The infections spreads to the
hilum and pleura fairly rapidly Pleurisy occurs. Marked by coughing and deep breathing. Red
Hepatization- Occurs in the 2-3 days after consolidation; At this point the consistency of the
lungs resembles that of the liver. The lungs become hypeaemic. Alveolar capillaries are engorged
with blood. Fibrinous exudates fill the alveoli. This stage is "characterized by the presence of
many erythrocytes, neutrophils, desquamated epithelial cells, and fibrin within the alveoli" Grey
Hepatization- Occurs in the 2-3 days after Red Hepatization. This is an avascular stage. The lung
appears "gray-brown to yellow because of fibrinopurulent exudates, disintegration of red cells,
and hemosiderin" The pressure of the exudates in the alveoli causes compression of the
capillaries. "Leukocytes migrate into the congested alveoli" Resolution- This stage is
characterized by the "resorption and restoration of the pulmonary architecture". A large number
of macrophages enter the alveolar spaces. Phagocytosis of the bacteria-laden leucocytes occurs.
"Consolidation tissue re-aerates and the fluid infiltrate causes sputum". "Fibrinous inflammation
may extend to and across the pleural space, causing a rub heard by auscultation, and it may lead
to resolution or to organization and pleural adhesions" ANGELA PAULINE P.
CALIMAGLOYOLA (TOP 8 - FEB 2015 MED BOARDS; TOPNOTCH MD FROM UST)
DIAGNOSTIC EXAM - AUG 2015 TOPNOTCH MEDICAL BOARD PREP INTERNAL
MEDICINE SUPEREXAM For inquiries visit www.topnotchboardprep.com.ph or email us at
[email protected] TOPNOTCH MEDICAL BOARD PREP INTERNAL
MEDICINE SUPEREXAM Page 17 of 95 For inquiries visit www.topnotchboardprep.com.ph or
email us at [email protected] Item # QUESTION EXPLANATION
AUTHOR TOPNOTCH EXAM 104 GOLD COPD staging uses four categories of severity for
COPD, based on the value of FEV1. A patient classified under Stage III, has a predicted FEV1
of: A. >90 % B. >80 % C. 60 % D. 30 % E. < 70%; FEV1 > or equal to 80% predicted; With or
without chronic symptoms (cough, sputum production) 2: Moderate COPDFEV1/FVC < 70%;
FEV1 between 50 and 80% predicted; With or without chronic symptoms (cough, sputum
production) 3: Severe COPDFEV1/FVC < 70%; FEV1 between 30 and 50% predicted; With or
without chronic symptoms (cough, sputum production) 4: Very Severe COPD- FEV1/FVC <
70%; FEV1 < or equal to 30% predicted or FEV1 < 50% predicted plus chronic respiratory
failure ANGELA PAULINE P. CALIMAGLOYOLA (TOP 8 - FEB 2015 MED BOARDS;
TOPNOTCH MD FROM UST) DIAGNOSTIC EXAM - AUG 2015 105 A 75 y/o male who
underwent left nephrectomy for renal cell carcinoma 6 months ago is admitted to the emergency
department with acute severe dyspnea and cyanosis. On PE, BP 100/60; PR 120, RR 25, Oxygen
saturation is 75% while breathing room air and fails to rise under supplemental oxygen. The
patient is intubated and hooked to mechanically ventilator with 100% oxygen, which results in
further drop of the arterial saturation to 65% despite correct positioning of the tube. Chest X-ray
shows clear lungs without infiltrates. Based on the patient's diagnosis, which of the following is a
possible finding in 2D echo? A. RV hypokinesis B. RV hyperkinesis C. LV hypokinesis D. LV
hyperkinesis E. No possible 2D echo findings Page 121 of IM Platinum. This patient is probably
having a massive PE.Dyspnea is the most common symptom and tachypnea is the most frequent
sign. In massive PE like this case, on 2D echo there would be RV hypokinesis. ANGELA
PAULINE P. CALIMAGLOYOLA (TOP 8 - FEB 2015 MED BOARDS; TOPNOTCH MD
FROM UST) DIAGNOSTIC EXAM - AUG 2015 106 These kidney disorders are characterized
by proteinuria > 3.0 g/day, hypoalbuminemia, hyperlipidemia and edema, except: A. Minimal
change disease B. Membranous GN C. Focal-segment GN D. DM nephropathy E. Cresenteric
GN Page 176 of IM Platinum. These are characteristics of Nephrotic syndrome. A-D are diseases
which present with nephrotic syndrome. Cresenteric GN is a Nephritic syndrome, presenting
with Oliguria, Hematuria, Hypertension and Azotemia. ANGELA PAULINE P.
CALIMAGLOYOLA (TOP 8 - FEB 2015 MED BOARDS; TOPNOTCH MD FROM UST)
DIAGNOSTIC EXAM - AUG 2015 107 Criteria used in the diagnosis of DM can be either of
the following, except: A. HbA1C >/= 7.0% B. FPG >/= 7.0 mmol/L C. OGTT 2 hour plasma
glucose >/= 11.1 mmol/L D. Random plasma glucose >/= 11.1 mmol/L + classic symptoms of
hyperglycemia or hyperglycemic crisis E. None of the above Page 186 of IM Platinum. Criteria
for diagnosis may either be one of the following HbA1C >/= 6.5% , FPG >/= 7.0 mmol/L, OGTT
2 hour plasma glucose >/= 11.1 mmol/L and Random plasma glucose >/= 11.1 mmol/L + classic
symptoms of hyperglycemia or hyperglycemic crisis ANGELA PAULINE P.
CALIMAGLOYOLA (TOP 8 - FEB 2015 MED BOARDS; TOPNOTCH MD FROM UST)
DIAGNOSTIC EXAM - AUG 2015 108 Which laboratory test is the most sensitive test for
pheochromocytoma? A. Urinary VMA B. Urinary Metanephrines C. Urinary Catecholamines D.
Plasma Metanephrines E. Plasma VMA Page 221 of IM Platinum. The most sensitive and less
susceptible to false elevations from stress, including venipuncture is the measurement of plasma
metanephrine. ANGELA PAULINE P. CALIMAGLOYOLA (TOP 8 - FEB 2015 MED
BOARDS; TOPNOTCH MD FROM UST) DIAGNOSTIC EXAM - AUG 2015 109 The
following parameters are components of the Child Pugh score, except: A. Serum bilirubin B.
Serum albumin C. Serum creatinine D. Prothrombin time E. Ascites Page 233 of IM Platinum.
Child Pugh is used to assess the prognosis of chronic liver disease, mainly cirrhosis. Although it
was originally used to predict mortality during surgery, it is now used to determine the prognosis,
as well as the required strength of treatment and the necessity of liver transplantation. Its
components are Serum bilirubin, serum albumin, prothrombin time, Ascites and hepatic
encephalopathy. ANGELA PAULINE P. CALIMAGLOYOLA (TOP 8 - FEB 2015 MED
BOARDS; TOPNOTCH MD FROM UST) DIAGNOSTIC EXAM - AUG 2015 110 A 71 y/o
male patient had complained of swallowing difficulty after ingesting meat two days previously.
Endoscopy was done and a meat bolus was found 30 cm from the incisors, the bolus was
removed and a lower esophageal ring at the esophagogastric junction is seen. Biopsy reported
that the tissue has normal esophageal squamous epithelium on one side and gastric columnar
epithelium on the distal side of the membrane. What is the probable diagnosis? A. Esophageal
webs B. Schatzki ring C. Zenker diverticulum D. Nutcracker esophagus E. Diffuse esophageal
spasm Page 261 of IM Platinum. This is a case of "steak house syndrome" or schatzki ring. This
is one of the most common cause of intermittent food impaction with meat as the typical
instigator. ANGELA PAULINE P. CALIMAGLOYOLA (TOP 8 - FEB 2015 MED BOARDS;
TOPNOTCH MD FROM UST) DIAGNOSTIC EXAM - AUG 2015 TOPNOTCH MEDICAL
BOARD PREP INTERNAL MEDICINE SUPEREXAM For inquiries visit
www.topnotchboardprep.com.ph or email us at [email protected]
TOPNOTCH MEDICAL BOARD PREP INTERNAL MEDICINE SUPEREXAM Page 18 of 95
For inquiries visit www.topnotchboardprep.com.ph or email us at
[email protected] Item # QUESTION EXPLANATION AUTHOR
TOPNOTCH EXAM 111 Hemmorhoidal plexuses which traverse the anal canal are commonly
located in the following positions, except: A. Left lateral B. Right lateral C. Right anterior D.
Right posterior E. None of the above Page 265 of IM Platinum. The three positions of the main
hemmorhoidal cushions are, Left lateral, right anterior and right posterior. ANGELA PAULINE
P. CALIMAGLOYOLA (TOP 8 - FEB 2015 MED BOARDS; TOPNOTCH MD FROM UST)
DIAGNOSTIC EXAM - AUG 2015 112 A 35 y/o male withprogressing flank pain radiating to R
testicle and diagnosed to have urolithiasis. The most common stone to expect is: A. Calcium
oxalate stone B. Struvite stone C. Cystine stone D. Uric acid stone E. Magnesium-ammonium-
phosphate stone Page 180 of IM Platinum. The most common stone to expect are calcium stones.
ANGELA PAULINE P. CALIMAGLOYOLA (TOP 8 - FEB 2015 MED BOARDS;
TOPNOTCH MD FROM UST) DIAGNOSTIC EXAM - AUG 2015 113 A 24 y/o female
complains of palpitations. On physical examination she has a midsystolic click followed by a
mid to late crescendo systolic murmur at the apex. A 2-D echocardiogram was requested. This
will most likely reveal: A. Doming and restriction of motion of the mitral valve leaflets. B.
Marked superior displacement of mitral leaflets w/ coaptation point at or superior to annular
plane C. Fluttering mitral valve leaflet D. Mitral valve leaflets thickness 2 mm superior
displacement of the mitral leaflets into the left atrium during systole, with a leaflet thickness of at
least 5 mm Nonclassic MVP: Displacement is > 2 mm, with a maximal leaflet thickness of < 5
mm Other echocardiographic findings that should be considered as criteria are leaflet thickening,
redundancy, annular dilatation, and chordal elongation ANGELA PAULINE P.
CALIMAGLOYOLA (TOP 8 - FEB 2015 MED BOARDS; TOPNOTCH MD FROM UST)
DIAGNOSTIC EXAM - AUG 2015 114 The auscultatory findings in patient w/ ASD will be: A.
Holosystolic murmur at the pulmonic area w/ loud pulmonic component of S2(P2) B.
Holosystolic murmur at the base w/ wide and fixed split S2 C. Holosystolic murmur at the 4th
ICS at the R parasternal area w/ (+) Carvallo sign D. Pansystolic murmur along lower left sternal
border E. Crescendo-decrescendo murmur on the second intercostal space along the left sternal
border Because the pressure in the left atria initially exceeds that in the right, the blood flows in a
left to right shunt. This high volume of blood next passes into the right ventricle, and the ejection
of the excess blood through a normal pulmonary valve produces the prominent mid-systolic flow
murmur as heard in this sample. This murmur is best heard over the “pulmonic area” of the chest,
and may radiate into the back. The most characteristic feature of an atrial septal defect is the
fixed split S2. A split S2 is caused physiologically during inspiration because the increase in
venous return overloads the right ventricle and delays the closure of the pulmonary valve. With
an atrial septal defect, the right ventricle can be thought of as continuously overloaded because
of the left to right shunt, producing a widely split S2. Because the atria are linked via the defect,
inspiration produces no net pressure change between them, and has no effect on the splitting of
S2. Thus, S2 is split to the same degree during inspiration as expiration, and is said to be “fixed.”
ANGELA PAULINE P. CALIMAGLOYOLA (TOP 8 - FEB 2015 MED BOARDS;
TOPNOTCH MD FROM UST) DIAGNOSTIC EXAM - AUG 2015 115 A manifestation of
severe aortic regurgitation characterized as jarring of the body and bobbing of the head with each
systole: A. Quincke's pulse B. Duroziez sign C. De Musset sign D. Traube sign E. Austin Flint
sign Page 72 of IM Platinum. ANGELA PAULINE P. CALIMAGLOYOLA (TOP 8 - FEB 2015
MED BOARDS; TOPNOTCH MD FROM UST) DIAGNOSTIC EXAM - AUG 2015 116 A
vascular phenomena included as a minor criteria in the diagnosis of Infective Endocarditis: A.
Osler's nodes B. Roth's spots C. Subcutaneous nodules D. Janeway lesions E. Erythema
marginatum Page 65 of IM Platinum. Janeway lesions are nontender, painless, small
hemorrhagic macular/nodular lesions on the palms or soles. Osler's nodes and Roth spots are
immunologic and not vascular. Erythema marginatum and subcutaneous nodules are not included
in the Dukes criteria but in the Jones criteria for RF, ANGELA PAULINE P.
CALIMAGLOYOLA (TOP 8 - FEB 2015 MED BOARDS; TOPNOTCH MD FROM UST)
DIAGNOSTIC EXAM - AUG 2015 117 The most common arrythmia causing out of hospital
deaths from Myocardial Infarction is: A. Atrial fibrillation B. Asystole C. Premature ventricular
contractions D. Junctional rhythms E. Ventricular fibrillation SIMILAR TO PREVIOUS
BOARD EXAM CONCEPT/PRINCIPL Page 59 of IM Platinum. Most out of hospital deaths are
from ventricular fibrillation. ANGELA PAULINE P. CALIMAGLOYOLA (TOP 8 - FEB 2015
MED BOARDS; TOPNOTCH MD FROM UST) DIAGNOSTIC EXAM - AUG 2015
TOPNOTCH MEDICAL BOARD PREP INTERNAL MEDICINE SUPEREXAM For inquiries
visit www.topnotchboardprep.com.ph or email us at [email protected]
TOPNOTCH MEDICAL BOARD PREP INTERNAL MEDICINE SUPEREXAM Page 19 of 95
For inquiries visit www.topnotchboardprep.com.ph or email us at
[email protected] Item # QUESTION EXPLANATION AUTHOR
TOPNOTCH EXAM 118 A 23 y/o patient is undergoing surgery for a zygomatic fracture. The
surgeon inadvertently applied too much pressure on the patient's eyeball. The anesthesiologist
then noted that the patient's ECG tracing converted from sinus rhythm to sinus bradycardia. The
anesthesiologist knows that this phenomenon is called: A. Aschner reflex B. Vasovagal reflex C.
Oculobradycardia phenomenon D. Neurocardiogenic syncope E. Oculovagal reflex SIMILAR
TO PREVIOUS BOARD EXAM CONCEPT/PRINCIPLE. The oculocardiac reflex, also known
as Aschner phenomenon, Aschner reflex, or Aschner-Dagnini reflex, is a decrease in pulse rate
associated with traction applied to extraocular muscles and/or compression of the eyeball. The
reflex is mediated by nerve connections between the ophthalmic branch of the trigeminal cranial
nerve via the ciliary ganglion, and the vagus nerve of the parasympathetic nervous system.
ANGELA PAULINE P. CALIMAGLOYOLA (TOP 8 - FEB 2015 MED BOARDS;
TOPNOTCH MD FROM UST) DIAGNOSTIC EXAM - AUG 2015 119 The most useful
indicator/index of LV function is the ejection fraction, to compute for EF the following
parameter is needed: A. Mean arterial pressure in mmHg B. Systemic vascular resistance in
dynes C. Cardiac output D. End systolic volume E. Stroke volume Page 39 of IM Platinum. The
formula for EF is EF= SV/EDV. ANGELA PAULINE P. CALIMAGLOYOLA (TOP 8 - FEB
2015 MED BOARDS; TOPNOTCH MD FROM UST) DIAGNOSTIC EXAM - AUG 2015 120
Dopamine has varying hemodynamic effects based on the dose. The dose needed to activate the
DA1 and DA2 receptors causing splanchnic and renal vasodilation is: A. 8 mcg/kg/min B. 6
mcg/kg/min C. 4 mcg/kg/min D. 1 mcg/kg/min E. 10 mcg/kg/min Page 27 of IM Platinum. The
renal dose of dopamine is 1-2 mcg/kg/min. Inotropic dose is 2- 4 mcg/kg/min, >5mcg/kg/min is
a vasoconstrictor dose. ANGELA PAULINE P. CALIMAGLOYOLA (TOP 8 - FEB 2015 MED
BOARDS; TOPNOTCH MD FROM UST) DIAGNOSTIC EXAM - AUG 2015 121 Patient
presents with a high-pitched, blowing, diastolic murmur, heard best in the third intercostal space
along the left sternal border. This is most likely a case of: A. Ventricular septal defect B. Mitral
stenosis C. Aortic regurgitation D. Atrial septal defect E. Pulmonic stenosis In patients with
severe AR, the aortic valve closure sound (A2) is usually absent. An S3 and systolic ejection
sound are frequently audible, and occasionally an S4 also may be heard. The murmur of chronic
AR is typically a high-pitched, blowing, decrescendo diastolic murmur, heard best in the 3rd
intercostal space along the left sternal border. In patients in whom the AR is caused by primary
valvular disease, the diastolic murmur is usually louder along the left than the right sternal
border. However, when the murmur is heard best along the right sternal border, it suggests that
the AR is caused by aneurysmal dilatation of the aortic root. Note: At least 3 questions about
heart murmurs were asked in IM Feb 2015 boards, mentioning only about the auscultation
finding. LYNN DARYL FELICIANO VILLAMATER, MD (TOP 5 - FEB 2015 MED
BOARDS; TOPNOTCH MD FROM EAC) MIDTERM 1 EXAM - AUG 2015 122 Ventricular
tachycardia lasting for more than 30 seconds and not terminated by therapy will lead to? A.
Hemodynamic collapse B. Fatal arrythmia C. Asynchronous rhythm D. Recurrent arrythmia E.
VT storm Sustained polymorphic VT, ventricular flutter, and VF all lead to immediate
hemodynamic collapse. Emergency asynchronous defibrillation is therefore required, with at
least 200-J monophasic or 100-J biphasic shock. (Harrison) SIMILAR TO PREVIOUS BOARD
EXAM CONCEPT. LYNN DARYL FELICIANO VILLAMATER, MD (TOP 5 - FEB 2015
MED BOARDS; TOPNOTCH MD FROM EAC) MIDTERM 1 EXAM - AUG 2015 123
Treatment for ischemic and nephrotoxic acute kidney injury include: A. None B. Hydration C.
Antibiotics D. Loop diuretics E. Immunosuppressives Many different approaches to attenuate
injury or hasten recovery have been tested in ischemic and nephrotoxic AKI. These include ANP,
low-dose dopamine, etc. Whereas many of these are beneficial in experimental models of
ischemic or nephrotoxic ATN, they have either failed to confer consistent benefit or proved
ineffective in humans. (Harrison's) SIMILAR TO PREVIOUS BOARD EXAM CONCEPT
LYNN DARYL FELICIANO VILLAMATER, MD (TOP 5 - FEB 2015 MED BOARDS;
TOPNOTCH MD FROM EAC) MIDTERM 1 EXAM - AUG 2015 124 The most useful test for
distinguishing prerenal acute renal failure from intrinsic renal failure is: A. BUN Creatinine
Ratio B. FENa C. Serum Sodium D. Creatinine clearance E. Serum creatinine The most useful
test for distinguishing prerenal ARF from ischemic or nephrotoxic intrinsic renal ARF is the
fractional excretion of sodium (FENa). Prerenal ARF typically have a FENa of 38C B.
Hypothermia of 24 D. Tachycardia of >90 E. Leukocytosis>10,000 . Harrison’s 18th edition
chapter 271 page 2223 table 271-1 and IM platinum page 297. according to these sources
tachypnea should be >24. I was actually shocked as i crossed reference it with other sources like
Schwartz 9th ed and medscape in which they considered tachypnea of >20. It is odd that
harrisons and medscape have the same source which is the american college of chest physicians
but they have different values HAROLD JAY S. BAYTEC, MD (TOP 10 - FEB 2015 MED
BOARDS; TOPNOTCH MD FROM FEU) MIDTERM 3 EXAM - AUG 2015 179 This is the
most active drug against M leprae and inhibits Folate synthesis. A. Dapsone B. Rifampicin C.
Clofazimine D. Isoniazid E. Sulfamethoxazole HAROLD JAY S. BAYTEC, MD (TOP 10 - FEB
2015 MED BOARDS; TOPNOTCH MD FROM FEU) MIDTERM 3 EXAM - AUG 2015 180
Which among the following findings is consistent for Moderate Risk Community Acquired
Pneumonia in a Diabetic patient? A. RR of 28 B. PR of 120 C. Temperature of 39C D. BP of
100/60 E. RBS of 300 CAP MR-- RR of 30 or above, PR of 125 or above, Temp of equal or
more than 40 or less than or equal to 36, altered mental state of acute onset. Suspected aspiration,
decompensated comorbidities. RBS of 300 means that there is decompensation of the DM which
poses more risk to infection HAROLD JAY S. BAYTEC, MD (TOP 10 - FEB 2015 MED
BOARDS; TOPNOTCH MD FROM FEU) MIDTERM 3 EXAM - AUG 2015 181 What is the
drug of choice for post-splenectomy sepsis? A. Penicillin 20-30M units/day IV continuous
infusion B. Ceftriaxone 1-2g/day IV/IM OD C. Cefuroxime 1.5g IV/IM q8 D. Vancomycin
500mg IV q6 E. Clindamycin 2g/day IV/IM q6-q12 SIMILAR TO PREVIOUS BOARD EXAM
CONCEPT/PRINCIPLE.. Ceftriaxone is the drug of choice for post-splenectomy sepsis JEAN
PAOLO M. DELFINO, MD (TOP 10 - FEB 2015 MED BOARDS; TOPNOTCH MD FROM
FATIMA) FINAL EXAM - AUG 2015 182 In pleural effusion, the following are factors
indicating the likely need for a procedure more invasive than a thoracentesis except? A.
Loculated pleural fluid B. Pleural fluid pH 3.3 mmol/L (20 years of age. The screen should
include a fasting lipid profile repeated every 5 years. GRACE ARVIOLA, MD (TOP 3 - AUG
2014 MED BOARDS; TOPNOTCH MD) DIAGNOSTIC EXAM - FEB 2015 203 Auscultation
of a patient with a known valvular heart defect revealed a holosystolic murmur noted at the left
parasternal border. The murmur characteristically increased in intensity during inspiration. The
valve affected is: A. Aortic valve B. Pulmonary valve C. Mitral valve D. Tricuspid valve E. Both
aortic and mitral valve This is Carvallo sign of tricuspid regurgitation. GRACE ARVIOLA, MD
(TOP 3 - AUG 2014 MED BOARDS; TOPNOTCH MD) DIAGNOSTIC EXAM - FEB 2015
204 Standard treatment of a patient with suspected multidrug-resistant healthcare-associated
pneumonia consist of: A. 1 Anti-pseudomonal + 1 MRSA-active antibiotics B. 1 Anti-
pseudomonal + 2 MRSA-active antibiotics C. 2 Anti-pseudomonal + 1 MRSA-active antibiotics
D. 2 Anti-pseudomonal + 2 MRSA-active antibiotics E. None of the above The standard
recommendation for patients with risk factors for MDR infection is for three antibiotics: two
directed at P> aeruginosa and one at MRSA. GRACE ARVIOLA, MD (TOP 3 - AUG 2014
MED BOARDS; TOPNOTCH MD) DIAGNOSTIC EXAM - FEB 2015 205 Adenosine
deaminase is used in the diagnosis of: A. TB pleural effusion B. Lung adenocarcinoma C. COPD
D. Asthma E. Silicosis Determination of the pleural concentration of adenosine deaminase
(ADA) is a useful screening test: tuberculosis is virtually excluded if the value is very low.
GRACE ARVIOLA, MD (TOP 3 - AUG 2014 MED BOARDS; TOPNOTCH MD)
DIAGNOSTIC EXAM - FEB 2015 206 A 56 year-old patient suffering from sudden onset of
dyspnea underwent 2D-echocardiography. Imaging showed a hypokinetic right ventricular free
wall with a normal apex. This indirect sign is highly suggestive of: A. COPD exacerbation B.
Bronchial asthma C. Pulmonary embolism D. Congestive heart failure E. Malignant pleural
effusion This is McConnell's sign of pulmonary embolism. GRACE ARVIOLA, MD (TOP 3 -
AUG 2014 MED BOARDS; TOPNOTCH MD) DIAGNOSTIC EXAM - FEB 2015 207 In the
computation of a patient's corrected plasma sodium concentration, what other laboratory value
must be considered? A. Albumin B. Calcium ion concentration C. Random blood sugar D.
Potassium E. Blood pH Plasma Na concentration falls by 1.6 to 2.4 mM for every 100 mg/dL
increase in glucose due to glucose-induced water efflux from cells. This true hyponatremia
resolves after correction of hyperglycemia. GRACE ARVIOLA, MD (TOP 3 - AUG 2014 MED
BOARDS; TOPNOTCH MD) DIAGNOSTIC EXAM - FEB 2015 208 The anemia seen in
chronic kidney disease is usually: A. Microcytic, hypochromic B. Normocytic, normochromic C.
Macrocytic, hyperchromic D. Normocytic, hypochromic E. Microcytic, hyperchromic A
normocytic, normochromic anemia is observed as early as CKD stage 3 and is almost universal
by stage 4. GRACE ARVIOLA, MD (TOP 3 - AUG 2014 MED BOARDS; TOPNOTCH MD)
DIAGNOSTIC EXAM - FEB 2015 209 Which of the following conditions does NOT predispose
to urolithiasis? A. Bacterial infection B. Dehydration C. Metabolic syndrome D. Diet high in
calcium E. Gout Studies have shown that low-calcium diets increase the risk of incident stone
formation, perhaps by reducing the amount of calcium in the intestine to bind oxalate, thereby
incerasing urine oxalate levels. GRACE ARVIOLA, MD (TOP 3 - AUG 2014 MED BOARDS;
TOPNOTCH MD) DIAGNOSTIC EXAM - FEB 2015 TOPNOTCH MEDICAL BOARD PREP
INTERNAL MEDICINE SUPEREXAM For inquiries visit www.topnotchboardprep.com.ph or
email us at [email protected] TOPNOTCH MEDICAL BOARD PREP
INTERNAL MEDICINE SUPEREXAM Page 30 of 95 For inquiries visit
www.topnotchboardprep.com.ph or email us at [email protected] Item #
QUESTION EXPLANATION AUTHOR TOPNOTCH EXAM 210 In the management of
diabetic ketoacidosis, what condition can potentially withold the administration of insulin? A.
Hypernatremia B. Hypokalemia C. Metabolic acidosis D. Impaired mental status E. Severe
hyperglycemia If the initial serum potassium is 50 mL/kg/day B. Urine osmolarity 65y/o in atrial
fibrillation with hypertension, mitral regurgitation or hx of heart failure. ASA for patients 95%
D. during ischemia, diastolic dysfunction precedes ECG changes and regional wall motion
abnormalities E. all of the above LEAN ANGELO SILVERIO, MD (TOP 4 - AUG 2014 MED
BOARDS; TOPNOTCH MD), MD MIDTERM EXAM 1 - FEB 2015 224 which of the
following is an absolute contraindication for fibrinolysis in STEMI? A. Thalamic hemorrhage 10
years ago B. Current use of anticoagulants C. Active peptic ulcer D. Traumatic or prolonged
(>10mins) CPR or major surgery ( 400U/L E. Severe jaundice poor prognostic markers for
alcoholic hepatitis: Discriminant Function of >32, encephalopathy, ascites, renal failure,
prolonged PT, bilirubin >20mg/dl. LEAN ANGELO SILVERIO, MD (TOP 4 - AUG 2014 MED
BOARDS; TOPNOTCH MD), MD MIDTERM EXAM 1 - FEB 2015 231 which of the folowing
condition is associated with SAAG >1.1g/dl? A. Nephrotic syndrome B. Colonic malignancy C.
Genitourinary TB D. Right sided heart failure E. All of the above A SAAG value of >1.1g/dl is
almost always indicate portal hypertension. LEAN ANGELO SILVERIO, MD (TOP 4 - AUG
2014 MED BOARDS; TOPNOTCH MD), MD MIDTERM EXAM 1 - FEB 2015 232 what is
the primary treatment for thrombotic thrombocytopenic purpura? A. Aspirin plus dipyridamole
B. Platelet transfusion C. plasmapharesis D. Intravenous IVIG E. Splenectomy TTP is a type of
microangiopathies wherein there is a deficiency in the vWF cleaving protease (ADAMTS13).
This result to large vWF multimers in the plasma promoting aggregant effect. The only treatment
for this case is plasma exchange ( plasmapheresis with infusion of FFP or cryosupernatant
infusion. platelet transfusion is generally contraindicated unless invasive procedures is required.
other ancillary treatment however with unknown benefit include: dipyridamole, ASA,
prednisone. IVIG is reserved for refractory cases. LEAN ANGELO SILVERIO, MD (TOP 4 -
AUG 2014 MED BOARDS; TOPNOTCH MD), MD MIDTERM EXAM 1 - FEB 2015 233 A
67 y/o male was worked up for persistent proteinemia. The laboratory results showed the ff: M
protein 2.4 g/dl, BMA 8% plasma cells, serum creatinine 0.9mg/dl, Hgb 145, bone radiograph
shows osteoporotic changes of the vertebra. what is your primary diagnosis? A. Multiple
myeloma B. waldenstrom macroglobulinemia C. MGUS D. plasmacytoma E. none of the above
MGUS is the most common dysproproteinemia, M protein is < 3g/dl and the plasma cell is <
10% ( MM >10% BMA plasma cell). Patient is asymptomatic. Serum creatinine, hemoglobin and
bone radiographs are normal. LEAN ANGELO SILVERIO, MD (TOP 4 - AUG 2014 MED
BOARDS; TOPNOTCH MD), MD MIDTERM EXAM 1 - FEB 2015 234 A 47 y/o male
asymptomatic presents in your clinic with an incidental finding of WBC count 110x109 /L,
basophilia, eosinophilia and obliterated traube space. Cytogenetic studies showed t(9,22). Which
of the following is true about his condition except? A. the standard therapy for this condition is
Imatinib B. leukocyte alkaline phosphatase score is low or zero C. increased serum Vitamin B12
level D. philadelphia chromosome is the hallmark of this condition E. all of the above this is a
classic case of CML. Characterized by increased WBC count, granulocytes in all stages of
maturation. Presence of philadelphia chromosome is the hallmark for this condition. This is
caused by translocation of bcr -abl gene. Leukocyte alkaline phosphatase ( marker of functional
activity of granulocytes) is low compared to reactive leukocytosis. there is an increaesed Vitamin
B12 level due to increased level of transcobalamin I. Imatinib is the treatment of choice. LEAN
ANGELO SILVERIO, MD (TOP 4 - AUG 2014 MED BOARDS; TOPNOTCH MD), MD
MIDTERM EXAM 1 - FEB 2015 TOPNOTCH MEDICAL BOARD PREP INTERNAL
MEDICINE SUPEREXAM For inquiries visit www.topnotchboardprep.com.ph or email us at
[email protected] TOPNOTCH MEDICAL BOARD PREP INTERNAL
MEDICINE SUPEREXAM Page 33 of 95 For inquiries visit www.topnotchboardprep.com.ph or
email us at [email protected] Item # QUESTION EXPLANATION
AUTHOR TOPNOTCH EXAM 235 68 y/o male with DM nephropathy with creatinine of 2.4
then suddenly came in creatinine of 9 mg/dl without any other hx of blood loss or diarrhea. he
also have anemia and Low back pain in the past 6 mo, 24 hr urine protein showed proteinuria
iof10g/day, the urine dipstick is negative ? A. Multiple Myeloma B. FSGS C. RPGN D. Uremia
E. any of the above proteinuria with negative urine dipstick accompanied by low back pain and
suddenc increase in creatinine is a classic manifestation of myeloma kidney. LEAN ANGELO
SILVERIO, MD (TOP 4 - AUG 2014 MED BOARDS; TOPNOTCH MD), MD MIDTERM
EXAM 1 - FEB 2015 236 which laboratory finding will distinguish acute tubular necrosis from
pre renal azotemia? A. Urine spgr 1.031 B. BUN/Creatinine 30:1 C. Urine osmolality of 300
mOsm/L D. FeNa% 0.8% E. Urine Na 20mEq/L Acute tubular necrosis: Urine Osm - 30 mg/dl;
FeNa >3% LEAN ANGELO SILVERIO, MD (TOP 4 - AUG 2014 MED BOARDS;
TOPNOTCH MD), MD MIDTERM EXAM 1 - FEB 2015 237 which of the following will
decrease in glomerulonephritis presenting with nephrotic syndrome? A. Urinary excretion of
antithrombin III B. Atrial natriuretic peptide C. vasopressin D. VLDL E. None of the above
urinary loss of antithrombin is main causative factor why nephrotic syndrome is prone to
thrombotic states. Loss of albumin leads to compensatory increase in the synthesis of lipoprotein
leading to increase in VLDL fraction. ADH is increase as well as aldosterone to compensate for
sodium and water loss. ANP is decreased at this state. LEAN ANGELO SILVERIO, MD (TOP 4
- AUG 2014 MED BOARDS; TOPNOTCH MD), MD MIDTERM EXAM 1 - FEB 2015 238
which of the following is not a manifestation of Fanconi Syndrome? A. anemia B. acidosis C.
aminoaciduria D. glycosuria E. Hyperuricosuria fanconi syndrome is a renal disease affecting the
proximal renal tubule. Glucose, amino acid, phosphate,uric acid,bicarbonate are completely
passes in urine instead of being reabsorbed. LEAN ANGELO SILVERIO, MD (TOP 4 - AUG
2014 MED BOARDS; TOPNOTCH MD), MD MIDTERM EXAM 1 - FEB 2015 239 which of
the following is true about acquired Immunodeficiency syndrome except? A. Diagnosis is
attained if anyone with HIV infection develop diseases associated with T cell defect and CD4
count of 10% is an AIDS defining illness. E. immune reconstitution syndrome is frequently seen
in the setting of mycobacterial infection anemia is the most common hematologic abnormality in
HIV patients and in the absence of a specific treatable cause is independently associated with
poor prognosis. LEAN ANGELO SILVERIO, MD (TOP 4 - AUG 2014 MED BOARDS;
TOPNOTCH MD), MD MIDTERM EXAM 1 - FEB 2015 240 this type of respiratory failure
results in alveolar hypoventilation secondary to impaired respiratory drive? A. Type 1 B. Type 2
C. Type 3 D. Type 4 E. None of the above type 1, acute hypoxemic respiratory failure -d/t
alveolar flooding and subsequent intrapulmonary shunt physiology. Type 3-d/t lung atelectasis.
Type 4 - hypoperfusion of respiratory muscles in patient in shock. SIMILAR TO PREVIOUS
BOARD EXAM CONCEPT/PRINCIPLE LEAN ANGELO SILVERIO, MD (TOP 4 - AUG
2014 MED BOARDS; TOPNOTCH MD), MD MIDTERM EXAM 1 - FEB 2015 241 A 50 year
old male 10 pack year smoker and hypertensive consulted due to dyspnea. History revealed
multiple episodic attacks of dyspnea and coughing over the past few years. He said that he has
had siblings with similar coughing episodes as well. He has verbalized that the dyspnea and
coughing would frequently accompany colds and an episode of flu would last a few weeks and
would eventually resolve only to return again after a few months. Initial PE revealed
symmetrical chest expansion, occasional wheezes on both lung bases, no other remarkable
findings, most likely condition is A. COPD - emphysema B. bronchial asthma C. congestive
heart failure D. anginal equivalent E. post nasal drip syndrome remissions and exacerbations, (+)
family history, PE and history point more to asthma KEVIN BRYAN LO, MD (TOP 7 - AUG
2014 MED BOARDS; TOPNOTCH MD) MIDTERM 2 EXAM - FEB 2015 TOPNOTCH
MEDICAL BOARD PREP INTERNAL MEDICINE SUPEREXAM For inquiries visit
www.topnotchboardprep.com.ph or email us at [email protected]
TOPNOTCH MEDICAL BOARD PREP INTERNAL MEDICINE SUPEREXAM Page 34 of 95
For inquiries visit www.topnotchboardprep.com.ph or email us at
[email protected] Item # QUESTION EXPLANATION AUTHOR
TOPNOTCH EXAM 242 30 year old female patient diagnosed case of rheumatic heart disease
with mitral regurgitation presented with new onset fever over the past 5 days, dyspnea, easy
fatigability, 2D echo revealed vegetations on the mitral valve, however blood cultures returned
negative, which of the following antibiotics would most likely be used for this patient? A.
vancomycin B. ampicillin C. ceftriaxone D. gentamicin E. penicillin G SIMILAR TO
PREVIOUS BOARD EXAM CONCEPT/PRINCIPLE, no culture isolated organism, give broad
spectrum IV antibiotic that covers for the most common organisms KEVIN BRYAN LO, MD
(TOP 7 - AUG 2014 MED BOARDS; TOPNOTCH MD) MIDTERM 2 EXAM - FEB 2015 243
Dilated cardiomyopathy is most commonly brought about by which of the following? A.
infectious etiology B. genetic factors C. hypertensive heart disease D. autoimmune destruction E.
immunologic dysfunction SIMILAR TO PREVIOUS BOARD EXAM CONCEPT/PRINCIPLE,
hypertensive heart disease still most common cause KEVIN BRYAN LO, MD (TOP 7 - AUG
2014 MED BOARDS; TOPNOTCH MD) MIDTERM 2 EXAM - FEB 2015 244 Which of the
following hepatitis virus causes the most cases of chronic liver disease in humans? A. hepatitis A
B. hepatitis B C. hepatitis C D. hepatitis D E. hepatitis E SIMILAR TO PREVIOUS BOARD
EXAM CONCEPT/PRINCIPLE, only about 1% of cases of Hep B go on to become chronic and
cause CLD KEVIN BRYAN LO, MD (TOP 7 - AUG 2014 MED BOARDS; TOPNOTCH MD)
MIDTERM 2 EXAM - FEB 2015 245 Which of the following drugs are classified to have a
narrow therapeutic index, is both nephrotoxic and ototoxic at the same time? A. vancomycin B.
imipinem cilastatin C. furosemide D. cyclophosphamide E. cisplatin SIMILAR TO PREVIOUS
BOARD EXAM CONCEPT/PRINCIPLE, only vancomycin reflects all characteristics,
(aminoglycosides also have the same characteristics) KEVIN BRYAN LO, MD (TOP 7 - AUG
2014 MED BOARDS; TOPNOTCH MD) MIDTERM 2 EXAM - FEB 2015 246 Nitroglycerin is
usually given to relieve chest pain during an acute anginal attack, which of the following is its
predominant mechanism of action? A. acts as a venodilator B. negative inotropic C. negative
chronotropic D. decreases peripheral vascular resistance E. prevents platelet aggregation
SIMILAR TO PREVIOUS BOARD EXAM CONCEPT/PRINCIPLE, venodilator, decrease
preload KEVIN BRYAN LO, MD (TOP 7 - AUG 2014 MED BOARDS; TOPNOTCH MD)
MIDTERM 2 EXAM - FEB 2015 247 A 32 year old female patient was noted to have rheumatic
heart disease. Patient comes in for follow up check up since she was referred back by her
cardiologist to her primary care physician, after routine vital signs checking,BP130/60 heart rate
of 96 RR of 20, the patient told her primary care physician that she was told by her cardiologist
that she had a valvular problem developed as complication of her condition, the primary care
physician most likely thinks this is? A. mitral stenosis B. mitral regurgitation C. aortic stenosis
D. aortic regurgitation E. none of the above SIMILAR TO PREVIOUS BOARD EXAM
CONCEPT/PRINCIPLE, AR = wide pulse pressure KEVIN BRYAN LO, MD (TOP 7 - AUG
2014 MED BOARDS; TOPNOTCH MD) MIDTERM 2 EXAM - FEB 2015 248 32 year old
female patient treated for SLE went into remission a few weeks ago and discontinued her
medications when she got well, 5 days prior, she started having cough and colds and body
malaise, she reported recent episodes of light headedness, muscle weakness, weight loss. BP was
120/80 at sitting but on standing dropped to 80/60, there was some noted hyperpigmentation of
the skin on the dorsum of the arms, which of the following conditions is most likely? A. SLE
flare B. sepsis secondary to pneumonia C. dehydration D. medication side effects E. adrenal
insufficiency SIMILAR TO PREVIOUS BOARD EXAM CONCEPT/PRINCIPLE, skin changes
and hypotensive episodes = primary adrenal insufficiency KEVIN BRYAN LO, MD (TOP 7 -
AUG 2014 MED BOARDS; TOPNOTCH MD) MIDTERM 2 EXAM - FEB 2015 TOPNOTCH
MEDICAL BOARD PREP INTERNAL MEDICINE SUPEREXAM For inquiries visit
www.topnotchboardprep.com.ph or email us at [email protected]
TOPNOTCH MEDICAL BOARD PREP INTERNAL MEDICINE SUPEREXAM Page 35 of 95
For inquiries visit www.topnotchboardprep.com.ph or email us at
[email protected] Item # QUESTION EXPLANATION AUTHOR
TOPNOTCH EXAM 249 60 year old male patient came in due to easy fatigability, no other
symptoms noted on ROS. On examination, patient was noted to be pale, no oral lesions, no
palpable cervical lympadenopathies, chest, abdominal PE findings were unremarkable, DRE
revealed smooth rectal vault, (-) blood on tactating finger, CBC revealed hgb 9.5g/dL which of
the following is the most likely pathology? A. chronic renal failure B. lymphoma C. occult GI
malignancy D. Pulmonary TB E. HIV SIMILAR TO PREVIOUS BOARD EXAM
CONCEPT/PRINCIPLE, unexplaine anemia in elderly rule out occult GI malignancy KEVIN
BRYAN LO, MD (TOP 7 - AUG 2014 MED BOARDS; TOPNOTCH MD) MIDTERM 2
EXAM - FEB 2015 250 40 year old female patient come in with neck mass of 3 years duration,
noted gradual increase in size, she also complained of occasional difficulty swallowing and
complained of the cosmetic disturbance due to the mass in her neck. The patient was worked up
and evaluated to be euthyroid, ultrasound revealed multinodular goiter, which of the following
treatments is most applicable? A. subtotal thyroidectomy B. radioactive iodine I131 C. thyroid
hormone levothyroxine supplementation D. PTU antithyroid medication E. SSKI (saturated
solution of potassium iodide) SIMILAR TO PREVIOUS BOARD EXAM
CONCEPT/PRINCIPLE, multinodular goiter, subtotal thyroidectomy is the best treatment of
choice for this condition KEVIN BRYAN LO, MD (TOP 7 - AUG 2014 MED BOARDS;
TOPNOTCH MD) MIDTERM 2 EXAM - FEB 2015 251 A 55 year old male patient admitted
for severe pneumonia went into septic shock, which of the following is a possible effect on the
patient's kidney? A. increase GFR decrease sodium and water retention B. decrease GFR
increase sodium and water retention C. no change in GFR decrease in sodium and water
retention D. no change in GFR increase in sodium and water retention E. none of the aove
SIMILAR TO PREVIOUS BOARD EXAM CONCEPT/PRINCIPLE, about 2 similar question
in this form of a physio question, personally I did not study for IM as a board subject, study your
pharma, patho, micro, physio and you will be ok plus your internship clerkship experiences as
well, one of the easiest subjects on the boards IM platinum would do ok as well which I read
during internship KEVIN BRYAN LO, MD (TOP 7 - AUG 2014 MED BOARDS; TOPNOTCH
MD) MIDTERM 2 EXAM - FEB 2015 252 A 56 year old male marathon runner collapsed on
track, when he was brought in for medical assistance, he was found to be very dehydrated and
hypotensive, assessment was heat exhaustion and severe dehydration, which type of renal failure
will this patient most likely develop if the condition is allowed to progress? A. pre renal B. intra
renal C. post renal D. all of the above E. none of the above SIMILAR TO PREVIOUS BOARD
EXAM CONCEPT/PRINCIPLE, pre renal due to shock dehydration KEVIN BRYAN LO, MD
(TOP 7 - AUG 2014 MED BOARDS; TOPNOTCH MD) MIDTERM 2 EXAM - FEB 2015 253
A 40 year old female patient came into the clinic complaining of epigastric pain 2-3 hours after
eating, this was accompanied by burning sensation going upward into the chest, which of the
following treatments is most suitable for this patient? A. AlMgOH take 1 tablet as needed B.
omeprazole 20mg/tablet 1 tablet OD C. omeprazole 40mg/tablet 1 tablet OD D. prednisone
10mg/tab 1 tab OD E. ISDN 5mg/tab 1 tab sublingual SIMILAR TO PREVIOUS BOARD
EXAM CONCEPT/PRINCIPLE, for GERD need to be high dose proton pump inhibitors
KEVIN BRYAN LO, MD (TOP 7 - AUG 2014 MED BOARDS; TOPNOTCH MD) MIDTERM
2 EXAM - FEB 2015 254 50 year old female patient known diabetic and hypertensive for 10
years recently complained of episodes of exertional dyspnea, bipedal edema, nocturnal cough
and 2 pillow orthopnea, which of the following oral hypoglycemic medications are
contraindicated for this patient? A. metformin B. glimepiride C. sitagliptin D. acarbose E.
pioglitazone thiazolidinediones can cause excess water retention which may exacerbate patient's
beginning heart failure thus is contraindicated KEVIN BRYAN LO, MD (TOP 7 - AUG 2014
MED BOARDS; TOPNOTCH MD) MIDTERM 2 EXAM - FEB 2015 TOPNOTCH MEDICAL
BOARD PREP INTERNAL MEDICINE SUPEREXAM For inquiries visit
www.topnotchboardprep.com.ph or email us at [email protected]
TOPNOTCH MEDICAL BOARD PREP INTERNAL MEDICINE SUPEREXAM Page 36 of 95
For inquiries visit www.topnotchboardprep.com.ph or email us at
[email protected] Item # QUESTION EXPLANATION AUTHOR
TOPNOTCH EXAM 255 24 year old military recruit residing with his platoon developed high
fever, nuchal rigidity, altered consciousness and rapid onset of purpuric rashes over the trunk and
lower extremities, the drug of choice for this condition is? A. penicillin G B. ceftriaxone C.
ampicillin sulbactam D. azithromycin E. clindamycin SIMILAR TO PREVIOUS BOARD
EXAM CONCEPT/PRINCIPLE, drug of choice is still PenG for meningo KEVIN BRYAN LO,
MD (TOP 7 - AUG 2014 MED BOARDS; TOPNOTCH MD) MIDTERM 2 EXAM - FEB 2015
256 A 65 year old male 20 pack year smoking history presents with a history of increasing
dyspnea, weight loss, occasional intermittent coughing. Chest xray was requested by his primary
physician which revealed a hilar mass on the right hemithorax causing collapse of the right upper
lung lobe, which of the following was the most likely PE findings in this patient? A. bilateral
crackles over lung bases occasional wheezing bilateral lung bases B. decreased breath sounds
over right upper lung field, occasional wheezing right hemithorax C. dullness on right lung base,
increased tactile and vocal fremiti D. tracheal shifted to the left, decreased breath sounds over the
right hemithorax E. (+) bronchophony and egophony right lung base, occasional crackles right
lung A more on asthma, C pneumonia, D trachea should be on the same side with atelectasis, E
more on pneumonia KEVIN BRYAN LO, MD (TOP 7 - AUG 2014 MED BOARDS;
TOPNOTCH MD) MIDTERM 2 EXAM - FEB 2015 257 In relation to the question above,
which of the following diagnostic procedures is most appropriate for patient's condition? A. CT
guided aspiration biopsy B. transbronchial biopsy C. resection biopsy via thoracotomy incision
D. VATS guided lung biopsy E. none of the above SIMILAR TO PREVIOUS BOARD EXAM
CONCEPT/PRINCIPLE, accessible hilar lung mass can be accessed through bronchoscopy
KEVIN BRYAN LO, MD (TOP 7 - AUG 2014 MED BOARDS; TOPNOTCH MD) MIDTERM
2 EXAM - FEB 2015 258 Which of the following conditions below could potentially produce
hypovolemic hyponatremia? A. SIADH B. water intoxication C. diuretic therapy D. Conn's
syndrome E. none of the above SIMILAR TO PREVIOUS BOARD EXAM
CONCEPT/PRINCIPLE, A and B, D are hypervolemic KEVIN BRYAN LO, MD (TOP 7 - AUG
2014 MED BOARDS; TOPNOTCH MD) MIDTERM 2 EXAM - FEB 2015 259 Which of the
following is the test of choice to document H. Pylori eradication following treatment? A. rapid
urease test B. urea breath test C. gastric mucosal biopsy D. H.pylori serological test E. all of the
above urea breath test to document cure KEVIN BRYAN LO, MD (TOP 7 - AUG 2014 MED
BOARDS; TOPNOTCH MD) MIDTERM 2 EXAM - FEB 2015 260 50 year old male patient
smoker and alcoholic with history of chronic epigastric pain passed black tarry stools, the
physician knows that the stool was probably in the GI tract for at least how long? A. 14 hours B.
16 hours C. 20 hours D. 24 hours E. 72 hours SIMILAR TO PREVIOUS BOARD EXAM
CONCEPT/PRINCIPLE, 14 hours for melena KEVIN BRYAN LO, MD (TOP 7 - AUG 2014
MED BOARDS; TOPNOTCH MD) MIDTERM 2 EXAM - FEB 2015 261 A 54 year old male
who has been hospitalized for severe acute pancreatitis complained of onset of dyspnea and
tachypnea. Chest X-ray reveals diffuse bilateral infiltrates and stat ABG revealed severe
hypoxemia. Pulmonary capillary wedge pressure was measured and was unremarkable. What is
the most likely pathophysiologic mechanism of this condition? A. Bronchial mucous plugging B.
Alveolar wall destruction C. Left ventricular infarction D. Endothelial damage E. Interstitial
fibrosis This is a case of ARDS which results from endothelial damage and leakage of protein-
rich material into the alveolar spaces. There are many causes, one of which is severe pancreatitis.
RAYMUND MARTIN LI, MD (TOP 1 - AUG 2014 MED BOARDS; TOPNOTCH MD)
MIDTERM 3 EXAM - FEB 2015 TOPNOTCH MEDICAL BOARD PREP INTERNAL
MEDICINE SUPEREXAM For inquiries visit www.topnotchboardprep.com.ph or email us at
[email protected] TOPNOTCH MEDICAL BOARD PREP INTERNAL
MEDICINE SUPEREXAM Page 37 of 95 For inquiries visit www.topnotchboardprep.com.ph or
email us at [email protected] Item # QUESTION EXPLANATION
AUTHOR TOPNOTCH EXAM 262 A patient with known coronary artery disease is prescribed
with a maintenance dose of aspirin. Which of the following is the mode of action of this drug in
preventing future ischemic events? A. Inhibition of cyclooxygenase 2-mediated synthesis of
prostaglandins B. Irreversible acetylation of prostacyclin C. Inhibition of prostaglandin synthase
1 D. Direct blockade of thromboxane action on platelets E. Reversible acetylation of
cyclooxygenase SIMILAR TO PREVIOUS BOARD EXAM CONCEPT/PRINCIPLE.
Prostaglandin synthase I or COX1 - irreversibly acetylated by aspirin to decrease synthesis of
thromboxane. Prostaglandin synthase II or COX2 is selectively expressed in inflammatory cells
and mediate inflammation. Aspirin inhibits both but its action on prostaglandin synthase I or
COX1 is what helps prevent platelet aggregation. RAYMUND MARTIN LI, MD (TOP 1 - AUG
2014 MED BOARDS; TOPNOTCH MD) MIDTERM 3 EXAM - FEB 2015 263 Given the
following clicical picture: (+) HbsAg, (- )anti-Hbs, (+) anti-Hbc IgG, (-) HbeAg, (+) anti-Hbe,
(+) anti-HbA IgG, (+) anti-HbC, what is the diagnosis? A. Past Hepatitis A infection, Chronic
Hepatitis B carrier, Hepatitis C infection B. Acute Hepatitis A infection, Chronic Hepatitis B
carrier, Hepatitis C infection C. Past Hepatitis A infection, Chronic Active Hepatitis B, Hepatitis
C infection D. Acute Hepatitis A infection, Chronic Hepatitis B carrier, Resolved Hepatitis C
infection E. Past Hepatitis A infection, Past Hepatitis B vaccination, Hepatitis C infection
Review hepatitis laboratory profile RAYMUND MARTIN LI, MD (TOP 1 - AUG 2014 MED
BOARDS; TOPNOTCH MD) MIDTERM 3 EXAM - FEB 2015 264 A 63 year old male patient
comes to your clinic complaining of redness and painful swelling of the right knee. Synovial
fluid analysis revealed an inflammatory picture and analysis under polarized microscopy shows
negative birefringerence. What is the most likely etiology? A. Autoimmune pathology B. Crystal
arthropathy C. Dissemininated gonococcal infection D. Wear and tear E. None of the above
Urate appears as needle like crystals with negative birefringerence under polarized light
RAYMUND MARTIN LI, MD (TOP 1 - AUG 2014 MED BOARDS; TOPNOTCH MD)
MIDTERM 3 EXAM - FEB 2015 265 A chronic alcoholic was hospitalized for lethargy and
confusion. Physical examination findings include icteric sclerae, positive fluid wave and shifting
dullness, bilateral pedal edema, and flapping tremors of the hand. Which of the following will
help improve that patient's current reason for admission? A. Alkalinization of urine B. Colonic
acidification C. Large-volume paracentesis D. Adequate intravenous hydration E. TIPS This is a
case of hepatic encephalopathy. Lactulose is converted by colonic bacteria into organic acids,
which traps ammonia and prevents diffusion back into the circulation RAYMUND MARTIN LI,
MD (TOP 1 - AUG 2014 MED BOARDS; TOPNOTCH MD) MIDTERM 3 EXAM - FEB 2015
266 A patient presents with anemia and enlarged spleen upon palpation. Peripheral smear reveals
RBC's with lack of central pallor. Family history is also significant for the condition. If a
splenectomy is performed on this patient, which of the following may be prevented? A. Portal
vein hypertension B. Neoplastic transformation C. Gallstone formation D. Thrombocytosis E.
Invasive bacterial infection Hereditary spherocytosis is an extravascular type pf hemolytic
anemia which may cause gallstone formation due to accumulation of unconjugated bilirubin.
Splenectomy will prevent this. RAYMUND MARTIN LI, MD (TOP 1 - AUG 2014 MED
BOARDS; TOPNOTCH MD) MIDTERM 3 EXAM - FEB 2015 267 In a patient who recently
had myocardial infarction, when do you expect the possibility of an autoimmune-mediated type
of pericarditis after MI? A. 12-24 hours B. 1-3 days C. 4-7 days D. 7-14 days E. >2 weeks
Fibrinous pericarditis may 1-3 days after MI as a response to the necrotic tissue. But the
autoimmune type of pericarditis called Dressler's syndrome occurs weeks to months after MI
RAYMUND MARTIN LI, MD (TOP 1 - AUG 2014 MED BOARDS; TOPNOTCH MD)
MIDTERM 3 EXAM - FEB 2015 268 An elderly hospitalized patient who has been confined to
bed complained of acute onset dypnea, tachypnea and shortness of breath. Further examination
revealed left calf pain and tenderness. What will be the expected ABG findings? A. pH=7.31,
pCO2=50, pO2=98 B. pH=7.29, pCO2=29, pO2=76 C. pH=7.39, pCO2=39, pO2=88 D.
pH=7.52, pCO2=49, pO2=54 E. pH=7.49, pCO2=29, pO2=72 Symptomatic pulmonary
embolism will cause tachypnea, hypoxemia, and respiratory alkalosis RAYMUND MARTIN LI,
MD (TOP 1 - AUG 2014 MED BOARDS; TOPNOTCH MD) MIDTERM 3 EXAM - FEB 2015
TOPNOTCH MEDICAL BOARD PREP INTERNAL MEDICINE SUPEREXAM For inquiries
visit www.topnotchboardprep.com.ph or email us at [email protected]
TOPNOTCH MEDICAL BOARD PREP INTERNAL MEDICINE SUPEREXAM Page 38 of 95
For inquiries visit www.topnotchboardprep.com.ph or email us at
[email protected] Item # QUESTION EXPLANATION AUTHOR
TOPNOTCH EXAM 269 A 5 year old patient is brought to the ER by her mother due to
complaints of polyuria, nausea and vomiting, and abdominal pain. Respiration is noted to be
deep and rapid and urine ketones is positive. If serum chemistry levels are measured, which of
the following will be expected? A. Increased K, Deacresed Na, Increased Glucose B. Increased
K, Increased Na, Increased Glucose C. Decreased K, Decreased Na, Increased Glucose D.
Increased K, Increased Na, Normal Glucose E. Decreased K, Decreased Na, Decreased Glucose
DKA will cause shift of potassium from intacellular to extracellular causing hyperkalemia. The
increase in osmotic pressure will cause brisk diuresis and loss of Na causing hyponatremia
RAYMUND MARTIN LI, MD (TOP 1 - AUG 2014 MED BOARDS; TOPNOTCH MD)
MIDTERM 3 EXAM - FEB 2015 270 What is the drug of choice in a patient with Addison's
disease presenting with acute adrenal crisis? A. Prednisone B. Prednisolone C. Hydrocortisone
D. Betamethasone E. Dexamethasone Hydrocortisone IV is the drug of choice in acute adrenal
crisis RAYMUND MARTIN LI, MD (TOP 1 - AUG 2014 MED BOARDS; TOPNOTCH MD)
MIDTERM 3 EXAM - FEB 2015 271 A 23 year old female patient complaining of epigastric
pain underwent endoscopy which revealed a duodenal ulcer. She was started on Proton pump
inhibitor therapy. The addition of antibiotics to the treament regimen will help: A. Decrease
recurrence risk B. Relieve symptoms C. Facilitate ulcer healing D. Prevent disease transmission
E. Prevent malignant transformation Antibiotic therapy helps eradicate H. pylori and primarily
decreases recurrence risk RAYMUND MARTIN LI, MD (TOP 1 - AUG 2014 MED BOARDS;
TOPNOTCH MD) MIDTERM 3 EXAM - FEB 2015 272 What is the most common cause of
acute kidney injury? A. Intrinsic AKI B. Postrenal AKI C. Acute Tubular Necrosis D. Prerenal
AKI E. Obstructive uropathy RAYMUND MARTIN LI, MD (TOP 1 - AUG 2014 MED
BOARDS; TOPNOTCH MD) MIDTERM 3 EXAM - FEB 2015 273 A 23 year old female
patient who was diagnosed with Grave's disease is being maintained on a thionamide medication.
In order to rapidly detect a rare idiosyncratic adverse reaction that is associated with this drug,
which laboratory tests should be monitored? A. Liver function test B. Alkaline phosphatase C.
CBC D. Serum potassium E. Serum creatinine Agranulocytosis is a rare idiosyncratic reaction of
thionamides (ex. Methimazole) RAYMUND MARTIN LI, MD (TOP 1 - AUG 2014 MED
BOARDS; TOPNOTCH MD) MIDTERM 3 EXAM - FEB 2015 274 A 72 year old male patient
who is a native of Baguio City is complaining of knee pain that is worse in the afternoon and
during exertion. He lives a sedentary lifestyle and is overweight. Family history reveals that this
runs in the family. There are no other associated symptoms. Which of the following is the most
important risk factor for causing this disease in this patient? A. Cold climate B. Gender C.
Obesity D. Family history E. Age Age is the primary predisposing factor for osteoarthritis
RAYMUND MARTIN LI, MD (TOP 1 - AUG 2014 MED BOARDS; TOPNOTCH MD)
MIDTERM 3 EXAM - FEB 2015 275 A 25 year old patient who is being treated for a urinary
tract infection develops conjunctival redness and difficulty in ambulation due to a swollen knee.
Which of the following is true about this condition? A. Reactive arthritis B. Associated with
HLA-B27 C. Seronegative type of arthritis D. A and B E. All of the above Reiter's
syndrome/Reactive arthritis - seronegative, associated with HLA B27, triad of urethritis,
conjunctivitis, and arthritis RAYMUND MARTIN LI, MD (TOP 1 - AUG 2014 MED BOARDS;
TOPNOTCH MD) MIDTERM 3 EXAM - FEB 2015 TOPNOTCH MEDICAL BOARD PREP
INTERNAL MEDICINE SUPEREXAM For inquiries visit www.topnotchboardprep.com.ph or
email us at [email protected] TOPNOTCH MEDICAL BOARD PREP
INTERNAL MEDICINE SUPEREXAM Page 39 of 95 For inquiries visit
www.topnotchboardprep.com.ph or email us at [email protected] Item #
QUESTION EXPLANATION AUTHOR TOPNOTCH EXAM 276 A elderly female patient who
recently suffered from left-sided hemiparesis is able to comprehend speech but is unable to
repeat words and has difficulty using words and sentences. Which of the following areas of the
cortex is most likely affected? A. Inferior frontal gyrus B. Superior frontal gyrus C. Superior
temporal gyrus D. Posterior parietal gyrus E. None of the above Broca's area - expressive aphasia
RAYMUND MARTIN LI, MD (TOP 1 - AUG 2014 MED BOARDS; TOPNOTCH MD)
MIDTERM 3 EXAM - FEB 2015 277 A 72 year old patient presents at the ER with complaints
of dypnea and chest discomfort. P.E. findings include decreased dullness upon percussion and
decreased breath sounds and vocal fremitus. She has a history of chronic hypertension. Chest
xray reveals cardiomegaly and blunting of the costophrenic angle. Which of the following
findings is most consistent with the patient's most likely diagnosis upon thoracentesis? A. Pleural
fluid LDH of >200 U/L B. Pleural fluid protein of >3 g C. Pleural fluid and serum protein
gradient of >31g/L D. Pleural -serum LDH ratio >0.6 E. All of the above transudative (ex. CHF,
nephrotic): PF LDH31g/L RAYMUND MARTIN LI, MD (TOP 1 - AUG 2014 MED BOARDS;
TOPNOTCH MD) MIDTERM 3 EXAM - FEB 2015 278 In the recovery phase of acute renal
failure, one should be most concerned of which of the following possible complications? A.
Volume overload B. Hypokalemia C. Hypernatremia D. Hyperuricemia E. Hyperkalemia
Recovery phase is the diuretic phase placing patient at risk for hypokalemia RAYMUND
MARTIN LI, MD (TOP 1 - AUG 2014 MED BOARDS; TOPNOTCH MD) MIDTERM 3
EXAM - FEB 2015 279 A 54 year old anxious and diaphoretic male patient presents at the ER
complaining of severe substernal chest pain that is unrelieved by rest. Physical examination
reveals bradycardia and ECG shows ST segment elevation of II, III, aVF. BP is 130/90, HR is 52,
and RR is 23. Which of the following is the most consistent with the patient's scenario? A.
Cardiogenic shock secondary to massive infarction B. Blockage of the vessel supplying the SA
and AV node C. Compensatory parasympathetic response D. Pericardial tamponade E. Coronary
atherosclerosis affecting the left anterior descending artery Inferior infarct - blockage of right
coronary artery which supplies SA and AV node RAYMUND MARTIN LI, MD (TOP 1 - AUG
2014 MED BOARDS; TOPNOTCH MD) MIDTERM 3 EXAM - FEB 2015 280 A patient who
was diagnosed with bacterial pharyngitis is treated with amoxicillin and develops a pruritic
maculopapular rash. Patient claims that he was treated with amoxicillin several years ago without
any reaction. What is your primary etiologic consideration? A. RNA virus B. Catalase positive
cocci C. DNA virus D. Fungal infection E. Drug-induced hypersensitivity reaction Infectious
mononucleosis (EBV) - Almost all patients with mononucleosis that are treated with
amoxicillin/ampicillin develops a generalized pruritic maculopapular rash RAYMUND
MARTIN LI, MD (TOP 1 - AUG 2014 MED BOARDS; TOPNOTCH MD) MIDTERM 3
EXAM - FEB 2015 281 A 58 year old male was brough to the ER due to a one week history of
progressive headache and fever. He has been diagnosed with Pulmonary Tuberculosis but has
been unable to start his medications due to financial difficulties. On examination, he was drowsy
but arousable with note of nuchal rigidity and a positive Brudzinski sign. Lumbar puncture was
done which showed an elevated WBC count of lymphocytic predominance, an elevated protein
level and a decreased glucose level. Which of the following statements is true regarding the most
probable condition of the patient? A. Polymerase chain reaction (PCR) is the gold standard for
diagnosis B. The disease often presents abruptly with a acute onset of severe headache and
vomiting C. Adjunctive treatment with dexamethasone promotes significantly reduces the
frequency of neurologic sequelae D. It is more common in elderly than in young children E.
Involvement is more pronounced at the base of the brain, with paresis of cranial nerves being a
Dx: TB meningitis. TB of the central nervous system is seen most often in young children but
also develops in adults, especially those infected with HIV. The disease often presents subtly as
headache and slight mental changes after a prodrome of weeks of low-grade fever, malaise,
anorexia, and irritability. Since meningeal involvement is pronounced at the base of the brain,
paresis of cranial nerves (ocular nerves in particular) is a frequent finding, and the involvement
of cerebral arteries may produce focal ischemia. Lumbar puncture is the cornerstone of
diagnosis. Adjunctive dexamethasone significantly enhanced the chances of survival among
persons >14 years of age but did not reduce the frequency of neurologic sequelae. ERIC ROYD
TALAVERA, MD (TOP 1 - AUG 2014 MED BOARDS; TOPNOTCH MD) FINAL EXAM -
FEB 2015 TOPNOTCH MEDICAL BOARD PREP INTERNAL MEDICINE SUPEREXAM
For inquiries visit www.topnotchboardprep.com.ph or email us at
[email protected] TOPNOTCH MEDICAL BOARD PREP INTERNAL
MEDICINE SUPEREXAM Page 40 of 95 For inquiries visit www.topnotchboardprep.com.ph or
email us at [email protected] Item # QUESTION EXPLANATION
AUTHOR TOPNOTCH EXAM frequent finding 282 Which of the following joints is spared in
Osteoarthritis? A. Knee B. Ankle C. Cervical Vertebrae D. Proximal interphalangeal joint E. Hip
OA affects certain joints, yet spares others. Usually spared are the wrist, elbow, and ankle. Some
joints, like the ankles, may be spared because their articular cartilage may be uniquely resistant
to loading stresses ERIC ROYD TALAVERA, MD (TOP 1 - AUG 2014 MED BOARDS;
TOPNOTCH MD) FINAL EXAM - FEB 2015 283 A 40 year old female presented with
progressive weight gain and marked swelling of her lower extremities. She has no known co-
morbids. PE is normal except for the presence of a grade 2 bipedal edema. CBC, BUN,
Electrolytes and Creatinine were all normal. Her urinalysis showed a 4+ albumin, oval fat bodies
and free fat droplets. Her 24 hour urine albumin excretion was elevated at 8.6 grams. Which of
the following findings would also be expected in this patient? A. Elevated bilirubin levels B.
Elevated serum cholesterol C. Elevated serum albumin D. Elevated blood uric acid E. None of
the above Dx: Nephrotic syndrome Nephrotic syndrome classically presents with heavy
proteinuria, minimal hematuria, hypoalbuminemia, hypercholesterolemia, edema, and
hypertension. If left undiagnosed or untreated, some of these syndromes will progressively
damage enough glomeruli to cause a fall in GFR, producing renal failure. ERIC ROYD
TALAVERA, MD (TOP 1 - AUG 2014 MED BOARDS; TOPNOTCH MD) FINAL EXAM -
FEB 2015 284 Which of the following clinical features argues against the diagnosis of Irritable
Bowel Syndrome (IBS)? A. Bloody diarrhea B. Alteration in bowel habits C. Exacerbation of
abdominal pain by stress D. Resolution of abdomina pain on passage of stool E. Increased
belching or flatulence Alteration in bowel habits is the most consistent clinical feature in IBS.
Abdominal Pain is often exacerbated by eating or emotional stress and improved by passage of
flatus or stools. Patients with IBS frequently complain of abdominal distention and increased
belching or flatulence. Bleeding is not a feature of IBS unless hemorrhoids are present. ERIC
ROYD TALAVERA, MD (TOP 1 - AUG 2014 MED BOARDS; TOPNOTCH MD) FINAL
EXAM - FEB 2015 285 A 40 year old female was referred for hematologic evaluation because of
findings of splenomegaly and a markedly elevated WBC count of 180,000/uL with the following
differential count: Blasts 2%, Promyelocyte 15%, Metamyelocyte 20%, Myelocyte 10%, Bands
8%, Segmenters 30%, Lymphocytes 10% and Basophils 5%. A primary diagnosis of CML was
made by the hematologist. Which of the following treatment options for this case induces rapid
disease control by lowering the WBC count? A. Imatinib B. Cytarabine C. Doxorubicin D.
Hydroxyurea E. Stem cell transplantation Dx: CML. Initial management of patients with
chemotherapy is currently reserved for rapid lowering of WBCs, reduction of symptoms, and
reversal of symptomatic splenomegaly. Hydroxyurea, a ribonucleotide reductase inhibitor,
induces rapid disease control. ERIC ROYD TALAVERA, MD (TOP 1 - AUG 2014 MED
BOARDS; TOPNOTCH MD) FINAL EXAM - FEB 2015 286 Which of the following
statements concerning Pheochromocytoma is false? A. Hypertension is the dominant sign B.
Catecholamine crises can lead to heart failure, pulmonary edema, arrhythmias, and intracranial
hemorrhage C. Elevated plasma and urinary levels of catecholamines and metanephrines are the
cornerstone for the diagnosis D. Complete tumor removal is the ultimate therapeutic goal E.
Restriction of salt intake is a necessary preoperative preparation Because patients are volume-
constricted, liberal salt intake and hydration are necessary to avoid orthostasis. Among the
presenting symptoms, episodes of palpitations, headaches, and profuse sweating are typical and
constitute a classic triad. The dominant sign is hypertension. Classically, patients have episodic
hypertension, but sustained hypertension is also common. Catecholamine crises can lead to heart
failure, pulmonary edema, arrhythmias, and intracranial hemorrhage. ERIC ROYD TALAVERA,
MD (TOP 1 - AUG 2014 MED BOARDS; TOPNOTCH MD) FINAL EXAM - FEB 2015
TOPNOTCH MEDICAL BOARD PREP INTERNAL MEDICINE SUPEREXAM For inquiries
visit www.topnotchboardprep.com.ph or email us at [email protected]
TOPNOTCH MEDICAL BOARD PREP INTERNAL MEDICINE SUPEREXAM Page 41 of 95
For inquiries visit www.topnotchboardprep.com.ph or email us at
[email protected] Item # QUESTION EXPLANATION AUTHOR
TOPNOTCH EXAM 287 A 60 year old male sought consult due to muscle weakness. His BP
was elevated at 180/110 with an abdominal bruit noted. Blood tests revealed hypokalemia. An
arterial duplex scan was done which showed a decreased blood flow through the right renal
artery. What are the expected plasma renin activity (PRA) and aldosterone level for this case? A.
Low PRA and low aldosterone B. High PRA and low aldosterone C. High PRA and high
aldosterone D. Low PRA and high aldosterone E. Normal PRA and low aldosterone Dx:
Secondary Hyperaldosteronism (secondary to Renal Artery Stenosis). Decreased RBF will
stimulate renin release which will stimulate aldosterone release. Aldosterone promotes sodium
absorption (hence an elevated BP) and K excretion (hence hypokalemia which presented as
muscle weakness) ERIC ROYD TALAVERA, MD (TOP 1 - AUG 2014 MED BOARDS;
TOPNOTCH MD) FINAL EXAM - FEB 2015 288 The murmur of mitral regurgitation most
commonly radiates to what location? A. Axilla B. Right parasternal area C. Neck D. Epigastric
area E. Left parasternal area systolic murmur of at least grade III/VI intensity is the most
characteristic auscultatory finding in chronic severe MR. It is usually holosystolic (see Fig. 227-
5A), but as previously noted it is decrescendo and ceases in mid- to late systole in patients with
acute severe MR. The systolic murmur of chronic MR is usually most prominent at the apex and
radiates to the axilla. ERIC ROYD TALAVERA, MD (TOP 1 - AUG 2014 MED BOARDS;
TOPNOTCH MD) FINAL EXAM - FEB 2015 289 A 29 year old female presented with weigh
loss, palpitation, tremors and heat intolerance. She is tachycardic at 115/bpm with the presence
of moderate to severe proptosis. The thyroid gland is diffusely enlarged and non tender. Reflexes
are hyperactive. The rest of the examination is normal. Which of the following treatment options
may worsen her proptosis? A. PTU B. Methimazole C. Propranolol D. RAI E. Prednisone The
presence of severe ophthalmopathy requires caution, and some authorities advocate the use of
prednisone, 40 mg/d, at the time of radioiodine treatment, tapered over 2–3 months to prevent
exacerbation of ophthalmopathy. ERIC ROYD TALAVERA, MD (TOP 1 - AUG 2014 MED
BOARDS; TOPNOTCH MD) FINAL EXAM - FEB 2015 290 A "locked-in" state of preseved
consciousness with quadriplegia and cranial nerve signs suggests complete infarction of what
area in the central nervous system? A. Thalamus and Basal Ganglia B. Cerebellum and Medulla
C. Hypothalamus and Pituitary D. Pons and Lower Midbrain E. Occipital lobe of the cerebral
hemisphere The picture of complete basilar occlusion, however, is easy to recognize as a
constellation of bilateral long tract signs (sensory and motor) with signs of cranial nerve and
cerebellar dysfunction. A "locked-in" state of preserved consciousness with quadriplegia and
cranial nerve signs suggests complete pontine and lower midbrain infarction. The therapeutic
goal is to identify impending basilar occlusion before devastating infarction occurs. ERIC
ROYD TALAVERA, MD (TOP 1 - AUG 2014 MED BOARDS; TOPNOTCH MD) FINAL
EXAM - FEB 2015 291 A 56 year old female developed dysphagia to both solids and liquids
from the onset. Which of the following disorders would be most compatible with this case? A.
Schatzki ring B. Peptic Stricture C. Achalasia D. Esophageal carcinoma E. Eosinophilic
esophagitis Intermittent dysphagia that occurs only with solid food implies structural dysphagia,
whereas constant dysphagia with both liquids and solids strongly suggests a motor abnormality
such as in achalasia ERIC ROYD TALAVERA, MD (TOP 1 - AUG 2014 MED BOARDS;
TOPNOTCH MD) FINAL EXAM - FEB 2015 292 What is the only pharmacologic therapy
demonstrated to unequivocally decrease mortality rates in patients with COPD ? A. LABA B.
Corticosteroids C. Supplemental Oxygen D. Theophylline E. Ipratropium Supplemental O2 is the
only pharmacologic therapy demonstrated to unequivocally decrease mortality rates in patients
with COPD. For patients with resting hypoxemia the use of O2 has been demonstrated to have a
significant impact on mortality rate. Patients meeting these criteria should be on continual
oxygen supplementation, as the mortality benefit is proportional to the number of hours/day
oxygen is used ERIC ROYD TALAVERA, MD (TOP 1 - AUG 2014 MED BOARDS;
TOPNOTCH MD) FINAL EXAM - FEB 2015 293 A 25 year old male, known case of
Hemophilia A with a history of multiple Factor VIII concentrate transfusion, was admitted due to
bilateral knee swelling. Despite transfusion with adequate levels of Factor VIII concentrate,
levels of the said factor remain to be decreased. Which of the following statements regarding the
patient's current predicament is true? A. It is an uncommon complication of hemophilia treatment
B. It is a transient condition which will spontaneously resolve C. The laboratory test required to
confirm its presence is an aPTT mixed with a normal plasma D. Intensive replacement therapy
such as for major surgery, intracranial bleeding, or trauma are risk factors for the development of
this phenomenon E. Both C and D are correct The formation of alloantibodies to FVIII or FIX is
currently the major complication of hemophilia treatment. The prevalence of inhibitors to FVIII
is estimated to be between 5 and 10% of all cases and 20% of severe hemophilia A patients.
Inhibitors usually appear early in life, at a median of 2 years of age, and after 10 cumulative days
of exposure. However, intensive replacement therapy such as for major surgery, intracranial
bleeding, or trauma increases the risk of inhibitor formation for patients of all ages. The clinical
diagnosis of an inhibitor is suspected when patients do not respond to factor replacement at
therapeutic doses. . ERIC ROYD TALAVERA, MD (TOP 1 - AUG 2014 MED BOARDS;
TOPNOTCH MD) FINAL EXAM - FEB 2015 TOPNOTCH MEDICAL BOARD PREP
INTERNAL MEDICINE SUPEREXAM For inquiries visit www.topnotchboardprep.com.ph or
email us at [email protected] TOPNOTCH MEDICAL BOARD PREP
INTERNAL MEDICINE SUPEREXAM Page 42 of 95 For inquiries visit
www.topnotchboardprep.com.ph or email us at [email protected] Item #
QUESTION EXPLANATION AUTHOR TOPNOTCH EXAM 294 Which of the following is a
clinical manifestation of secondary syphilis? A. Condylomata lata B. Gumma C. Tabes dorsalis
D. Argyll Robertson Pupil E. Chancre In warm, moist, intertriginous areas (commonly the
perianal region, vulva, and scrotum), papules can enlarge to produce broad, moist, pink or
graywhite, highly infectious lesions [condylomata lata (see Fig. e7-20)] in 10% of patients with
secondary syphilis. ERIC ROYD TALAVERA, MD (TOP 1 - AUG 2014 MED BOARDS;
TOPNOTCH MD) FINAL EXAM - FEB 2015 295 A 56 year old male, with no known co-
morbids presented with gradual abdominal enlargement. On PE, the abdomen was tense with a
positive fluid wave and shifting dullness. The rest of the examination was essentially
unremarkable. A diagnostic paracentesis was done which showed a serum ascites albumin
gradient (SAAG) of > 1.1 g/dL. Which of the following clincial conditions are compatible with
such finding? A. TB peritonitis B. Pancreatic ascites C. Peritoneal carcinomatosis D. Portal
Hypertension E. None of the above If SAAG 1.1 almost always secondary to portal HTN ERIC
ROYD TALAVERA, MD (TOP 1 - AUG 2014 MED BOARDS; TOPNOTCH MD) FINAL
EXAM - FEB 2015 296 What is the target blood pressure in chronic kidney disease (CKD)
patients with diabetes or proteinuria of > 1g/24hr? A. 130/85 mmHg B. 125/75 mmHg C. 120/80
mmHg D. 130/80 mmHg E. 140/90 mmHg CKD patients with diabetes or proteinuria >1 g per
24 h, blood pressure should be reduced to 125/75, if achievable without prohibitive adverse
effects. ERIC ROYD TALAVERA, MD (TOP 1 - AUG 2014 MED BOARDS; TOPNOTCH
MD) FINAL EXAM - FEB 2015 297 A 66 year old female, known hypertensive and diabetic,
was brough to the ER due to a sudden onset of left sided weakness. Vital signs are as follows: BP
is 120/70, PR 97, RR 20, T 36.5. Neurologic examination showed preferential gaze to the right,
shallow left nasolabial fold, MMT of 2/5 on both left upper and lower extremites and a positive
Babinski on the left. A plain cranial CT scan was done which ruled out the presence of
intracerebral hemorrhage. To be considered a candidate for thrombolytic therapy, the onset of
symptoms to time of drug administration should be less than or equal to how many hours ? A. 8
hours B. 2 hours C. 3 hours D. 10 hours E. 14 hours The National Institute of Neurological
Disorders and Stroke (NINDS) recombinant tPA (rtPA) Stroke Study showed a clear benefit for
IV rtPA in selected patients with acute stroke. The NINDS study used IV rtPA (0.9 mg/kg to a
90-mg max; 10% as a bolus, then the remainder over 60 minutes) versus placebo in patients with
ischemic stroke within 3 hours of onset. ERIC ROYD TALAVERA, MD (TOP 1 - AUG 2014
MED BOARDS; TOPNOTCH MD) FINAL EXAM - FEB 2015 298 What is the most common
manifestation of diffuse CNS lupus? A. Headache B. Cognitive dysfunction C. Psychosis D.
Seizures E. Myelopathy The most common manifestation of diffuse CNS lupus is cognitive
dysfunction, including difficulties with memory and reasoning. ERIC ROYD TALAVERA, MD
(TOP 1 - AUG 2014 MED BOARDS; TOPNOTCH MD) FINAL EXAM - FEB 2015 299 A 23
year old male sought consult due to polyuria. PE was essentially normal. Blood tests showed a
normal FBS and HBA1C. Plasma osmolality and serum sodium were both elevated. Urinalysis
was done which showed a decreased specific gravity. Water deprivation test was done which did
not concentrate the urine. Vasopressin was then administered however it also failed to
concentrate the urine. What is the most likely diagnosis for this case? A. SIADH B. Central
diabetes insipidus C. Psychogenic polydipsia D. Nephrogenic diabetes insipidus E. None of the
above If water deprivation test resulted in concentration of urine: think of primary polydipsia. If
administration of vasopressin resulted in concentration of urine: think of central DI ERIC ROYD
TALAVERA, MD (TOP 1 - AUG 2014 MED BOARDS; TOPNOTCH MD) FINAL EXAM -
FEB 2015 300 Which of the following tests can be done to distinguish IgA nephropathy with
acute post streptococcal glomerulonephritis (APSGN)? A. KUB ultrasound B. Serum Creatinine
C. Lipid profile D. CBC E. Serum C3 level ASPGN will present with a low C3 level while it is
normal in IgA nephropathy ERIC ROYD TALAVERA, MD (TOP 1 - AUG 2014 MED
BOARDS; TOPNOTCH MD) FINAL EXAM - FEB 2015 TOPNOTCH MEDICAL BOARD
PREP INTERNAL MEDICINE SUPEREXAM For inquiries visit
www.topnotchboardprep.com.ph or email us at [email protected]
TOPNOTCH MEDICAL BOARD PREP INTERNAL MEDICINE SUPEREXAM Page 43 of 95
For inquiries visit www.topnotchboardprep.com.ph or email us at
[email protected] Item # QUESTION EXPLANATION AUTHOR
TOPNOTCH EXAM 301 the presence of rapid x+y descent in jugular venous waveform is
correlated in what cardiac condition? A. Atrioventricular dissociation B. Tricuspid stenosis C.
Tricuspid regurgitation D. Constrictive pericarditis E. Pericardial tamponade the following are
JVP abnormalities and its correlated cardiac conditions: increase JVPCongestive heart failure;
large a wave- tricuspid stenosis, RVH, pulmonary hypertension, cannon a wave - atrioventricular
dissociation, rapid x+y descent - constrictive pericarditis, kussmaul signpericardial tamponade,
RVH, and constrictive pericarditis; large fused cv wave - tricuspid regurgitation. Mayo clinic IM
concise textbook 1st ed p 37 LEAN ANGELO SILVERIO, MD (TOP 4 - AUG 2014 MED
BOARDS; TOPNOTCH MD), MD BACK-UP MIDTERM EXAM - FEB 2015 302 which of the
following is not an absolute contraindication for fibrinolysis in STEMI? A. Presence of
malignant intracranial neoplasm B. Active peptic ulcer disease C. Suspected aortic dissection D.
Concomittant ischemic stroke in evolution ( < 1hr) E. bleeding diathesis absolute
contraindication for fibrinolysis in STEMI: any prior ICH, known strucutral cerebrovascular
lesion, known malignant intracranial neoplasm, ischemic stroke within 3 months EXCEPT acute
ischemic stroke in evolution (2.5cm) should have a follow up colonoscopy or radiographic study
in 3 years. CT scan is only done for staging once colon cancer is diagnosed and not just by
presence of polyps alone. Routine colonoscopy after 10 years is recommended if the initial
screening is negative. surgery is recommended if the diagnosis is colonic malignancy. LEAN
ANGELO SILVERIO, MD (TOP 4 - AUG 2014 MED BOARDS; TOPNOTCH MD), MD
BACK-UP MIDTERM EXAM - FEB 2015 306 A 49 y/o (+) htn, (+) T2DM male was admitted
at the ICU secondary to MVA. After 5 days of hospitalization. Patient experienced nonproductive
cough and fever. Supine CXR revealed new bibasilar pulmonary infiltrates. urine antigen is
positive for legionella infection. which of the following medication is not effective in this present
case? A. Levofloxacin B. Aztreonam C. Cotrimoxazole D. Doxycycline E. Clindamycin
aztreonam, beta lactams and cephalosporin are generally ineffective in Legionella infection since
the latter is an intracellular bacteria. LEAN ANGELO SILVERIO, MD (TOP 4 - AUG 2014
MED BOARDS; TOPNOTCH MD), MD BACK-UP MIDTERM EXAM - FEB 2015 307 Which
of the following is an indication for adjunct glucocorticoids in a patient diagnosed with
Pneumocystics Jiroveci infection? A. Aa gradient of 42mmHg B. Acute Respiratory distress
syndrome C. pneumothorax D. Lack of improvement after 3 days of treatment E. all of the above
Initial antibiotic management of PCP is characterized by worsening of condition due to lysis of
organism and immune mounted response to the intracellular contents. adjunct glucocorticoid
administration is recommended in moderate or severe cases of PCP infection as determined by
room air PaO2 < 70mmHg and Aa gradient of >35mmHg. LEAN ANGELO SILVERIO, MD
(TOP 4 - AUG 2014 MED BOARDS; TOPNOTCH MD), MD BACK-UP MIDTERM EXAM -
FEB 2015 308 which of the following manuevers would be expected to increase the intensity of
murmur in hypertrophic obstructive cardiomyopathy? A. Squatting B. Valsalva C. handgrip D. A
and C E. B and C In hypertrophic cardiomyopathy, there is an assymetric hypertrophy of the
interventricular septum, creating a dynamic outflow tract obstruction. Maneuvers that will
decrease the left ventricular volume will increase the intensity of the murmur. In the choices,
only valsalva maneuver will decrease the left ventricular preload. LEAN ANGELO SILVERIO,
MD (TOP 4 - AUG 2014 MED BOARDS; TOPNOTCH MD), MD BACK-UP MIDTERM
EXAM - FEB 2015 TOPNOTCH MEDICAL BOARD PREP INTERNAL MEDICINE
SUPEREXAM For inquiries visit www.topnotchboardprep.com.ph or email us at
[email protected] TOPNOTCH MEDICAL BOARD PREP INTERNAL
MEDICINE SUPEREXAM Page 44 of 95 For inquiries visit www.topnotchboardprep.com.ph or
email us at [email protected] Item # QUESTION EXPLANATION
AUTHOR TOPNOTCH EXAM 309 Which of the following is not a potential complication of
patient who underwent massive resection of small bowel due to superior mesenteric artery
occlusion? A. Calcium oxalate renal stones B. Cholesterol gallstones C. Progressive coronary
atherosclerosis D. gastric ulcers E. none of the above this patient may develop short bowel
syndrome following resection of larger portion of the small and large intestine. Multiple factors
contribute to diarrhea and steatorrhea including gastric acid hypersecretion, increased bile acids
in the small bowel due to decreased absorption in the remaining bowel. and lactose intolerance
due to gastric acid hypersecretion. the propensity to develop oxalate renal stones is because of
the binding of free fatty acids in the colon to the calcium. thus the calcium is not free to bind to
oxalate and the latter is easily absorbed in the large intestine. The increase bile acid pool size will
yield to supersaturation of gallbladder contents leading to cholesterol gallstones. loss of
inhibitory transmitters coming from the small intestine will make the gastric acid secretion
unchecked. there is no relation between atherosclerosis and shortbowel syndrome. LEAN
ANGELO SILVERIO, MD (TOP 4 - AUG 2014 MED BOARDS; TOPNOTCH MD), MD
BACK-UP MIDTERM EXAM - FEB 2015 310 what is the earliest radiographic finding in
rheumatoid arthritis ? A. Periarticular soft tissue swelling B. joint space loss C. Periarticular
osteopenia D. Capsular thickening E. Subchondral erosions ( SIMILAR TO PREVIOUS
BOARD EXAM CONCEPT/PRINCIPLE). Harrisons 18 th chapter 321 LEAN ANGELO
SILVERIO, MD (TOP 4 - AUG 2014 MED BOARDS; TOPNOTCH MD), MD BACK-UP
MIDTERM EXAM - FEB 2015 311 the following statement is true about acromegaly except? A.
Increased risk of premalignant colon polyps and colon cancer B. A random serum level of GH is
not helpful in establishing the diagnosis C. Surgical excision is the treatment of choice D. serum
concentration of IGF1 is increased in only 50% of patient with active acromegaly E. none of the
above measuring the serum concentration of IGF-1 is the best screening test for acromegaly. It is
always increased in patient with active acromegaly. Physiologic increased is seen in the
following condition: pregnancy, adolescence, sleep apnea. LEAN ANGELO SILVERIO, MD
(TOP 4 - AUG 2014 MED BOARDS; TOPNOTCH MD), MD BACK-UP MIDTERM EXAM -
FEB 2015 312 The following condition are associated with type A gastritis except ? A. Gastric
carcinoids B. Pernicious anemia C. H pylori infection D. Atrophic gastritis E. Hypergastrinemia
type A gastritis involves the fundus and/or body of the stomach. It is associated with
autoimmune, atrophic gastritis and pernicious anemia. The serum levels of gastrin since
achlorydia develops in this patient. Gastrin is potent growth factor for the development of gastric
carcinoid tumors and gastric polyps. on the otherhand, type B gastritis involves primarily the
antrum and it is associated with H pylori infection. LEAN ANGELO SILVERIO, MD (TOP 4 -
AUG 2014 MED BOARDS; TOPNOTCH MD), MD BACK-UP MIDTERM EXAM - FEB
2015 313 which of the following is not part of the treatment goals in diabetes mellitus according
ADA? A. HBA1c 60mg/dl. E. All of the above ADA treatment goals for DM: Hba1c 40mg/dl
(men), >50mg/dl (women), TAG 60years old C. Prsence of pleural effusion D. BMI > 30 E.
BUN > 22mg% BISAP scoring is predictive of hospital mortality secondary to acute pancreatitis.
The presence of 3 or more is donates a higher risk. BUN> 22mg%, impaired mental status, SIRS
2/4, Age >60 y/o, Pleural effusion. Harrison 18th d page 2640 LEAN ANGELO SILVERIO, MD
(TOP 4 - AUG 2014 MED BOARDS; TOPNOTCH MD), MD BACK-UP MIDTERM EXAM -
FEB 2015 315 which of the following drugs have a direct toxic effect on hepatocytes ? A.
halothane B. simvastatin C. Acetaminophen D. isoniazid E. All of the above drug induced liver
injury can be categorized into two types: direct toxic effect and idiosyncratic effect. The former
is predictable, dose related pattern of injury and short time to effect duration. The most common
example is acetaminophen ( NAPQI) others are Carbon tetrachloride,
trichloroethylene,tetracycline, and amanitin. examples of drugs that causes idiosyncratic liver
injury are the following: halothane, isothane, isoniazid, statins and chlorpromazine. LEAN
ANGELO SILVERIO, MD (TOP 4 - AUG 2014 MED BOARDS; TOPNOTCH MD), MD
BACK-UP MIDTERM EXAM - FEB 2015 316 which in the following condition is not
associated with digital clubbing? A. hyperthryroidism B. Hepatocellular carcinoma C. Idiopathic
pulmonary fibrosis D. Chronic obstructive pulmonary disease E. None of the above digital
clubbing is most commonly associated with advanced lung diseases such as bronchiectasis,
cystic fibrosis, sarcoidosis, idiopathic pulmonary fibrosis, tuberculosis, empyema , and lung
abscess. extrapulmonary causes include cyanotic congenital heart disease, subacute bacterial
endocarditis, crohns disease, ulcerative colitis, and untreated hyperthyroidism. COPD does not
cause clubbing. LEAN ANGELO SILVERIO, MD (TOP 4 - AUG 2014 MED BOARDS;
TOPNOTCH MD), MD BACK-UP MIDTERM EXAM - FEB 2015 TOPNOTCH MEDICAL
BOARD PREP INTERNAL MEDICINE SUPEREXAM For inquiries visit
www.topnotchboardprep.com.ph or email us at [email protected]
TOPNOTCH MEDICAL BOARD PREP INTERNAL MEDICINE SUPEREXAM Page 45 of 95
For inquiries visit www.topnotchboardprep.com.ph or email us at
[email protected] Item # QUESTION EXPLANATION AUTHOR
TOPNOTCH EXAM 317 A 47 y/o male asymptomatic presents in your clinic with an incidental
finding of WBC count 110x109 /L, basophilia, eosinophilia and obliterated traube space.
Cytogenetic studies showed t(9,22). Which of the following is true about his condition except?
A. the standard therapy for this condition is Imatinib B. leukocyte alkaline phosphatase score is
low or zero C. increased serum Vitamin B12 level D. philadelphia chromosome is the hallmark
of this condition E. all of the above this is a classic case of CML. Characterized by increased
WBC count, granulocytes in all stages of maturation. Presence of philadelphia chromosome is
the hallmark for this condition. This is caused by translocation of bcr -abl gene. Leukocyte
alkaline phosphatase ( marker of functional activity of granulocytes) is low compared to reactive
leukocytosis. there is an increaesed Vitamin B12 level due to increased level of transcobalamin I.
Imatinib is the treatment of choice LEAN ANGELO SILVERIO, MD (TOP 4 - AUG 2014 MED
BOARDS; TOPNOTCH MD), MD BACK-UP MIDTERM EXAM - FEB 2015 318 the most
common genetic cause of End stage Renal disease in children and adolesence? A. ARPKD B.
ADPKD C. Nephronopthisis D. Medullary cystic kidney disease E. Medullary sponge kidney
Harrisons 18th ed p 2359 LEAN ANGELO SILVERIO, MD (TOP 4 - AUG 2014 MED
BOARDS; TOPNOTCH MD), MD BACK-UP MIDTERM EXAM - FEB 2015 319 which of the
following is not an recommended empiric antibiotic treatment for patients diagnosed with
Healthcare associated Pneumonia and without risk factors for MDR pathogens? A. ceftriaxone B.
meropenem C. moxifloxacin D. ampicillin/sulbactam E. none of the above recommended
empiric antibiotics for HCAP without risk factors for MDR Pathogen ( pls see table 257-1
Harrison 18th ed page 2130) are the following: ceftriaxone, moxifloxacin,levofloxacin,
ciprofloxacin, ampicillin sulbactam, and ertapenem. Harrison 18th ed p 2139 ( SIMILAR TO
PREVIOUS BOARD EXAM CONCEPT/PRINCIPLE) LEAN ANGELO SILVERIO, MD (TOP
4 - AUG 2014 MED BOARDS; TOPNOTCH MD), MD BACK-UP MIDTERM EXAM - FEB
2015 320 minimum definition of clinically important involuntary weight loss is? A. >5% loss of
body weight in 6-12 months B. >10% loss of body weight in 5 months C. >5% loss of body
weight in 1 month D. >10% loss of body weight in 6-12 months E. none of the above Harrison
18th ed pag 641. LEAN ANGELO SILVERIO, MD (TOP 4 - AUG 2014 MED BOARDS;
TOPNOTCH MD), MD BACK-UP MIDTERM EXAM - FEB 2015 321 What is the major
symptom of acute pancreatitis? A. abdominal pain B. dyspnea C. hypotension D. abdominal
tenderness Pain is characteristically steady and boring in character, located in the epigastrium
and periumbilical region and often radiates to the back, chest, flank, and lower abdomen. C & D
are not symptoms, but are signs. Harrison's Principles of Internal Medicine 18th edition, p. 2636
DEBBIE ROSE TANENGSY, MD (TOP 5 - AUG 2014 MED BOARDS; TOPNOTCH MD)
BACK-UP MIDTERM EXAM - FEB 2015 322 Supportive care measure/s that form the
hallmark in the treatment in acute pancreatitis, except: A. bowel rest B. hydration with
crystalloid C. analgesia D. pancreatic enzyme supplementation E. none of the above DEBBIE
ROSE TANENGSY, MD (TOP 5 - AUG 2014 MED BOARDS; TOPNOTCH MD) BACK-UP
MIDTERM EXAM - FEB 2015 323 The most common valvular abnormality in rheumatoid
arthritis? A. tricupid regurgitation B. mitral stenosis C. mitral regurgitation D. aortic
regurgitation Harrison's Principles of Internal Medicine 18th edition, p. 2739 DEBBIE ROSE
TANENGSY, MD (TOP 5 - AUG 2014 MED BOARDS; TOPNOTCH MD) BACK-UP
MIDTERM EXAM - FEB 2015 324 In the treatment of nongonococcal bacterial arthritis, the
following may be done, except: A. needle aspiration B. weight bearing C. intravenous
bactericidal agents D. arthrotomy Weight bearing should be avoided until signs of inflammation
have subsided, but frequent passive motion of joint is indicated to maintain full mobility.
Arthrotomy best manages septic arthritis of the hip in young children. Harrison's Principles of
Internal Medicine 18th edition, p. 2845 DEBBIE ROSE TANENGSY, MD (TOP 5 - AUG 2014
MED BOARDS; TOPNOTCH MD) BACK-UP MIDTERM EXAM - FEB 2015 325 True
regarding diabetic ketoacidosis, except: A. fluid deficit is often 3-5 L B. Insulin deficiency and
glucagon excess are necessary for its development C. Inadequate fluid intake is the underlying
cause D. initial intravenous fluid replacement: 0.9% NSS E. Bicarbonate replacement is not
necessary. Relative insulin deficiency and inadequate fluid intake are underlying causes of HHS.
HHS free water deficit: 9 - 10 L. Bicarbonate replacement often impairs heart function, decreases
tissue oxygenation, and promotes hypokalemia. Harrison's Principles of Internal Medicine 18th
edition, p. 2977-2979 DEBBIE ROSE TANENGSY, MD (TOP 5 - AUG 2014 MED BOARDS;
TOPNOTCH MD) BACK-UP MIDTERM EXAM - FEB 2015 326 This is the single best
measure of hepatic synthetic function: A. serum globulins B. serum albumin C. clotting factors
D. alkaline phosphatase E. liver span Serum albumin is not a good indicator of acute hepatic
dysfunction because of its slow turnover (T1/2: 18-20 days). Alkaline phosphatase (the liver
isoenzyme) is increased in cholestasis. Harrison's Principles of Internal Medicine 18th edition, p.
2529-2530 DEBBIE ROSE TANENGSY, MD (TOP 5 - AUG 2014 MED BOARDS;
TOPNOTCH MD) BACK-UP MIDTERM EXAM - FEB 2015 TOPNOTCH MEDICAL
BOARD PREP INTERNAL MEDICINE SUPEREXAM For inquiries visit
www.topnotchboardprep.com.ph or email us at [email protected]
TOPNOTCH MEDICAL BOARD PREP INTERNAL MEDICINE SUPEREXAM Page 46 of 95
For inquiries visit www.topnotchboardprep.com.ph or email us at
[email protected] Item # QUESTION EXPLANATION AUTHOR
TOPNOTCH EXAM 327 True of Crohn disease but not true of ulcerative colitis: A. gross blood
and mucus in stool B. systemic symptoms C. pain D. recurrence after surgery E. ANCA (+)
ANCA is more frequently associated with ulcerative colitis. Crohn - ASCA (+) Harrison's
Principles of Internal Medicine 18th edition, p. 2486 table 295-5 DEBBIE ROSE TANENGSY,
MD (TOP 5 - AUG 2014 MED BOARDS; TOPNOTCH MD) BACK-UP MIDTERM EXAM -
FEB 2015 328 True in the treatment of uric acid lithiasis, except: A. raise urine pH above 6.5 B.
lower excessive uric acid secretion C. give supplemental alkali D. low purine diet E. allopurinol
Raising the urine pH above 6.5 won't provide additional benefit but increases the risk of calcium
phosphate stone formation. Harrison's Principles of Internal Medicine 18th edition, p. 2386
DEBBIE ROSE TANENGSY, MD (TOP 5 - AUG 2014 MED BOARDS; TOPNOTCH MD)
BACK-UP MIDTERM EXAM - FEB 2015 329 The most common hematologic abnormality in
rheumatoid arthritis is: A. thrombocytopenia B. neutropenia C. thrombocytosis D. microcytic,
hypochromic anemia E. normocytic, normochromic anemia Harrison's Principles of Internal
Medicine 18th edition, p. 2740 DEBBIE ROSE TANENGSY, MD (TOP 5 - AUG 2014 MED
BOARDS; TOPNOTCH MD) BACK-UP MIDTERM EXAM - FEB 2015 330 Usually the
earliest manifestation of ophthalmopathy in Grave's Disease: A. periorbital edema B. chemosis
C. proptosis D. excess tearing E. diplopia The earliest manifestations of ophthalmopathy in
Grave's Disease are usually a sensation of grittiness, eye discomfort, and excess tearing.
Harrison's Principles of Internal Medicine 18th edition, p. 2924 DEBBIE ROSE TANENGSY,
MD (TOP 5 - AUG 2014 MED BOARDS; TOPNOTCH MD) BACK-UP MIDTERM EXAM -
FEB 2015 331 In the diagnosis of pheochromocytoma, the measurement of this is the most
sensitive and is less susceptible to false (+) increase from stress: A. 24hour urine VMA B. 24hour
urine metanephrines C. plasma metanephrine D. 24hour urine catecholamine E. plasma
catecholamine DEBBIE ROSE TANENGSY, MD (TOP 5 - AUG 2014 MED BOARDS;
TOPNOTCH MD) BACK-UP MIDTERM EXAM - FEB 2015 332 One of the following is not a
contraindication to the administration of IV rtPA for acute ischemic stroke: A. coma B. GI bleed
in the preceeding 21 days C. prior stroke in 3 months D. onset to adminstration in 41.5C B. >
106.7F C. AM temperature of >37.2C D. PM temperature of >37.5C E. none of the above A & B
pertain to hyperpyrexia. A PM temperature of 37.7C also defines a fever. Harrison's Principles of
Internal Medicine 18th edition, p. 143 DEBBIE ROSE TANENGSY, MD (TOP 5 - AUG 2014
MED BOARDS; TOPNOTCH MD) BACK-UP MIDTERM EXAM - FEB 2015 339 Causes of
hypovolemic hyponatremia include the following, except: A. primary hypoaldosteronism B.
diarrhea C. burns D. nephrotic syndrome E. none of the above Nephrotic syndrome is a cause of
hypervolemic hyponatremia. DEBBIE ROSE TANENGSY, MD (TOP 5 - AUG 2014 MED
BOARDS; TOPNOTCH MD) BACK-UP MIDTERM EXAM - FEB 2015 340 A true statement
regarding potassium level derangements: A. Oral replacement with KCl is the mainstay of
therapy for hypokalemia. B. Reduced tissue uptake is the most common cause of hyperkalemia.
C. Intravenous bicarbonate is routinely used in the treatment of hyperkalemia. D. none of the
above The use of intravenous administration should be limited to patients unable to utilize the
enteral route or in the setting of severe complications. A decrease in renal K excretion is the most
common underlying cause of hyperkalemia. Intravenous bicarbonate should be reserved for
patients with hyperkalemia concomitant metabolic acidosis. DEBBIE ROSE TANENGSY, MD
(TOP 5 - AUG 2014 MED BOARDS; TOPNOTCH MD) BACK-UP MIDTERM EXAM - FEB
2015 341 What is the significance of (+) PPD? A. Exposure to TB B. Infection with TB C.
Active TB D. Highly infectious TB JESSICA MAE SANCHEZ, MD (TOP 4 - AUG 2014 MED
BOARDS; TOPNOTCH MD) BACK-UP MIDTERM EXAM - FEB 2015 342 Why is
monotherapy with isoniazid highly discouraged in treatment of active TB? A. Increases the risk
for drug resistance B. Has not been proven effective as chemoprophylaxis C. Due to
hepatotoxicity D. Increased occurrence of peripheral neuropathy if not combined with other
medications JESSICA MAE SANCHEZ, MD (TOP 4 - AUG 2014 MED BOARDS;
TOPNOTCH MD) BACK-UP MIDTERM EXAM - FEB 2015 343 Which autoantibody is
highly specific for SLE? A. Anti-histone B. Anti-RNP C. Anti-Ro/SSA D. Anti-Sm JESSICA
MAE SANCHEZ, MD (TOP 4 - AUG 2014 MED BOARDS; TOPNOTCH MD) BACK-UP
MIDTERM EXAM - FEB 2015 344 A newly diagnosed diabetic patient was brought to the ER
because of loss of consciousness. His CBG is 35 mg/dL. He was recently started on a medication
for diabetes by their family physician. Which is the most likely medication given to him? A.
Thiazolidinedione B. Sulfonylurea C. Alpha glucosidase inhibitor D. Metformin Sulfonylureas,
especially those of the 1st generation, are notorious for causing hypoglycaemia. JESSICA MAE
SANCHEZ, MD (TOP 4 - AUG 2014 MED BOARDS; TOPNOTCH MD) BACK-UP
MIDTERM EXAM - FEB 2015 345 A 20 year old female consults due to tea-colored urine and
bipedal edema. She had recent fever and sore throat. Her BP is 150/90 and serum creatinine is 1
mg/dL. C3 level is 130 mg/dL (NV = 83-177 mg/dL). What feature would suggest IgA
nephropathy over acute post-streptococcal glomerulonephritis? A. C3 levels B. Bipedal C. If the
fever and cough occurred 16 days ago D. Her blood pressure C3 levels are decreased in acute
PSGN JESSICA MAE SANCHEZ, MD (TOP 4 - AUG 2014 MED BOARDS; TOPNOTCH
MD) BACK-UP MIDTERM EXAM - FEB 2015 346 A 45 year old lawyer came to the ER
because of headache and right sided weakness. BP was noted to be 220/120, HR 110/bpm.
Which antihypertensive agent is recommended? A. Clonidine sublingual B. Esmolol IV C.
Nicardipine IV D. Nifedipine sublingual This is a case of hypertensive emergency for which IV
antihypertensives are indicated. JESSICA MAE SANCHEZ, MD (TOP 4 - AUG 2014 MED
BOARDS; TOPNOTCH MD) BACK-UP MIDTERM EXAM - FEB 2015 347 Which of the
following is an absolute contraindication to fibrinolytic therapy? A. Suspected aortic dissection
B. Hypertension with SBP > 160 mmHg and/or DBP >90 mmHg C. Recent invasive surgical
procedure (< 40 mg/dL for males C. BP reading of 120/90 D. B and C E. All of the above Central
obesity: Waist circumference >102 cm (M), >88 cm (F) Hypertriglyceridemia: Triglycerides
≥150 mg/dL OR specific medication Low HDL cholesterol: 220 mmHg systolic and/or >110
mmHg diastolic at any time during the acute presentation B. History of a nonhemorrhagic stroke
within the last two years C. Active internal bleeding including menses D. A and B E. None of the
above BP >180mmHg/ > 110mmHg at any time during acute presentation; history of
hemorrhagic CVD at any time of nonhemorrhagic CVD within the past year; active internal
bleeding excluding menses; suspicion of aortic dissection HPIM 18th 2028 MAIRRE JAMES
GADDI, MD (TOP 4 - AUG 2013 MED BOARDS; TOPNOTCH MD) BACK-UP MIDTERM
EXAM - FEB 2015 378 Hemoptysis from tuberculosis results from: A. erosion of a blood vessel
in the wall of a cavity B. rupture of a dilated vessel in a cavity C. aspergilloma formation D. A
and B E. All of the above Hemoptysis develops in 20–30% of cases, and massive hemoptysis
may ensue as a consequence of the erosion of a blood vessel in the wall of a cavity. Hemoptysis,
however, may also result from rupture of a dilated vessel in a cavity (Rasmussen’s aneurysm) or
from aspergilloma formation in an old cavity. HPIM 18th 1345 MAIRRE JAMES GADDI, MD
(TOP 4 - AUG 2013 MED BOARDS; TOPNOTCH MD) BACK-UP MIDTERM EXAM - FEB
2015 379 Tthe following can precipitate acute decompensation in patients with CHF EXCEPT?
A. Liver failure B. NSAID use C. Infection D. Pregnancy E. No exception Factors That May
Precipitate Acute Decompensation in Patients With Chronic Heart Failure include dietary
indiscretion, myocardial ischemia/infarction, arrhythmias (tachycardia or bradycardia),
discontinuation of HF therapy, infection, anemia, initiation of medications that worsen HF:
{Calcium antagonists (verapamil, diltiazem), Beta blockers, Nonsteroidal anti-inflammatory
drugs, antiarrhythmic agents [all class I agents, sotalol (class III)] Anti-TNF antibodies}, alcohol
consumption, pregnancy, worsening hypertension and acute valvular insufficiency HPIM 18th pg
1907 MAIRRE JAMES GADDI, MD (TOP 4 - AUG 2013 MED BOARDS; TOPNOTCH MD)
BACK-UP MIDTERM EXAM - FEB 2015 380 Which of the following is/are TRUE regarding
ankylosing spondylitis (AS)? A. Associated with HLA-B25 B. Outer annular fibers are eroded
and eventually replaced by bone ultimately bridging the adjacent vertebral bodies C. NSAIDs are
the first line of pharmacologic therapy for AS. D. B and C E. All of the above Associated with
HLA B27; B - The outer annular fibers are eroded and eventually replaced by bone, forming the
beginning of a syndesmophyte, which then grows by continued endochon- dral ossification,
ultimately bridging the adjacent vertebral bodies. Ascending progression of this process leads to
the “bamboo spine.” HPIM 18th pg 2774 and 2777 MAIRRE JAMES GADDI, MD (TOP 4 -
AUG 2013 MED BOARDS; TOPNOTCH MD) BACK-UP MIDTERM EXAM - FEB 2015 381
What is the recommended duration of treatment for extrapulmonary tuberculosis? A. 6-9 months
B. 9-12 months C. 12-15 months D. 15-18 months E. 2 years SIMILAR TO PREVIOUS
BOARD EXAM CONCEPT/PRINCIPLE. Extrapulmonary tuberculosis is generally treated with
anti-Koch's regimen for 6-9 months, except for CNS tuberculosis, which is treated for 9-12
months. SCOTT RILEY ONG, MD (TOP 5 - AUG 2014 MED BOARDS; TOPNOTCH MD)
BACK-UP MIDTERM EXAM - FEB 2015 TOPNOTCH MEDICAL BOARD PREP
INTERNAL MEDICINE SUPEREXAM For inquiries visit www.topnotchboardprep.com.ph or
email us at [email protected] TOPNOTCH MEDICAL BOARD PREP
INTERNAL MEDICINE SUPEREXAM Page 52 of 95 For inquiries visit
www.topnotchboardprep.com.ph or email us at [email protected] Item #
QUESTION EXPLANATION AUTHOR TOPNOTCH EXAM 382 Which of the following ECG
findings is consistent with hyperkalemia? A. Tall, peaked T wave B. Widened QRS complex C.
Prolonged PR interval D. Appearance of sine waves E. All of the above SCOTT RILEY ONG,
MD (TOP 5 - AUG 2014 MED BOARDS; TOPNOTCH MD) BACK-UP MIDTERM EXAM -
FEB 2015 383 Which of the following acid-base abnormalities would you expect to see in a
patient sufferring from aspirin toxicity? A. High anion gap acidosis B. Metabolic alkalosis C.
Respiratory acidosis D. Respiratory alkalosis E. Both A and D SCOTT RILEY ONG, MD (TOP
5 - AUG 2014 MED BOARDS; TOPNOTCH MD) BACK-UP MIDTERM EXAM - FEB 2015
384 Which of the following drugs constitute the cornerstone of treatment for congestive heart
failure? A. ACEI and digoxin B. Furosemide and ACEI C. ACEI and beta blocker D. aspirin and
digoxin E. beta blocker and thiazide SCOTT RILEY ONG, MD (TOP 5 - AUG 2014 MED
BOARDS; TOPNOTCH MD) BACK-UP MIDTERM EXAM - FEB 2015 385 Which of the
following CNS diseases are characterized by a pathologic triad of demyelination, inflammation
and gliosis? A. Ischemic stroke B. Parkinson's disease C. Multiple sclerosis D. Guillain-Barre
syndrome E. Progressive multifocal leukoencephalopathy SIMILAR TO PREVIOUS BOARD
EXAM CONCEPT/PRINCIPLE. SCOTT RILEY ONG, MD (TOP 5 - AUG 2014 MED
BOARDS; TOPNOTCH MD) BACK-UP MIDTERM EXAM - FEB 2015 386 A 56-year old
obese male was admitted for STelevation myocardial infarction. On his 7th hospital day, he was
referred for sudden decrease in sensorium, hypotension and irregular pulses. You suspected a
reinfarction. Which of the following cardiac enzyme would confirm your diagnosis? A. CK-MB
B. Troponin I C. Lactate dehydrogenase D. AST E. Myoglobin CK-MB returns to normal in 2-3
days following a myocardial infarction. New-onset elevation may indicate reinfarction. SCOTT
RILEY ONG, MD (TOP 5 - AUG 2014 MED BOARDS; TOPNOTCH MD) BACK-UP
MIDTERM EXAM - FEB 2015 387 Among patients with prosthetic valves, which of the
following organisms is the most common cause of infective endocarditis A. Staphylococcus
aureus B. viridans streptococci C. enterococci D. Staphylococcus epidermidis E. beta-hemolytic
streptococci IV drug user and right-sided acute IE: S. aureus. Native valves and left-sided
subacute IE: viridans streptococci. Prosthetic valves: S. epidermidis. GI malignancy: enterococci.
SCOTT RILEY ONG, MD (TOP 5 - AUG 2014 MED BOARDS; TOPNOTCH MD) BACK-UP
MIDTERM EXAM - FEB 2015 388 Which of the following is the most commonly involved
organ in sarcoidosis? A. Skin B. Heart C. Lung D. Kidney E. Liver SIMILAR TO PREVIOUS
BOARD EXAM CONCEPT/PRINCIPLE. SCOTT RILEY ONG, MD (TOP 5 - AUG 2014
MED BOARDS; TOPNOTCH MD) BACK-UP MIDTERM EXAM - FEB 2015 389 Which of
the following imaging findings would be most consistent with sarcoidosis? A. Enlarged mass of
confluent lymph nodes in the anterior mediastinum B. Bilateral symmetric hilar and mediastinal
lymphadenopathy with discrete node enlargement C. Bilateral asymmetric hilar lymph node
enlargement, usually more pronounced on the right D. A conglomerate mass of lymph nodes in
the bilateral hila E. Any of the above can suggest sarcoidosis SIMILAR TO PREVIOUS
BOARD EXAM CONCEPT/PRINCIPLE. SCOTT RILEY ONG, MD (TOP 5 - AUG 2014
MED BOARDS; TOPNOTCH MD) BACK-UP MIDTERM EXAM - FEB 2015 390 Graham-
Steel murmur is characteristically heard in which of the following conditions? A. Pulmonic
regurgitation B. Tricuspid regurgitation C. Aortic regurgitation D. Mitral stenosis E. Ventricular
septal defect SCOTT RILEY ONG, MD (TOP 5 - AUG 2014 MED BOARDS; TOPNOTCH
MD) BACK-UP MIDTERM EXAM - FEB 2015 TOPNOTCH MEDICAL BOARD PREP
INTERNAL MEDICINE SUPEREXAM For inquiries visit www.topnotchboardprep.com.ph or
email us at [email protected] TOPNOTCH MEDICAL BOARD PREP
INTERNAL MEDICINE SUPEREXAM Page 53 of 95 For inquiries visit
www.topnotchboardprep.com.ph or email us at [email protected] Item #
QUESTION EXPLANATION AUTHOR TOPNOTCH EXAM 391 Among elderly patients,
which of the following is the most common cause of lower GI bleeding? A. Diverticulosis B.
Colorectal cancer C. Anal fissure D. Ulcerative colitis E. Amoebic colitis However, colorectal
cancer is the most worrisome cause of lower GI bleeding and must first be ruled out in any
elderly patient with such complaint. SCOTT RILEY ONG, MD (TOP 5 - AUG 2014 MED
BOARDS; TOPNOTCH MD) BACK-UP MIDTERM EXAM - FEB 2015 392 A 26-year old
patient presented with 1-month history of productive cough with associated weight loss and night
sweats. Chest radiograph showed minimal reticulonodular infiltrates in the right lung apex.
Sputum examinations were all negative despite adequate sputum collection. You began the
patient on a 7-day regimen of amoxicillin therapy However, symptoms persisted despite good
compliance. You then decided to start him on antiKoch's treatment. Under WHO guidelines, the
patient is classified as: A. Category I B. Category II C. Category III D. You should have ordered
TB culture first. E. You should have started the patient on another 7- day trial with azithromycin.
Category III: new smear (-) without extensive parenchymal disease SCOTT RILEY ONG, MD
(TOP 5 - AUG 2014 MED BOARDS; TOPNOTCH MD) BACK-UP MIDTERM EXAM - FEB
2015 393 Which of the following clinical features would warrant a classifcation of moderate-risk
communityacquired pneumonia? A. RR 24 breaths/min B. HR 100 beats/min C. Temperature
38.0oC D. Bilateral lung infiltrates on chest radiograph E. All of the above Criteria for moderate-
risk CAP: RR > 30, HR >125, BP < 90/60, T 40, altered mental state, decompensated
comorbidity, suspected aspiration, multilobar infiltrate, pleurall effusion, abscess formation
SCOTT RILEY ONG, MD (TOP 5 - AUG 2014 MED BOARDS; TOPNOTCH MD) BACK-UP
MIDTERM EXAM - FEB 2015 394 Which of the following laboratory parameters would
suggest pre-renal etiology for azotemia? A. BUN:creatinine ratio > 20 B. FeNa > 2% C. Urine
osmolarity < 350 mOsm/kg D. Urine:plasma creatinine ratio < 20 E. All of the above The other
parameters suggest an intrinsic renal etiology. SCOTT RILEY ONG, MD (TOP 5 - AUG 2014
MED BOARDS; TOPNOTCH MD) BACK-UP MIDTERM EXAM - FEB 2015 395 Which of
the following laboratory findings are consistent with tumor lysis syndrome? A. Hyperkalemia B.
Hyperphosphatemia C. Hyperuricemia D. Hypocalcemia E. All of the above SCOTT RILEY
ONG, MD (TOP 5 - AUG 2014 MED BOARDS; TOPNOTCH MD) BACK-UP MIDTERM
EXAM - FEB 2015 396 At what stage of chronic kidney disease will you expect to fiirst see
normocytic, normochromic anemia? A. Stage I B. Stage II C. Stage III D. Stage IV E. Stage V
Normocytic, normochomic anemia can be seen as early as stage III CKD and is universal by
stage IV. SCOTT RILEY ONG, MD (TOP 5 - AUG 2014 MED BOARDS; TOPNOTCH MD)
BACK-UP MIDTERM EXAM - FEB 2015 397 Which of the following bacteria is known as the
most common etiologic agent in spontaneous bacterial peritonitis? A. Bacteroides fragilis B.
Enterococcus faecalis C. Escherichia coli D. viridans streptococci E. Enterobacter spp. Empiric
drug of choice for SBP: cefotaxime SCOTT RILEY ONG, MD (TOP 5 - AUG 2014 MED
BOARDS; TOPNOTCH MD) BACK-UP MIDTERM EXAM - FEB 2015 398 The presence of
bifid pulse is associated with which of the following conditions? A. Cardiac tamponade B. Aortic
regurgitation C. Aortic stenosis D. Pericarditis E. Hypertrophic obstructive cardiomyopathy
Cardiac tamponade: pulsus paradoxus, pulsus alternans. Aortic stenosis: pulsus parvus et tardus.
Aortic regurgitation: wide pulse pressure. SCOTT RILEY ONG, MD (TOP 5 - AUG 2014 MED
BOARDS; TOPNOTCH MD) BACK-UP MIDTERM EXAM - FEB 2015 TOPNOTCH
MEDICAL BOARD PREP INTERNAL MEDICINE SUPEREXAM For inquiries visit
www.topnotchboardprep.com.ph or email us at [email protected]
TOPNOTCH MEDICAL BOARD PREP INTERNAL MEDICINE SUPEREXAM Page 54 of 95
For inquiries visit www.topnotchboardprep.com.ph or email us at
[email protected] Item # QUESTION EXPLANATION AUTHOR
TOPNOTCH EXAM 399 In a patient who has recently recovered from an episode of rheumatic
fever without any sequelae, prophylactic treatment with benzathin penicillin 1.2 units IM every
2-4 weeks is recommended until when? A. For 3 years after attack or until 21 years old B. For 5
years after attack or until 21 years old C. For 10 years after attack or until 21 years old D. For 10
years after attack or until 40 years old E. For 15 years after attack or until 40 years old RF
without carditis: 5 years after attack or until 21 years old. RF with mild carditis: 10 years after
attack or until 21 years old. RF with moderate to severe carditis: 10 years after attack or until 40
years old, or even lifetime SCOTT RILEY ONG, MD (TOP 5 - AUG 2014 MED BOARDS;
TOPNOTCH MD) BACK-UP MIDTERM EXAM - FEB 2015 400 Which of the following is a
posthepatic etiology for portal hypertension? A. Congenital hepatic fibrosis B. Alcoholic liver
cirrhosis C. Schistosomiasis D. Budd-Chiari syndrome E. Portal vein thrombosis Options A-C
are intrahepatic causes. Option E is a prehepatic cause. SCOTT RILEY ONG, MD (TOP 5 -
AUG 2014 MED BOARDS; TOPNOTCH MD) BACK-UP MIDTERM EXAM - FEB 2015 401
A patient came to you with recurrent anal fistula, you noted on her radiologic studies that there
are noticeable "skip lesions". Which of the following medications may be given to the patient? A.
Sulfadiazine B. Sulfasalazine C. Sulfamethoxazole D. Salicylic Acid E. Mycophenolate mofetil
SIMILAR TO PREVIOUS BOARD EXAM CONCEPT/PRINCIPLE JOSE CARLO
MASANGKAY III, MD (TOP 8 - FEB 2014 MED BOARDS; TOPNOTCH MD) DIAGNOSTIC
EXAM - AUG 2014 402 A 40 year old asymptomatic patient has just completed 1 week
treatment for CAP-moderate risk, Upon follow-up you requested a Chest Xray still with noted
lobar infiltrates. What would be your next step of management? A. Extend antimicrobial therapy
B. Change antimicrobial therapy to a broader spectrum C. Consider PTB D. Re-admit, do sputum
culture and sensitivity and do re-treatment E. None of the above Patient is already asymptomatic,
CXR findings in a CAP treated patient takes 4-12 weeks to clear. JOSE CARLO MASANGKAY
III, MD (TOP 8 - FEB 2014 MED BOARDS; TOPNOTCH MD) DIAGNOSTIC EXAM - AUG
2014 403 You encountered an ECG tracing with ST elevations in all leads. What will be your
primary consideration? A. Inferior wall MI B. High lateral wall MI C. Anterior Wall MI D.
Pancardiac MI E. None of the above Diffuse ST elevation is seen in Pericarditis. JOSE CARLO
MASANGKAY III, MD (TOP 8 - FEB 2014 MED BOARDS; TOPNOTCH MD) DIAGNOSTIC
EXAM - AUG 2014 404 A chest Xray was done in a patient with severe dyspnea, the radiologic
report stated a noticed Hampton's Hump above the diaphragm. What would be your next step of
management? A. Antibiotic therapy B. Further investigation for a primary malignancy C. Refer
to surgery for immediate removal D. Fibrinolytic therapy E. Give a bronchodilator with
Corticosteroids Hampton's Hump is a peripheral wedge shaped density on CXR, usually above
the diaphragm seen in Pulmonary embolism JOSE CARLO MASANGKAY III, MD (TOP 8 -
FEB 2014 MED BOARDS; TOPNOTCH MD) DIAGNOSTIC EXAM - AUG 2014 405
Metabolic Syndrome is defined using the following criteria, EXCEPT: A. Waist line more than
40 in in Males B. TG> 200 mg/dL C. HDL less than 40mg/dl in Males D. BP >/= 130/ >/= 85 E.
FBS >100 mg/dL TG >150 mg/dL is more proper. JOSE CARLO MASANGKAY III, MD (TOP
8 - FEB 2014 MED BOARDS; TOPNOTCH MD) DIAGNOSTIC EXAM - AUG 2014 406 A
patient with a history of RHD came in with a 2d Echocardiography result revealing an Aortic
Regurgitation, upon your physical examination you noticed that the patient's Upper extremity BP
was 140/100 while his Lower extremity BP was 100/70. This sign is termed as a: A. Hill's Sign
B. Demusset's Sign C. Quincke's Sign D. Corrigan's Sign E. Duroziez's Sign JOSE CARLO
MASANGKAY III, MD (TOP 8 - FEB 2014 MED BOARDS; TOPNOTCH MD) DIAGNOSTIC
EXAM - AUG 2014 407 Transfusion-associated Viral Hepatitis is due to: A. Hepatitis A B.
Hepatitis B C. Hepatitis C D. Hepatitis D E. Hepatitis E Although both Hepatitis B and C can be
transmitted thru blood transfusion, Hepatitis C is more associated with blood transfusions.
(SIMILAR TO PREVIOUS BOARD EXAM CONCEPT/PRINCIPLE) JOSE CARLO
MASANGKAY III, MD (TOP 8 - FEB 2014 MED BOARDS; TOPNOTCH MD) DIAGNOSTIC
EXAM - AUG 2014 TOPNOTCH MEDICAL BOARD PREP INTERNAL MEDICINE
SUPEREXAM For inquiries visit www.topnotchboardprep.com.ph or email us at
[email protected] TOPNOTCH MEDICAL BOARD PREP INTERNAL
MEDICINE SUPEREXAM Page 55 of 95 For inquiries visit www.topnotchboardprep.com.ph or
email us at [email protected] Item # QUESTION EXPLANATION
AUTHOR TOPNOTCH EXAM 408 An elderly female patient is suffering from occasional
arthralgias on her right knee, the Rheumatologist performed arthrocentesis with microscopic
findings of rhomboid and rod-like crystals with weak positive birefringence, which of the
following would be the Rheumatologist's primary consideration? A. Gout B. Pseudogout C.
Rheumatoid Arthritis D. Acute Bacterial Arthritis E. Osteoarthritis The description is compatible
with CPPD crystals of Pseudogout. JOSE CARLO MASANGKAY III, MD (TOP 8 - FEB 2014
MED BOARDS; TOPNOTCH MD) DIAGNOSTIC EXAM - AUG 2014 409 Which of the
following is NOT a risk factor for OSTEOPOROSIS? A. Family History B. s/p TAHBSO C.
Injury D. Obesity E. Age JOSE CARLO MASANGKAY III, MD (TOP 8 - FEB 2014 MED
BOARDS; TOPNOTCH MD) DIAGNOSTIC EXAM - AUG 2014 410 A patient undergoing
therapy for Crohn's Disease, had a reactivation of his Latent TB, what Drug may have been
responsible? A. Rituximab B. Sulfasalazine C. 5-ASA D. Infliximab E. Alosetron a TNF-alpha
inhibitor prevents granulation formation hence reactivation of TB occurs. JOSE CARLO
MASANGKAY III, MD (TOP 8 - FEB 2014 MED BOARDS; TOPNOTCH MD) DIAGNOSTIC
EXAM - AUG 2014 411 A patient with manifestations of Cushing's Syndrome, to determine the
source of his disease a Dexamethasone Suppression Test was done: Cortisol Levels were not
suppressed by Low dose Dexamethasone meanwhile High Dose Dexamethasone suppressed
Cortisol levels. What is the probable primary disease? A. Pituitary CS B. Adrenal CS C. Ectopic
CS D. Exogenous Steroid Use E. Addison's Disease JOSE CARLO MASANGKAY III, MD
(TOP 8 - FEB 2014 MED BOARDS; TOPNOTCH MD) DIAGNOSTIC EXAM - AUG 2014
412 This heart sound, coincides with early diastole or rapid ventricular filling, commonly seen in
a dilated cardiomyopathy: A. S1 B. S2 C. S3 D. S4 E. C and D JOSE CARLO MASANGKAY
III, MD (TOP 8 - FEB 2014 MED BOARDS; TOPNOTCH MD) DIAGNOSTIC EXAM - AUG
2014 413 Among which of the following thyroid function tests is most accurate in diagnosing
Hyperthyroidism? A. TSH B. TRH C. Total T3 D. Free T4 E. Calcitonin SIMILAR TO
PREVIOUS BOARD EXAM CONCEPT/PRINCIPLE. The free fraction of Thyroid hormines is
the "true" Thyroid level. JOSE CARLO MASANGKAY III, MD (TOP 8 - FEB 2014 MED
BOARDS; TOPNOTCH MD) DIAGNOSTIC EXAM - AUG 2014 414 To prevent Nitroglycerin
tolerance which of the following should be done? A. Increase dose of NTG B. Stop NTG for 6-8
hours a day C. Decrease Dose of NTG D. Replace NTG with Sodium Nitroprusside E. Do
Nitroglycerin desensitization a 6-8-hour NTG free period every 24 hours will prevent NTG
tolerance. JOSE CARLO MASANGKAY III, MD (TOP 8 - FEB 2014 MED BOARDS;
TOPNOTCH MD) DIAGNOSTIC EXAM - AUG 2014 415 Which of the following is not seen
in Diabetes insipidus? A. Increased Urine osmolality after water deprivation test B. Random
Urine Specific gravity of 1.005 C. Serum osmolality of 390 mOsm/L D. No response to
desmopressin E. Specific Gravity of 1.030 after HCTZ administration SIMILAR TO PREVIOUS
BOARD EXAM CONCEPT/PRINCIPLE. A: urine osmolality should not increase after water
deprivation test; B and C are both seen in DI; D and E are seen in a nephrogenic type of Diabetes
insipidus. JOSE CARLO MASANGKAY III, MD (TOP 8 - FEB 2014 MED BOARDS;
TOPNOTCH MD) DIAGNOSTIC EXAM - AUG 2014 416 The following are characteristics of
a Type I Diabetes mellitus, EXCEPT: A. Islet Amyloid Deposit on histology B. Associated with
HLA system C. Severe Glucose intolerance D. Decreased C-peptide levels E. Weak genetic
predisposition Based on USMLE step 1. JOSE CARLO MASANGKAY III, MD (TOP 8 - FEB
2014 MED BOARDS; TOPNOTCH MD) DIAGNOSTIC EXAM - AUG 2014 TOPNOTCH
MEDICAL BOARD PREP INTERNAL MEDICINE SUPEREXAM For inquiries visit
www.topnotchboardprep.com.ph or email us at [email protected]
TOPNOTCH MEDICAL BOARD PREP INTERNAL MEDICINE SUPEREXAM Page 56 of 95
For inquiries visit www.topnotchboardprep.com.ph or email us at
[email protected] Item # QUESTION EXPLANATION AUTHOR
TOPNOTCH EXAM 417 SIADH presents with the following, EXCEPT: A. Excessive water
retention B. Maybe seen in Lung Cancer C. Hyponatremia D. Serum osmolarity more than the
urine osmolarity E. Treated with Demeclocycline Serum osmolarity should be less than the urine.
JOSE CARLO MASANGKAY III, MD (TOP 8 - FEB 2014 MED BOARDS; TOPNOTCH MD)
DIAGNOSTIC EXAM - AUG 2014 418 A patient presenting with a Parathyroid adenoma,
Prolactinoma, and Insulinoma, with several stomach ulcers and a history of Urolithiasis is
presenting with what syndrome? A. MEN 1 B. MEN 2A C. MEN 2B D. Sipple's Syndrome E.
Simmond's Syndrome PPP(parathyroid, Pituitary and Pancreatic tumors) are seen in MEN 1 or
Wermer's Syndrome. JOSE CARLO MASANGKAY III, MD (TOP 8 - FEB 2014 MED
BOARDS; TOPNOTCH MD) DIAGNOSTIC EXAM - AUG 2014 419 A woman on
maintenance for Hyperthyroidism unknowingly became pregnant. After giving birth, the
Pediatrician told her that her child has Aplasia cutis congenita. What drug was responsible? A.
PTU B. Methimazole C. Propranolol D. I 131 E. Lugol's Iodine JOSE CARLO MASANGKAY
III, MD (TOP 8 - FEB 2014 MED BOARDS; TOPNOTCH MD) DIAGNOSTIC EXAM - AUG
2014 420 What stage of Pneumonia corresponds with a predominance of macrophages? A.
Edema B. Red hepatization C. Gray hepatization D. Resolution E. None of the above the
resolution phase is described with a predominance of macrophages and cessation of
inflammatory response. JOSE CARLO MASANGKAY III, MD (TOP 8 - FEB 2014 MED
BOARDS; TOPNOTCH MD) DIAGNOSTIC EXAM - AUG 2014 421 A chest radiograph
showing "water bottle" sign, suggesting a multichambered heart enlargement is most consistent
with: A. Cardiac tamponade B. Myocardial infarction C. Endocarditis D. Congestive heart failure
*IM is much like an "anything under the sun" type of exam. It will test your ability to integrate
the basic subjects (biochem, physio, ana, patho, pharma) into clinical perspective. But again, just
similar with the other subjects, it has many SIMILAR TO PREVIOUS BOARD EXAM
CONCEPT/PRINCIPLEs (not really of IM, but of physio, pharma, etc) and very few case
analysis. WEBSTER ALINDOG, MD (TOP 3 - FEB 2014 MED BOARDS; TOPNOTCH MD)
MIDTERM 1 EXAM - AUG 2014 422 The anti-TB drug which is found to be more active
against slowly replicating bacilli and is known to cause hyperuricemia in some TB patients is: A.
Isoniazid B. Rifampicin C. Pyrazinamide D. Ethambutol Must know some notable side effects of
anti-TB drugs: Hepatotoxicity (H>R>Z); GI upset, orange urine, flu-like, anemia,
thrombocytopenia (Rifampicin); Ototoxicity, vestibular impairment, local irritation due to
injection (Streptomycin); Peripheral neuropathy, psychosis, convulsion (Isoniazid); Optic neuritis
(Ethambutol) WEBSTER ALINDOG, MD (TOP 3 - FEB 2014 MED BOARDS; TOPNOTCH
MD) MIDTERM 1 EXAM - AUG 2014 423 The recognized primary mediator of bronchial
asthma: A. Histamine B. Acetylcholine C. Dust mites D. Slow reacting substances of anaphylaxis
SRSA are leukotrienes C4, D4, E4. Histamine, although associated with hypersensitivity states,
is not a mediator of asthma, and hence antihistamines are not used as reliever or controller for
asthma. Dust mites are not mediators, but triggers. WEBSTER ALINDOG, MD (TOP 3 - FEB
2014 MED BOARDS; TOPNOTCH MD) MIDTERM 1 EXAM - AUG 2014 424 Of the
following clinical findings, which will differentiate diffuse systemic scleroderma from the
CREST variant? A. Hands turning painful and white or blue in the cold B. Distal skin thickening
C. Renal involvement D. Esophageal dysmotility E. All of the above The CREST syndrome
includes subcutaneous calcinosis, Reynaud's phenomenon (choice A), esophageal dysmotility,
sclerodactyly and telangiectasia (also present in the systemic type but less common). Another
defining finding in systemic sclerosis is that its pulmonary involvement is most likely to be
severe than in CREST. WEBSTER ALINDOG, MD (TOP 3 - FEB 2014 MED BOARDS;
TOPNOTCH MD) MIDTERM 1 EXAM - AUG 2014 425 This urinalysis finding is
pathognomonic of an endstage renal disease; a very poor prognostic sign: A. Hyaline casts B.
"Coffin lid" crystals C. Broad waxy casts D. Oil droplets Broad waxy casts suggest presence of
significant tubular degeneration and implies a poor prognosis. Hyaline casts are composed
almost entirely of Tamm-Horsfall protein and a 0-2/lpf count is considered normal; increased
amounts are found during exercise, fever, dehydration, and heart failure. Coffin lid crystals are
typical of triple phosphate. Oil droplets on the other hand, may only be artefactual during
urinalysis. WEBSTER ALINDOG, MD (TOP 3 - FEB 2014 MED BOARDS; TOPNOTCH MD)
MIDTERM 1 EXAM - AUG 2014 TOPNOTCH MEDICAL BOARD PREP INTERNAL
MEDICINE SUPEREXAM For inquiries visit www.topnotchboardprep.com.ph or email us at
[email protected] TOPNOTCH MEDICAL BOARD PREP INTERNAL
MEDICINE SUPEREXAM Page 57 of 95 For inquiries visit www.topnotchboardprep.com.ph or
email us at [email protected] Item # QUESTION EXPLANATION
AUTHOR TOPNOTCH EXAM 426 The Chinese restaurant syndrome, a condition characterized
by burning and tightness in the upper body accompanied by diaphoresis, nausea, flushing and
cramps is attributed to: A. Monosodium glutamate intoxication B. Puffer fish poisoning C. Fried
rice (Yangchow) contaminated with enteric bacteria D. Rancid cooking oil poisoning WEBSTER
ALINDOG, MD (TOP 3 - FEB 2014 MED BOARDS; TOPNOTCH MD) MIDTERM 1 EXAM
- AUG 2014 427 A 26 year-old male came in for consult due to painful urethral discharge. He
admits sexual promiscuity and practice of unsafe sex. Which of the following infectious agents
most likely accounts for his condition? A. Treponema B. Chlamydia C. Trichomonas D.
Granuloma inguinale WEBSTER ALINDOG, MD (TOP 3 - FEB 2014 MED BOARDS;
TOPNOTCH MD) MIDTERM 1 EXAM - AUG 2014 428 Which of the following statements
about insulin synthesis and secretion is not true? A. Blood glucose level of >70 mg/dl stimulates
insulin synthesis. B. Glucose phosphorylation by glucokinase is the rate-limiting step in glucose-
regulated insulin secretion. C. Glucose is transported to the pancreatic beta cells by an active
glucose transport. D. All of the above E. None of the above Glucose stimulation of insulin
secretion starts with entry of glucose molecules into pancreatic beta cells by means of a
facilitative glucose transporter (GLUT 2). Options A and B are true. (HPIM, DM Chapter 18th
edition) WEBSTER ALINDOG, MD (TOP 3 - FEB 2014 MED BOARDS; TOPNOTCH MD)
MIDTERM 1 EXAM - AUG 2014 429 A 32 year-old man was found to have chronic fatigue and
AST elevations 4x the upper limit. He had a series of blood transfusions 4 years ago in a
secondary hospital in a remote island due to vehicular trauma. Vascular biopsy was done and
showed polyarteritis. Which of the following is the most likely cause of his condition? A.
Hepatitis A B. Hepatitis B C. Hepatitis C D. Hepatitis D Hepatitis B is associated with
polyarteritis (PAN). WEBSTER ALINDOG, MD (TOP 3 - FEB 2014 MED BOARDS;
TOPNOTCH MD) MIDTERM 1 EXAM - AUG 2014 430 A 63 year-old previous MI male
patient was seen at the emergency room after experiencing severe abdominal pain that is out of
proportion from physical findings. On exam he has soft abdomen with hypoactive bowels. An
abdominal radiograph reveals 'thumbprinting" or bowel loops. Which of the following is
consistent with the most likely diagnosis? A. Metabolic alkalosis is also present. B. CBC will
show leukopenia but with relative increase in bands. C. There is involvement of the superior
mesenteric artery. D. Barium enema can be used to reverse the condition. E. None of the above.
This is a case of acute mesenteric ischemia which is more common in patients with history or
evidence of arterial vascular compromise including MI and peripheral arterial diseases. The
superior mesenteric artery is usually affected. "Thumbrpinting" or bowel looping is a common
finding in Xrays. Patients will often have abdominal PE findings that are relatively benign
compared with the patient's symptoms. Metabolic acidosis will usually be present as well.
WEBSTER ALINDOG, MD (TOP 3 - FEB 2014 MED BOARDS; TOPNOTCH MD)
MIDTERM 1 EXAM - AUG 2014 431 Which of the following drugs can cause acute
pericarditis? A. Isoniazid B. Hydralazine C. Procainamide D. All of the above WEBSTER
ALINDOG, MD (TOP 3 - FEB 2014 MED BOARDS; TOPNOTCH MD) MIDTERM 1 EXAM
- AUG 2014 432 The best way to confirm the diagnosis of aortic stenosis is by performing: A.
An electrocardiogram B. An echocardiogram C. An exercise stress test D. A radionuclide
ventriculogram Echo study with Doppler can detect the valvular problem, quantify its severity
and assess left ventricular function. Although a ventriculogram can tell about ventricular
function, it cannot assess disease severity. ECG is non-specific whereas a stress test is
contraindicated in symptomatic patients. WEBSTER ALINDOG, MD (TOP 3 - FEB 2014 MED
BOARDS; TOPNOTCH MD) MIDTERM 1 EXAM - AUG 2014 TOPNOTCH MEDICAL
BOARD PREP INTERNAL MEDICINE SUPEREXAM For inquiries visit
www.topnotchboardprep.com.ph or email us at [email protected]
TOPNOTCH MEDICAL BOARD PREP INTERNAL MEDICINE SUPEREXAM Page 58 of 95
For inquiries visit www.topnotchboardprep.com.ph or email us at
[email protected] Item # QUESTION EXPLANATION AUTHOR
TOPNOTCH EXAM 433 A 60 year-old hypertensive male patient came in for regular follow-up.
He has no new complaints except for easy fatigability and weakness. On examination, there is no
hepatomegaly or jaundice but there is pallor. CBC was ordered showing a hemoglobin of 9 g/dl
with MCV less than 80 fl. Liver function tests and iron studies were all normal. The next
procedure that should be performed is: A. Colonoscopy B. Bone marrow aspirate C. Bone scan
D. Liver biopsy Any elderly male patient, who has anemia especially if its microcytic, and does
not have a demonstrable source of anemia warrants investigation for a colonic mass or tumor (for
occult blood loss). And in this case, the best test will be a colonoscopy. WEBSTER ALINDOG,
MD (TOP 3 - FEB 2014 MED BOARDS; TOPNOTCH MD) MIDTERM 1 EXAM - AUG 2014
434 A liver function panel shows: NORMAL AST, ALT, and LDH; LOW total protein and
albumin; HIGH bilirubin and ammonia. The most likely hepatic condition based on these lab
results pattern is: A. Hepatitis B. Biliary obstruction C. Passive hepatic congestion D. Cirrhosis
E. None of the above Cirrhosis is the most consistent. There is considerable destruction of
hepatocytes in cirrhosis which can lead to "normal" hepatic enzymes (AST, ALT) in the blood, in
contrast to elevated levels as seen when there is on going destruction of liver cells (hepatitis).
The destruction of hepatocytes also leads to diminished synthesizing functions of the liver (low
proteins/albumin, low coagulation factors); similarly, its ability to detoxify and excrete wastes is
also compromised (high bilirubin and ammonia). WEBSTER ALINDOG, MD (TOP 3 - FEB
2014 MED BOARDS; TOPNOTCH MD) MIDTERM 1 EXAM - AUG 2014 435 Evidence of
pulmonary embolism may also be seen through a chest radiograph. A sign where there is a
peripheral wedge-shaped density above the diaphragm is known as: A. Westermark's B. Palla's
sign C. Humpton's D. McConnell's E. WEBSTER ALINDOG, MD (TOP 3 - FEB 2014 MED
BOARDS; TOPNOTCH MD) MIDTERM 1 EXAM - AUG 2014 436 Which of the following
pleural fluid analysis findings is consistent with a transudative effusion? A. PF pro-BNP of
>1500 pg/ml B. Loculated PF C. PF pH 10mmHg with inspiration. B. It is the beat to beat
variability of pulse amplitude seen in severe LV systolic heart failure. C. It refers to posterior calf
pain on active dorsiflexion of the foot againsts resistance suggestive of DVT. D. all of the above
E. none of the above B. Pulsus alternans; C. Homan's sign JAN CHARMAINE PALOMAR, MD
(TOP 9 - FEB 2014 MED BOARDS; TOPNOTCH MD) BACK-UP MIDTERM EXAM AUG
2014 502 Imelda, a 76 year old vendor went to visit your clinic complaining of dizziness. Vital
signs revealed BP=150/110mmHg, HR=78bpm; RR=20rpm; and Temperature=36.8C. Classify
hypertension of patient: A. Normal B. Pre-hypertension C. Stage 1 hypertension D. Stage 2
hypertension E. Isolated Systolic hypertension Normal 160/>100mmHg; Isolated Systolic
hypertension=>140/88cm in a female B. Triglyceride >100mg/dL C. HDL 102cm in males;
TAG>150mg/dL or use of specific med; low HDL; BP >130/85mmHg; FBS>100mg/dL or
previously diagnosed T2DM JAN CHARMAINE PALOMAR, MD (TOP 9 - FEB 2014 MED
BOARDS; TOPNOTCH MD) BACK-UP MIDTERM EXAM AUG 2014 TOPNOTCH
MEDICAL BOARD PREP INTERNAL MEDICINE SUPEREXAM For inquiries visit
www.topnotchboardprep.com.ph or email us at [email protected]
TOPNOTCH MEDICAL BOARD PREP INTERNAL MEDICINE SUPEREXAM Page 66 of 95
For inquiries visit www.topnotchboardprep.com.ph or email us at
[email protected] Item # QUESTION EXPLANATION AUTHOR
TOPNOTCH EXAM 504 A 59 year old man was rushed to the emergency room because of
severe chest pain described as heaviness . Pain radiates to left shoulders and arm. What ancillary
procedure should you request? A. Stress testing B. 2D Echo C. 12- lead ECG D. CBC with
platelet E. CT scan of the chest JAN CHARMAINE PALOMAR, MD (TOP 9 - FEB 2014 MED
BOARDS; TOPNOTCH MD) BACK-UP MIDTERM EXAM AUG 2014 505 This drug acts as
an HMG-CoA reductase inhibitor which results to plaque stabilization, lowering of LDL
cholesterol and TAG and raising HDL cholesterol: A. Fenofibrate B. Clopidogrel C. Metoprolol
D. Nifedipine E. Atorvastatin SIMILAR TO PREVIOUS BOARD EXAM
CONCEPT/PRINCIPLE JAN CHARMAINE PALOMAR, MD (TOP 9 - FEB 2014 MED
BOARDS; TOPNOTCH MD) BACK-UP MIDTERM EXAM AUG 2014 506 In which of the
following patients is Coronary artery bypass grafting indicated? A. Elena, 53 years old, with 3-
vessel coronary artery disease B. Mario, 47 years old with 70% occlusion of the left anterior
descending and left circumflex artery C. Inang, 61 years old with stenosis of the left main
coronary artery D. All of the above E. None of the above IM platinum 55 JAN CHARMAINE
PALOMAR, MD (TOP 9 - FEB 2014 MED BOARDS; TOPNOTCH MD) BACK-UP
MIDTERM EXAM AUG 2014 507 The following patients cannot be given streptokinase and
tissue plasminogen activators, except: A. Riza, 49 years old, complaining of severe chest
heaviness, with ST elevation in leads II, III, AVF, day 2 of menses B. Minda, 67 years old,
suspected aortic dissection C. Roel, 52 year old with blood pressure of 190/120mmHg at the ER
D. Rommel, 74 years old with history of hemorrhagic stroke in the basal ganglia 20 years ago E.
Sanse, 67 years old, with history of ischemic stroke in the thalamus 8 months ago Absolute
contraindications to thrombolysis: any prior intracranial hemorrhage, non-hemorrhagic stroke or
other CV event withn the past year, marked hypertension (SBP>180 or DBP>110) at any time
during the acute presentation, suspected aortic dissection, and active bleeding or bleeding
diathesis (excluding menses) JAN CHARMAINE PALOMAR, MD (TOP 9 - FEB 2014 MED
BOARDS; TOPNOTCH MD) BACK-UP MIDTERM EXAM AUG 2014 508 The following
clinical findings can be appreciated in a patient with aortic regurgitation, except: A. De Musset
sign B. Gallavardin effect C. Corrigan's pulse D. Duroziez sign E. Austin Flint murmur A. De
Musset sign - jarring of the body and bobbing of the head with each systole in severe AR B.
Gallavardin effect - in AS, low-pitched midsystolic ejection murmur at 2nd R ICS that may be
transmitted to the apex resembling murmur of MR C. Corrigan's pulse - water hammer pulse D.
Duroziez sign - To and fro murmur when femoral artery is compressed E. Austin Flint murmur-
soft low-pitched rumbling mid to late diastolic murmur JAN CHARMAINE PALOMAR, MD
(TOP 9 - FEB 2014 MED BOARDS; TOPNOTCH MD) BACK-UP MIDTERM EXAM AUG
2014 509 Jemima is a 23 year old sales clerk who developed fever days prior to consult with
associated cough, colds and difficulty of breathing. The impression was Moderate Risk
Pneumonia, what is the expected chest exam findings? A. Resonant to percussion, normal
fremitus, no adventitious sounds noted B. Hyperresonant to percussion, decreased fremitus, with
occassional wheezing C. Dull to percussion, Increased fremitus, with bibasal crackles D. Dull to
percussion, decreased fremitus, with pleural friction rub E. Resonant to percussion, normal
fremitus, with wheezes A. Resonant to percussion, normal fremitus, no adventitious sounds noted
- Normal B. Hyperresonant to percussion, decreased fremitus, with occassional wheezing-COPD
C. Dull to percussion, Increased fremitus, with bibasal crackles -Pneumonia D. Dull to
percussion, decreased fremitus, with pleural friction rub-Pleural effusion E. Resonant to
percussion, normal fremitus, with wheezes-Asthma JAN CHARMAINE PALOMAR, MD (TOP
9 - FEB 2014 MED BOARDS; TOPNOTCH MD) BACK-UP MIDTERM EXAM AUG 2014
510 This refers to a previously treated pulmonary tuberculosis patient who has been declared
cured and is now diagnosed with culture positive tuberculosis: A. New B. Relapse C. Failure D.
Return after default E. Transfer in (IM platinum 99) Failure- a patient, who while on treatment, is
sputum smear positive at 5 months or later during the course of treatment; Return after default- a
patient who returns to treatment with positive bacteriology following interruption of treatment
for 2 months or more; Transfer in - patient who was transferred from another facility with proper
referral slip to continue treatment -IM 99 JAN CHARMAINE PALOMAR, MD (TOP 9 - FEB
2014 MED BOARDS; TOPNOTCH MD) BACK-UP MIDTERM EXAM AUG 2014
TOPNOTCH MEDICAL BOARD PREP INTERNAL MEDICINE SUPEREXAM For inquiries
visit www.topnotchboardprep.com.ph or email us at [email protected]
TOPNOTCH MEDICAL BOARD PREP INTERNAL MEDICINE SUPEREXAM Page 67 of 95
For inquiries visit www.topnotchboardprep.com.ph or email us at
[email protected] Item # QUESTION EXPLANATION AUTHOR
TOPNOTCH EXAM 511 The presence of this antigen determines Rh positivity: A. D antigen B.
d antigen C. C antigen D. c antigen E. H antigen JAN CHARMAINE PALOMAR, MD (TOP 9 -
FEB 2014 MED BOARDS; TOPNOTCH MD) BACK-UP MIDTERM EXAM AUG 2014 512
What is the single best test to request in a patient suspected of hyperthyroidism? A. T3, T4 B.
TSH C. FT3 D. Thyroglobulin E. TSI TSH is supressed. IM platinum 198 JAN CHARMAINE
PALOMAR, MD (TOP 9 - FEB 2014 MED BOARDS; TOPNOTCH MD) BACK-UP
MIDTERM EXAM AUG 2014 513 Arterial blood gas result of a patient at the ER reads pH of
7.10, pCO2=68mmHg and plasma bicarbonate=32mmol/L. Interpret: A. Respiratory acidosis
with renal compensation B. Metabolic acidosis with respiratory compensaton C. Respiratory
alkalosis with renal compensation D. Metabolic alkalosis with respiratory compensaton E. None
of the above JAN CHARMAINE PALOMAR, MD (TOP 9 - FEB 2014 MED BOARDS;
TOPNOTCH MD) BACK-UP MIDTERM EXAM AUG 2014 514 This management yields the
quickest resolution of the hyperthyroid state, however, it leads to hypothyroidism requiring
lifelong thyroid replacement therapy: A. Propylthiouracil B. Methimazole C. Radioactive Iodine
therapy D. Propranolol E. None of the above RAI damages gland thru cytotoxic effect. Absolute
contraindications: pregnancy and breast feeding JAN CHARMAINE PALOMAR, MD (TOP 9 -
FEB 2014 MED BOARDS; TOPNOTCH MD) BACK-UP MIDTERM EXAM AUG 2014 515
The following are chronic macrovascular complications of diabetes mellitus, except: A. Diabetic
ketoacidosis B. Retinopathy C. Neuropathy D. Nephropathy E. All of the above *Acute
complications of DM- DKA, HHS; *Chronic microvascular complications-Retinopathy,
Neuropathy, Nephropathy; *Chronic macrovascular complications- CAD, PAD, Cerebrovascular
disease JAN CHARMAINE PALOMAR, MD (TOP 9 - FEB 2014 MED BOARDS;
TOPNOTCH MD) BACK-UP MIDTERM EXAM AUG 2014 516 Minda, 36 year old G2P1 25
weeks AOG, come to your clinic for follow up check up. You diagnosed her to have gestational
diabetes last month for which you prescribed Metformin 500mg tab once a day. You want to
know the glycemic status over the prior 2 weeks. What test should you request? A. FBS B. 2
hour OGTT C. Random blood sugar D. HbA1C E. Fructosamine assay HbA1C- reflects the
glycemic status over the prior 3 months; Fructosamine assay- reflects the glycemic status over
the prior 2 weeks JAN CHARMAINE PALOMAR, MD (TOP 9 - FEB 2014 MED BOARDS;
TOPNOTCH MD) BACK-UP MIDTERM EXAM AUG 2014 517 These antacids can cause
diarrhea and constipation respectively: A. Magnesium hydroxide and aluminum hydroxide B.
Aluminum hydroxide and magnesium hydroxide C. Cimetidine and ranitidine D. Ranitidine and
cimetidine E. None of the above JAN CHARMAINE PALOMAR, MD (TOP 9 - FEB 2014
MED BOARDS; TOPNOTCH MD) BACK-UP MIDTERM EXAM AUG 2014 518 What is the
gold standard in the diagnosis of Typhoid fever? A. Blood culture B. Stool culture C. Urine
culture D. Serology E. Bone marrow culture Blood culture is the gold standard for the diagnosis
of typhoid fever JAN CHARMAINE PALOMAR, MD (TOP 9 - FEB 2014 MED BOARDS;
TOPNOTCH MD) BACK-UP MIDTERM EXAM AUG 2014 519 What is the level of dopamine
in Parkinson's Disease and in Schizophrenia, respectively? A. Increased, Decreased B.
Decreased, Increased C. Increased, Increased D. Decreased,Decreased E. None of the above Im
platinum 353 JAN CHARMAINE PALOMAR, MD (TOP 9 - FEB 2014 MED BOARDS;
TOPNOTCH MD) BACK-UP MIDTERM EXAM AUG 2014 TOPNOTCH MEDICAL BOARD
PREP INTERNAL MEDICINE SUPEREXAM For inquiries visit
www.topnotchboardprep.com.ph or email us at [email protected]
TOPNOTCH MEDICAL BOARD PREP INTERNAL MEDICINE SUPEREXAM Page 68 of 95
For inquiries visit www.topnotchboardprep.com.ph or email us at
[email protected] Item # QUESTION EXPLANATION AUTHOR
TOPNOTCH EXAM 520 What is the hallmark sign of liver disease and is the most reliable
marker of severity? A. Fatigue B. Abdominal distention C. Elevated ALT D. Jaundice E. Right
upper quadrant tenderness Jaundice is hallmark of liver disease and the most reliable marker of
severity Fatigue- most common and most characteristic symptom of liver disease -IM platinum
267 JAN CHARMAINE PALOMAR, MD (TOP 9 - FEB 2014 MED BOARDS; TOPNOTCH
MD) BACK-UP MIDTERM EXAM AUG 2014 521 A 43 year old hypertensive man comes to
the physician because of headache, blurred vision, and confusion for 2 days. He has not had
weakness or numbness, difficulty ambulating, chest pain or discomfort, diplopia, or vertigo.
Blood pressure is 190/120 mmHg on presentation. The neck is supple. Fundoscopic examination
shows enlargement of optic discs with indistinct border. Blood vessels near the discs are
indistinct. The lungs are clear to auscultation. There is an S4 with no murmur. Neurologic
examination shows no abnormalities. Which of the following is the most likely diagnosis? A)
Cerebral infarction B) Malignant hypertension C) Subarachnoid hemorrhage D) Transient
ischemic attack MIGUEL RAFAEL RAMOS, MD (TOP 3 - FEB 2012 MED BOARDS;
TOPNOTCH MD) MIDTERM 1 EXAM - FEB 2013 522 A 27 year old woman is brought to the
physician because of a 3 days history of visual loss and aching discomfort of her right eye.
Examination shows markedly reduced visual acuity on the right; the peripheral visual fields are
full to confrontation. Color vision is decreased in the right eye. The right eye does not react to
direct light but has a normal consensual response. Fundoscopic examination shows no
abnormalities. She has an ataxic gait. Babinski sign is present bilaterally. Which of the following
is the most appropriate next step in diagnosis? A) Carotid ultrasonography B) Visual evoked
potentials C) CT scan of the head with contrast D) MRI of the brain with contrast MIGUEL
RAFAEL RAMOS, MD (TOP 3 - FEB 2012 MED BOARDS; TOPNOTCH MD) MIDTERM 1
EXAM - FEB 2013 523 A 52 year old man comes to the physician because he has had a 14kg
weight loss during the past 6 months. He has noticed oily, floating stools during the past 2
months. He received the diagnosis of acute pancreatitis 2 years ago and has had 1 to 3 hour
episodes of severe abdominal pain since ten. The patient is a 30 year pack smoker. He is an
alcoholic but has been abstinent for the past 2 years. The abdomen is scaphoid with mild diffuse
tenderness. The liver edge is firm and is palpated 2 cm below the right costal margin.
Laboratories show Amylase 90 (slightly increased), Lipase 43 (normal), alkaline phosphatase
120 (normal), AST 23, ALT 29. Which of the following is the most appropriate step in
management? A) Dietary supplementation with multivitamins and iron B) Insulin therapy C)
Pancreatic enzyme replacement therapy D) Parenteral nutrition MIGUEL RAFAEL RAMOS,
MD (TOP 3 - FEB 2012 MED BOARDS; TOPNOTCH MD) MIDTERM 1 EXAM - FEB 2013
524 A 42 year old woman comes to the physician because of progressive shortness of breath
during the past 6 months. She now has to rest three or four times when climbing one flight of
stairs. She is a 30 pack year smoker. She has a chronic nonproductive cough and has wheezing
controlled with an albuterol inhaler two to three times weekly. Arterial blood gas analysis reveals
pH 7.37 pCO2 48 pO2 62 O2 sat 92%. Her FEV1 is 75% of predicted, and total lung capacity is
50% of predicted. Which of the following is the most likely diagnosis? A) Chronic obstructive
pulmonary disease B) Congestive heart failure C) Interstitial pneumonia D) Restrictive lung
disease MIGUEL RAFAEL RAMOS, MD (TOP 3 - FEB 2012 MED BOARDS; TOPNOTCH
MD) MIDTERM 1 EXAM - FEB 2013 TOPNOTCH MEDICAL BOARD PREP INTERNAL
MEDICINE SUPEREXAM For inquiries visit www.topnotchboardprep.com.ph or email us at
[email protected] TOPNOTCH MEDICAL BOARD PREP INTERNAL
MEDICINE SUPEREXAM Page 69 of 95 For inquiries visit www.topnotchboardprep.com.ph or
email us at [email protected] Item # QUESTION EXPLANATION
AUTHOR TOPNOTCH EXAM 525 An asymptomatic 47 year old woman comes to the
physician for a routine health maintenance examination. She has no history of rheumatic fever.
She takes no medications. Her pulse is 70 bpm, and blood pressure is 150/60 mmHg. A grade 2/6
decrescendo murmur that begins after S2 is heard at the sternal border. Which of the following is
the most likely diagnosis? A) Aortic valve insufficiency B) Aortic valve stenosis C) Mitral valve
regurgitation D) Mitral valve stenosis MIGUEL RAFAEL RAMOS, MD (TOP 3 - FEB 2012
MED BOARDS; TOPNOTCH MD) MIDTERM 1 EXAM - FEB 2013 526 A 37 year old woman
comes to the physician because of a 3 week history of discomfort in both knees. She is an avid
runner, 15 pack year smoker, with a BMI of 31. Her parents both have mild osteoarthritis. The
patient asks how she can decrease her risk for osteoarthritis in the future. Which of the following
is the most appropriate recommendation? A) Avoidance of high-impact physical activities B)
Smoking cessation C) Weight loss D) Daily use of calcium supplement MIGUEL RAFAEL
RAMOS, MD (TOP 3 - FEB 2012 MED BOARDS; TOPNOTCH MD) MIDTERM 1 EXAM -
FEB 2013 527 A 24 year old man is brought to the emergency department comatose after he
sustained severe head trauma in a head on motor vehicle collision. His medical history is
unknown. Over the next hour, he receives 80 mL of 0.45% saline and has a urine output of 900
mL. Laboratory studies show Na 147, glucose 124, osmolality 294, urine specific gravity 1.001.
A CT scan of the head shows scattered contusions and a subarachnoid hemorrhage. Which of the
following is the most likely cause of this patient’s increased urinary output? A) Syndrome of
inappropriate secretion of ADH B) Traumatic nephropathy C) Hypernatremia D) Diabetes
insipidus MIGUEL RAFAEL RAMOS, MD (TOP 3 - FEB 2012 MED BOARDS; TOPNOTCH
MD) MIDTERM 1 EXAM - FEB 2013 528 A 21-year-old African American college student has
had increasing fatigue over the past 3 weeks. Since an episode of cystitis treated with
trimethoprimsulfamethoxazole 3 weeks ago, he has been unable to keep up with his physical
education classes. For 6 months, he has been following a vegetarian diet that has been supervised
by student health services. Examination shows no abnormalities. His hemoglobin level is 10
g/dL, mean corpuscular volume is 85 μm3, and reticulocyte count is 15%. Which of the
following is the most likely cause of anemia in this patient? A) Anemia secondary to infection B)
Antibiotic therapy C) Gastrointestinal blood loss D) Sickle cell disease MIGUEL RAFAEL
RAMOS, MD (TOP 3 - FEB 2012 MED BOARDS; TOPNOTCH MD) MIDTERM 1 EXAM -
FEB 2013 529 A 28-year-old woman at 28 weeks' gestation reports excessive fatigability and
dyspnea. Her blood pressure is 118/74 mm Hg, pulse is 110/min and regular, and lungs are clear
to auscultation. The cardiac apex is not palpable. S1 is loud, and there is a sharp sound after S2.
A low-frequency diastolic murmur is heard at the apex that increases in intensity before S1.
Which of the following is the most likely diagnosis? A) Aortic regurgitation B) Ebstein's
anomaly C) Mitral regurgitation D) Mitral stenosis MIGUEL RAFAEL RAMOS, MD (TOP 3 -
FEB 2012 MED BOARDS; TOPNOTCH MD) MIDTERM 1 EXAM - FEB 2013 TOPNOTCH
MEDICAL BOARD PREP INTERNAL MEDICINE SUPEREXAM For inquiries visit
www.topnotchboardprep.com.ph or email us at [email protected]
TOPNOTCH MEDICAL BOARD PREP INTERNAL MEDICINE SUPEREXAM Page 70 of 95
For inquiries visit www.topnotchboardprep.com.ph or email us at
[email protected] Item # QUESTION EXPLANATION AUTHOR
TOPNOTCH EXAM 530 A 42-year-old woman comes to the physician for evaluation of
persistently increased blood pressures. At her last two office visits during the past 3 months, her
blood pressure has ranged between 150– 170/105–115 mm Hg. During this period, she has had
occasional headaches. In addition, she has had an increased urine output over the past 6 weeks
that she attributes to a diet high in sodium. She is otherwise healthy and takes no medications.
Her blood pressure today is 168/115 mm Hg, pulse is 68/min, and respirations are 14/min.
Funduscopic examination shows mild arteriovenous nicking. The point of maximal impulse is
not displaced. There is no edema, abdominal bruits, or masses. Serum studies show Na 144, Cl
90, K 2.9, HCO3 32, BUN 20, Creatinine 1.2. Which of the following is the most likely
underlying cause of this patient's hypertension? A) Autonomous production of aldosterone B)
Catecholamine-producing tumor C) Decreased arterial distensibility caused by atherosclerosis D)
Excess production of atrial natriuretic peptide MIGUEL RAFAEL RAMOS, MD (TOP 3 - FEB
2012 MED BOARDS; TOPNOTCH MD) MIDTERM 1 EXAM - FEB 2013 531 A previously
healthy 16-year-old high school wrestler comes to the physician because of a rash on his
forearms and the back of his legs for 1 week. He is allergic to pollen and dust. Examination
shows patches of erythema with mild lichenification over the antecubital and popliteal fossae.
There are clusters of painful umbilicated vesicles at sites of active skin inflammation. Which of
the following is the most likely diagnosis? A) Eczema herpeticum B) Herpes zoster C) Keratosis
pilaris D) Lichen planus MIGUEL RAFAEL RAMOS, MD (TOP 3 - FEB 2012 MED BOARDS;
TOPNOTCH MD) MIDTERM 1 EXAM - FEB 2013 532 A 20-year-old man is brought to the
emergency department on a summer day 20 minutes after developing headache, nausea, and
unsteady gait while running the last 2 miles of a marathon. On arrival, he is confused and
disoriented. His temperature is 40 C (104 F), blood pressure is 100/60 mm Hg, and pulse is
155/min. His skin is warm and dry. Neurologic examination shows no focal findings. Which of
the following is the most likely mechanism of this patient's condition? A) Depletion of total body
potassium B) Depletion of total body sodium C) Release of creatine kinase from muscle cells D)
Inadequate dissipation of body heat MIGUEL RAFAEL RAMOS, MD (TOP 3 - FEB 2012 MED
BOARDS; TOPNOTCH MD) MIDTERM 1 EXAM - FEB 2013 533 A 72-year-old man with
hypertension has had increasingly severe back pain over the past 2 months. He had a myocardial
infarction 4 years ago. He has marked tenderness over T11, T12, L1, and L2. An x-ray film of the
lumbosacral spine shows osteoblastic lesions in these vertebrae. Which of the following is the
most likely diagnosis? A) Abdominal aneurysm B) Fibrosarcoma C) Metastatic prostate
carcinoma D) Multiple myeloma Osteoblastic lesions >> prostate mets MIGUEL RAFAEL
RAMOS, MD (TOP 3 - FEB 2012 MED BOARDS; TOPNOTCH MD) MIDTERM 1 EXAM -
FEB 2013 TOPNOTCH MEDICAL BOARD PREP INTERNAL MEDICINE SUPEREXAM
For inquiries visit www.topnotchboardprep.com.ph or email us at
[email protected] TOPNOTCH MEDICAL BOARD PREP INTERNAL
MEDICINE SUPEREXAM Page 71 of 95 For inquiries visit www.topnotchboardprep.com.ph or
email us at [email protected] Item # QUESTION EXPLANATION
AUTHOR TOPNOTCH EXAM 534 A 50-year-old woman has had progressive dyspnea over the
past 2 weeks and constant, sharp chest pain for 4 days. The pain is localized to the center of the
chest and is worse while supine. She underwent a right, modified radical mastectomy and
adjuvant chemotherapy for breast cancer 3 years ago. She has a history of hypothyroidism
treated with thyroid replacement therapy. She has smoked one pack of cigarettes daily for 30
years and drinks two ounces of alcohol daily. She is dyspneic and diaphoretic. Her temperature is
37.2 C (99 F), blood pressure is 90/70 mm Hg with a pulsus paradoxus of 20 mm Hg, pulse is
110/min, and respirations are 28/min. Examination shows jugular venous distention to the angle
of the mandible. The liver span is 14 cm with 4 cm of shifting abdominal dullness. Arterial blood
gas analysis on room air shows a pH of 7.50, PCO2 of 30 mm Hg, and PO2 of 70 mm Hg. An x-
ray film of the chest shows an enlarged cardiac silhouette with a globular configuration. An ECG
shows sinus tachycardia with nonspecific ST-segment changes diffusely. Which of the following
is the most appropriate next step in management? A ) Echocardiography B ) CT scan of the
abdomen C ) Ventilation-perfusion lung scans D ) Bronchoscopy Echo to diagnose cardiac
tamponade MIGUEL RAFAEL RAMOS, MD (TOP 3 - FEB 2012 MED BOARDS;
TOPNOTCH MD) MIDTERM 1 EXAM - FEB 2013 535 A 52-year-old woman with alcoholism
comes to the physician after a serum cholesterol level of 290 mg/dL was found on a routine
screening. She drinks a pint of vodka daily. She takes captopril for hypertension and glyburide
for type 2 diabetes mellitus. She also has intermittent episodes of gout. Fasting serum studies
show: Total cholesterol 252 mg/dL HDL-cholesterol 80 mg/dL Triglycerides 300 mg/dL Glucose
118 mg/dL Thyroid-stimulating hormone 4.5 μU/mL Which of the following is the most
appropriate next step in management? A) Alcohol cessation B) Thyroid replacement therapy C)
Switch from captopril to calcium-channel blocking agent therapy D) Gemfibrozil therapy
MIGUEL RAFAEL RAMOS, MD (TOP 3 - FEB 2012 MED BOARDS; TOPNOTCH MD)
MIDTERM 1 EXAM - FEB 2013 536 A 67-year-old man is brought to the emergency
department 4 hours after the onset of severe midlumbar back pain. He is anxious, pale, and
diaphoretic. His temperature is 37.1 C (98.8 F), blood pressure is 105/65 mm Hg, and pulse is
120/min. Examination shows no other abnormalities. X-ray films of the lumbar spine show
degenerative disc disease with calcifications anterior to the vertebral bodies. Which of the
following is the most likely diagnosis? A) Aortoiliac occlusion B) Herniated nucleus pulposus C)
Lumbar discitis D) Ruptured aortic aneurysm hints >> calcifications anterior to the vertebral
bodies+ BP 105/65 mm Hg MIGUEL RAFAEL RAMOS, MD (TOP 3 - FEB 2012 MED
BOARDS; TOPNOTCH MD) MIDTERM 1 EXAM - FEB 2013 537 A 24-year-old woman
comes to the physician because of constant, severe pain in her neck, shoulders, and back for 3
months. She has been unable to enjoy her usual activities because of the pain. Use of over-the-
counter ibuprofen and aspirin has not relieved her symptoms. She has a history of irritable bowel
syndrome. Examination shows multiple tender spots over the neck, shoulders, and lumbar spine.
Range of motion of all joints is full. There is no evidence of synovitis. Fluorescent serum
antinuclear antibody and rheumatoid factor assays are negative. Which of the following is the
most likely diagnosis? A) Ankylosing spondylitis B) Fibromyalgia C) Polymyalgia rheumatica
D) Polymyositis hint >> multiple tender spots over the neck, shoulders, and lumbar spine
MIGUEL RAFAEL RAMOS, MD (TOP 3 - FEB 2012 MED BOARDS; TOPNOTCH MD)
MIDTERM 1 EXAM - FEB 2013 TOPNOTCH MEDICAL BOARD PREP INTERNAL
MEDICINE SUPEREXAM For inquiries visit www.topnotchboardprep.com.ph or email us at
[email protected] TOPNOTCH MEDICAL BOARD PREP INTERNAL
MEDICINE SUPEREXAM Page 72 of 95 For inquiries visit www.topnotchboardprep.com.ph or
email us at [email protected] Item # QUESTION EXPLANATION
AUTHOR TOPNOTCH EXAM 538 A 72-year-old man comes to the physician because of a 6-
month history of mild to moderate shortness of breath when climbing stairs. He had a myocardial
infarction 2 years ago and has had an ejection fraction of 35% since then. His only medication is
a β-adrenergic blocking agent. The lungs are clear to auscultation. Cardiac examination shows an
S4 gallop. There is no peripheral edema. Laboratory studies are within normal limits. An ECG
shows no acute changes. Which of the following is the most appropriate pharmacotherapy? A) α-
Adrenergic blocking agent B) Angiotensin-converting enzyme (ACE) inhibitor C) Angiotensin2-
receptor blocking agent D) Nitrates ACEI show mortality benefit, ARBs do no MIGUEL
RAFAEL RAMOS, MD (TOP 3 - FEB 2012 MED BOARDS; TOPNOTCH MD) MIDTERM 1
EXAM - FEB 2013 539 A 57-year-old man is brought to the emergency department 30 minutes
after he was found on the floor of his house. He has renal failure but has missed his last two
dialysis treatments. His renal failure was caused by inadvertent ingestion of ethylene glycol.
Medications include amlodipine and doxazosin. On arrival, his temperature is 37.5 C (99.5 F),
blood pressure is 150/100 mm Hg, pulse is 95/min and regular, and respirations are 24/min. His
breathing is rapid and deep. Crackles are heard in the lung bases. Laboratory studies show: Na+
135 mEq/L Cl– 102 mEq/L K+ 7.1 mEq/L HCO3– 12 mEq/L pH 7.22 PCO2 31 mm Hg PO2 61
mm Hg An ECG shows peaked T-waves. It will be at least 45 minutes before dialysis can be
started. Which of the following is the most appropriate next step in management? A) Intravenous
sodium bicarbonate B) Intravenous calcium gluconate C) Intravenous glucose and insulin D)
Intravenous 0.9% saline ECG changes with hyperkalemia >> immediate IV calcium gluconate
MIGUEL RAFAEL RAMOS, MD (TOP 3 - FEB 2012 MED BOARDS; TOPNOTCH MD)
MIDTERM 1 EXAM - FEB 2013 TOPNOTCH MEDICAL BOARD PREP INTERNAL
MEDICINE SUPEREXAM For inquiries visit www.topnotchboardprep.com.ph or email us at
[email protected] TOPNOTCH MEDICAL BOARD PREP INTERNAL
MEDICINE SUPEREXAM Page 73 of 95 For inquiries visit www.topnotchboardprep.com.ph or
email us at [email protected] Item # QUESTION EXPLANATION
AUTHOR TOPNOTCH EXAM 540 A 26-year-old woman is brought to the emergency
department because of marked confusion for 2 hours; she also has had a flu-like illness for 3
days. Over the past 6 weeks, she has had increased fatigue, weakness, and nausea. She recently
started thyroid hormone replacement therapy for autoimmune thyroiditis; 1 week ago, her serum
thyroidstimulating hormone level was 3 μU/mL. Her temperature is 38 C (100.4 F), blood
pressure is 80/40 mm Hg, and pulse is 140/min. She appears confused and lethargic.
Examination shows cool, mottled skin. There is generalized hyperpigmentation, especially
involving the palmar creases. The lungs are clear to auscultation. Abdominal examination shows
diffuse mild tenderness and no rebound. Laboratory studies show: Hemoglobin 10 g/dL
Leukocyte count 9000/mm3 Segmented neutrophils 55% Eosinophils 20% Lymphocytes 25%
Serum Na+ 124 mEq/L Cl– 92 mEq/L K+ 6.4 mEq/L HCO3– 16 mEq/L An x-ray film of the
chest and urinalysis show normal findings. An ECG shows sinus tachycardia with peaked T
waves. Which of the following is most likely to confirm the primary cause of this patient's
condition? A) Measurement of serum antithyroglobulin antibody level B) Dexamethasone
suppression test C) Measurement of serum thyroid-stimulating hormone level D) ACTH
stimulation test Addison'a Disease MIGUEL RAFAEL RAMOS, MD (TOP 3 - FEB 2012 MED
BOARDS; TOPNOTCH MD) MIDTERM 1 EXAM - FEB 2013 541 A 45/M comes in with
severe, prolonged substernal chest pain associated with diaphoresis and nausea. ECG reveals ST
segment elevation in the anterior leads. Among other medications, you start him on Metoprolol.
What is the mechanism of action of this drug in the treatment of ischemia in myocardial
infarction? A. Systemic venodilation with reduction in LV enddiastolic volume B. Dilation of
epicardial coronary vessels C. Increased blood flow in collateral vessels D. Reduction of
myocardial O2 demand by inhibiting increases in HR, BP and myocardial contractility E. All of
the above The rest are actions of nitrates. ABDELSIMAR OMAR II, MD (TOP 2 - AUG 2013
MED BOARDS; TOPNOTCH MD - 200 QUESTIONS) AND MARC DENVER TIONGSON,
MD (40 QUESTIONS) FINAL EXAM - FEB 2014 542 Four months after an MI, a 45/M starts
experiencing gradually worsening exertional dyspnea associated with 3-pillow orthopnea and
paroxysmal nocturnal dyspnea. On PE, you note bibasilar rales on auscultation and Grade 2
bipedal edema. Which of the following are cornerstones of treatment? A. Fluid restriction B.
Caloric supplementation C. ACE inhibitors and beta blockers D. Diuretics and digoxin E. All of
the above In heart failure, fluid restriction is generally unnecessary unless with hyponatremia and
volume overload. Caloric supplementation is warranted for those with cardiac cachexia.
Diuretics are only used to restore normovolemic status; while digoxin is used only for
symptomatic LV dysfunction with concomitant AF as add on to standard therapy.
ABDELSIMAR OMAR II, MD (TOP 2 - AUG 2013 MED BOARDS; TOPNOTCH MD - 200
QUESTIONS) AND MARC DENVER TIONGSON, MD (40 QUESTIONS) FINAL EXAM -
FEB 2014 543 According to JNC7, which of the following lifestyle modification
recommendations leads to the greatest reduction in average systolic blood pressure? A. DASH
eating plan B. Dietary sodium restriction C. Moderation of alcohol consumption D. Weight
reduction (10 kg) E. Aerobic physical activity Weight reduction = 5 - 20 mmHg/10 kg
ABDELSIMAR OMAR II, MD (TOP 2 - AUG 2013 MED BOARDS; TOPNOTCH MD - 200
QUESTIONS) AND MARC DENVER TIONGSON, MD (40 QUESTIONS) FINAL EXAM -
FEB 2014 TOPNOTCH MEDICAL BOARD PREP INTERNAL MEDICINE SUPEREXAM
For inquiries visit www.topnotchboardprep.com.ph or email us at
[email protected] TOPNOTCH MEDICAL BOARD PREP INTERNAL
MEDICINE SUPEREXAM Page 74 of 95 For inquiries visit www.topnotchboardprep.com.ph or
email us at [email protected] Item # QUESTION EXPLANATION
AUTHOR TOPNOTCH EXAM 544 You are performing the chest PE on a patient who came in
for dyspnea. The left lung fields were dull on percussion while vocal fremitus was increased.
This is consistent with: A. Emphysema B. Pneumothorax C. Pleural effusion D. Asthma E.
Consolidation Emphysema and pneumothorax: hyperresonant, decreased. Pleural effusion: dull,
decreased. Asthma: hyper-resonant, normal ABDELSIMAR OMAR II, MD (TOP 2 - AUG 2013
MED BOARDS; TOPNOTCH MD - 200 QUESTIONS) AND MARC DENVER TIONGSON,
MD (40 QUESTIONS) FINAL EXAM - FEB 2014 545 A 55/M heavy smoker comes in for
cough productive of sputum and progressive and persistent exertional dyspnea. On PE, you note
hyperresonant lung fields and poor diagphragmatic excursion on percussion. Which of the
following interventions have been demonstrated to influence the natural history of his condition?
A. Smoking cessation B. Treatment with inhaled corticosteroids C. Treatment with long acting
beta agonists D. Treatment with anticholinergics E. All of the above In COPD, only 3
interventions influence the natural history: smoking cessation, lung volume reduction surgery
and oxygen therapy in the chronically hypoxemic patients. ABDELSIMAR OMAR II, MD (TOP
2 - AUG 2013 MED BOARDS; TOPNOTCH MD - 200 QUESTIONS) AND MARC DENVER
TIONGSON, MD (40 QUESTIONS) FINAL EXAM - FEB 2014 546 High levels of adenosine
deaminase (>40 IU/L) in pleural fluid is virtually diagnostic of: A. Parapneumonic effusion B.
Malignant pleural effusion C. Effusion secondary to congestive heart failure D. Tuberculous
pleural effusion E. Cirrhosis ABDELSIMAR OMAR II, MD (TOP 2 - AUG 2013 MED
BOARDS; TOPNOTCH MD - 200 QUESTIONS) AND MARC DENVER TIONGSON, MD
(40 QUESTIONS) FINAL EXAM - FEB 2014 547 A 35/F, diagnosed case of SLE, comes in for
a follow up. Routine labs done revealed the following results: ABG: pH 7.35, pCO2 34, pO2 of
98; Na 132; K 3.2; Cl 108; and HCO3 12. The patient's metabolic disorder is likely due to: A.
Uremia B. Diarrhea C. Diabetic ketoacidosis D. Lactic acidosis E. Renal tubular acidosis Lab
data consistent with NORMAL ANION GAP METABOLIC ACIDOSIS. Only E causes
NAGMA among choices. Lupus is a known cause of type 1 RTA. ABDELSIMAR OMAR II,
MD (TOP 2 - AUG 2013 MED BOARDS; TOPNOTCH MD - 200 QUESTIONS) AND MARC
DENVER TIONGSON, MD (40 QUESTIONS) FINAL EXAM - FEB 2014 548 The presence of
ECG changes in hyperkalemia should be considered as an emergency and thus should be
managed immediately. Which of the following ECG abnormalities is characteristic of
hyperkalemia? A. Prolonged PR interval B. Presence of U waves C. ST segment depression D.
Loss of P waves E. Shortened QT interval Classic ECG findings: peaked T waves (5.5 - 6.5
mM); loss of P waves (6.5 - 7.5); widened QRS (7 - 8 mM); and sinusoidal pattern (>8 mM).
ABDELSIMAR OMAR II, MD (TOP 2 - AUG 2013 MED BOARDS; TOPNOTCH MD - 200
QUESTIONS) AND MARC DENVER TIONGSON, MD (40 QUESTIONS) FINAL EXAM -
FEB 2014 549 A 25/M, recently diagnosed case of type 1 DM, comes in for routine follow up
after he noted passing foamy urine. You order a urinalysis which reveals 3+ proteinuria. You
refer the patient to an Ophthalmologist who found no evidence of retinopathy. The most likely
cause of proteinuria in the patient is: A. Diabetic nephropathy B. Hypertensive kidney disease C.
Minimal change disease D. Membranous glomerulonephritis E. Focal segmental
glomerulosclerosis Usually, DM nephropathy develops after 10 years duration; occurring in the
presence of other microvascular complications, e.g. retinopathy. PEARL: Presence of
nephropathy in diabetic in the absence of eye disease should warrant investigation for other
causes of nephropathy. The most common cause of nephrotic syndrome in adults is MGN; the
most common cause in kids is MCD. ABDELSIMAR OMAR II, MD (TOP 2 - AUG 2013 MED
BOARDS; TOPNOTCH MD - 200 QUESTIONS) AND MARC DENVER TIONGSON, MD
(40 QUESTIONS) FINAL EXAM - FEB 2014 550 You are assessing glycemic control in a
diabetic patient who does self monitoring of blood glucose. You note that the patient's post-
prandial glucose levels are elevated. Which of the following hypoglycemic agents target post-
prandial glucose in particular? A. Metformin B. Glibenclamide C. Pioglitazone D. Miglitol E. All
of the above Agents which target PPG in particular include meglitinides, incretin-related drugs
(sitagliptin, saxagliptin) and a-glucosidase inhibitors (acarbose and miglitol.) ABDELSIMAR
OMAR II, MD (TOP 2 - AUG 2013 MED BOARDS; TOPNOTCH MD - 200 QUESTIONS)
AND MARC DENVER TIONGSON, MD (40 QUESTIONS) FINAL EXAM - FEB 2014 551
What is the MOST COMMON sign of hyperthyroidism? A. Tremor B. Goiter C. Warm, moist
skin D. Tachycardia E. Lid lag SIMILAR TO PREVIOUS BOARD EXAM
CONCEPT/PRINCIPLE. The most common SYMPTOM is hyperactivity, irritability and
dysphoria. ABDELSIMAR OMAR II, MD (TOP 2 - AUG 2013 MED BOARDS; TOPNOTCH
MD - 200 QUESTIONS) AND MARC DENVER FINAL EXAM - FEB 2014 TOPNOTCH
MEDICAL BOARD PREP INTERNAL MEDICINE SUPEREXAM For inquiries visit
www.topnotchboardprep.com.ph or email us at [email protected]
TOPNOTCH MEDICAL BOARD PREP INTERNAL MEDICINE SUPEREXAM Page 75 of 95
For inquiries visit www.topnotchboardprep.com.ph or email us at
[email protected] Item # QUESTION EXPLANATION AUTHOR
TOPNOTCH EXAM TIONGSON, MD (40 QUESTIONS) 552 A 28/F with Cushingoid features
complained of blurring of vision. Formal visual field testing revealed bitemporal hemianopsia.
You suspect a functioning pituitary adenoma. Which of the following laboratory findings is NOT
consistent with your primary impression? A. Elevated serum cortisol B. Elevated urine free
cortisol C. Cortisol level not suppressed with low-dose dexamethasone D. Cortisol level not
suppressed with high-dose dexamethasone E. None of the above A high-dose dexamethasone
suppression test can differentiate pituitary Cushing's syndrome (cortisol is suppressed) from
adrenal/ectopic Cushing's syndrome (cortisol is NOT suppressed). ABDELSIMAR OMAR II,
MD (TOP 2 - AUG 2013 MED BOARDS; TOPNOTCH MD - 200 QUESTIONS) AND MARC
DENVER TIONGSON, MD (40 QUESTIONS) FINAL EXAM - FEB 2014 553 After a night of
binge-drinking, a 53/M comes in to the ER complaining of sudden onset severe mid epigastric
abdominal pain radiating to the back; associated with nausea, vomiting and anorexia. Vital signs
are as follows: BP 90/50, HR 114, RR 24, T 39C. Cornerstones for the management of this
condition include the following except: A. Analgesivs B. IV hydration C. No oral alimentation
(NPO) D. Prophylactic antibiotics E. None of the above Classic pancreatitis. Prophylactic
antibiotics have NO role in either interstitial or necrotizing pancreatitis ABDELSIMAR OMAR
II, MD (TOP 2 - AUG 2013 MED BOARDS; TOPNOTCH MD - 200 QUESTIONS) AND
MARC DENVER TIONGSON, MD (40 QUESTIONS) FINAL EXAM - FEB 2014 554 A 28/M
who presents with fever and jaundice has the following hepatitis profile: IgM Anti-HAV (+);
HBsAg (+); IgM Anti-HBc (-); and Anti-HCV (-). He has: A. Acute hepatitis A B. Acute hepatitis
B C. Acute hepatitis A and B D. Acute hepatitis A superimposed on chronic hepatitis B E. Acute
hepatitis C ABDELSIMAR OMAR II, MD (TOP 2 - AUG 2013 MED BOARDS; TOPNOTCH
MD - 200 QUESTIONS) AND MARC DENVER TIONGSON, MD (40 QUESTIONS) FINAL
EXAM - FEB 2014 555 The following are recommendations of the Surviving Sepsis Guidelines
in the management of sepsis, EXCEPT: A. Start IV fluids to maintain CVP > 12 mmHg in
mechanically ventilated patients. B. Maintain MAP above 65 mmHg. C. Epinephrine and
dopamine are the initial vasopressors of choice. D. Start with broad-spectrum IV antibiotics with
adequate penetration. E. Hydrocortisone may be indicated for adult septic shock when
hypotension responds poorly to sufficient fluid resuscitation and vasopressors. Start with
norepinephrine and dopamine. ABDELSIMAR OMAR II, MD (TOP 2 - AUG 2013 MED
BOARDS; TOPNOTCH MD - 200 QUESTIONS) AND MARC DENVER TIONGSON, MD
(40 QUESTIONS) FINAL EXAM - FEB 2014 556 A 12/M was attacked by a stray dog and
sustained minor abrasions without bleeding. The patient has had no rabies vaccination. Which of
the following is MOST appropriate? A. Wash exposed skin with immediately with soap and
water. No vaccine or RIG is needed. B. Administer rabies immunoglobulin immediately. C. Start
rabies vaccination and rabies immunoglobulin. D. Start rabies vaccination without rabies
immunoglobulin. E. Start rabies vaccination, rabies immunoglobulin, and oral antibiotics. Patient
had a Category II exposure (minor abrasions without bleeding). Start vaccine only.
ABDELSIMAR OMAR II, MD (TOP 2 - AUG 2013 MED BOARDS; TOPNOTCH MD - 200
QUESTIONS) AND MARC DENVER TIONGSON, MD (40 QUESTIONS) FINAL EXAM -
FEB 2014 557 A 40/M complains of excruciating pain and swelling in his left big toe that
developed suddenly. He is obese and is a heavy beer drinker. On PE, you note marked swelling,
redness, and warmth of the left big toe. You may give all of the following for acute attacks
except: A. NSAIDs B. Glucocorticoids C. Colchicine D. Allopurinol E. Ice compress Do NOT
start hypouricemic therapy during acute attacks of gout. ABDELSIMAR OMAR II, MD (TOP 2
- AUG 2013 MED BOARDS; TOPNOTCH MD - 200 QUESTIONS) AND MARC DENVER
TIONGSON, MD (40 QUESTIONS) FINAL EXAM - FEB 2014 TOPNOTCH MEDICAL
BOARD PREP INTERNAL MEDICINE SUPEREXAM For inquiries visit
www.topnotchboardprep.com.ph or email us at [email protected]
TOPNOTCH MEDICAL BOARD PREP INTERNAL MEDICINE SUPEREXAM Page 76 of 95
For inquiries visit www.topnotchboardprep.com.ph or email us at
[email protected] Item # QUESTION EXPLANATION AUTHOR
TOPNOTCH EXAM 558 Which of the following autoantibodies is SLE-specific and correlates
with disease activity, nephritis and vasculitis? A. ANA B. Anti-dsDNA C. Anti-Sm D. Anti-Ro E.
Antihistone C is highly specific but has NO definite clinical correlations. A is the best screening
test. D is nonspecific for SLE and associated with sicca syndrome, and neonatal lupus. E is
associated more with drug-induced lupus. ABDELSIMAR OMAR II, MD (TOP 2 - AUG 2013
MED BOARDS; TOPNOTCH MD - 200 QUESTIONS) AND MARC DENVER TIONGSON,
MD (40 QUESTIONS) FINAL EXAM - FEB 2014 559 Anemia with a reticulocyte index > 2.5
is consistent with: A. Iron deficiency anemia B. Thalassemia C. Hereditary spherocytosis D.
Anemia of chronic disease E. Bone marrow infiltration RI > 2.5 suggests hemolysis or
hemorrhage. ABDELSIMAR OMAR II, MD (TOP 2 - AUG 2013 MED BOARDS; TOPNOTCH
MD - 200 QUESTIONS) AND MARC DENVER TIONGSON, MD (40 QUESTIONS) FINAL
EXAM - FEB 2014 560 Platelet count and bleeding time within normal limits, associated with
prolonged PT and PTT, is classically seen in: A. Immune thrombocytopenic pupura B. Bernard-
Soulier syndrome C. von Willebrand disease D. Hemophilia E. Vitamin K deficiency
ABDELSIMAR OMAR II, MD (TOP 2 - AUG 2013 MED BOARDS; TOPNOTCH MD - 200
QUESTIONS) AND MARC DENVER TIONGSON, MD (40 QUESTIONS) FINAL EXAM -
FEB 2014 561 Obesity is the driving force behind the metabolic syndrome, thus, the primary
approach to this disorder is/are A. Treatment of dyslipidemia B. ACE-inhibitors C. Smoking
cessation D. Weight reduction Obesity is the driving force behind the metabolic syndrome. Thus,
weight reduction is the primary approach to the disorder. With weight reduction, the
improvement in insulin sensitivity is often accompanied by favorable modifications in many
components of the metabolic syndrome. In general, recommendations for weight loss include a
combination of caloric restriction, increased physical activity, and behavior modification
BLAKE WARREN ANG, MD (TOP 1 - AUG 2013 MED BOARDS; TOPNOTCH MD)
MIDTERM 2 EXAM - FEB 2014 562 The drug of choice for lowering fasting triglycerides A.
Gemfibrozil B. Atorvastatin C. Nicotinic Acid D. Ezetimibe A fibrate (gemfibrozil or
fenofibrate) is the drug of choice to lower fasting triglycerides and typically achieve a 35–50%
reduction BLAKE WARREN ANG, MD (TOP 1 - AUG 2013 MED BOARDS; TOPNOTCH
MD) MIDTERM 2 EXAM - FEB 2014 563 Indications for coronary arteriography EXCEPT:
(HPIM 18, p2005) A. Patients with known or possible angina pectoris who have survived cardiac
arrest B. Patients with chronic stable angina pectoris who are severely symptomatic despite
medical therapy and who are being considered for revascularization C. Patients with troublesome
symptoms that present diagnostic difficulties in whom there is a need to confirm or rule out IHD
D. Patients with no evidence of ischemia on noninvasive testing and no clinical or laboratory
evidence of ventricular dysfunction Answer: Coronary arteriography is indicated in: (1) patients
with chronic stable angina pectoris who are severely symptomatic despite medical therapy and
are being considered for revascularization, i.e., a percutaneous coronary intervention (PCI) or
coronary artery bypass grafting (CABG), (2) patients with troublesome symptoms that present
diagnostic difficulties in whom there is a need to confirm or rule out the diagnosis of IHD, (3)
patients with known or possible angina pectoris who have survived cardiac arrest, (4) patients
with angina or evidence of ischemia on noninvasive testing with clinical or laboratory evidence
of ventricular dysfunction, and (5) patients judged to be at high risk of sustaining coronary
events based on signs of severe ischemia on noninvasive testing, regardless of the presence or
severity of symptoms BLAKE WARREN ANG, MD (TOP 1 - AUG 2013 MED BOARDS;
TOPNOTCH MD) MIDTERM 2 EXAM - FEB 2014 564 At least how many hours each day
must a patient be kept nitrate-free to minimize tolerance and restore any useful response (HPIM
18, p2010) A. 4 hours B. 6 hours C. 8 hours D.12 hours Different preparations and/or
administration during the daytime should be tried only to prevent discomfort while avoiding side
effects such as headache and dizziness. Individual dose titration is important to prevent side
effects. To minimize the effects of tolerance, the minimum effective dose should be used and a
minimum of 8 h each day kept free of the drug to restore any useful response(s). BLAKE
WARREN ANG, MD (TOP 1 - AUG 2013 MED BOARDS; TOPNOTCH MD) MIDTERM 2
EXAM - FEB 2014 565 Mechanism of action of thienopyridines (HPIM 18, p2011, 2017) A.
Inhibition of thromboxane synthesis B. Inhibition of the IIB/IIIA receptor C. Inhibition of
clotting factor Xa D. Inhibition of P2Y12 ADP receptor Inhibition of P2Y12 ADP Receptor
BLAKE WARREN ANG, MD (TOP 1 - AUG 2013 MED BOARDS; TOPNOTCH MD)
MIDTERM 2 EXAM - FEB 2014 TOPNOTCH MEDICAL BOARD PREP INTERNAL
MEDICINE SUPEREXAM For inquiries visit www.topnotchboardprep.com.ph or email us at
[email protected] TOPNOTCH MEDICAL BOARD PREP INTERNAL
MEDICINE SUPEREXAM Page 77 of 95 For inquiries visit www.topnotchboardprep.com.ph or
email us at [email protected] Item # QUESTION EXPLANATION
AUTHOR TOPNOTCH EXAM 566 NOT an indication for discontinuation of stress testing: A.
Chest discomfort B. Ventricular tachyarrhythmia C. Fall in systolic blood pressure 0.2 mV (2
mm), a fall in systolic blood pressure >10 mmHg, or the development of a ventricular
tachyarrhythmia. BLAKE WARREN ANG, MD (TOP 1 - AUG 2013 MED BOARDS;
TOPNOTCH MD) MIDTERM 2 EXAM - FEB 2014 567 NOT a contraindication to exercise
stress testing: (HPIM 18, p2004) A. Severe pulmonary hypertension B. Severe aortic
regurgitation C. Uncontrolled heart failure D. Active infective endocarditis Contraindications to
exercise stress testing include rest angina within 48 h, unstable rhythm, severe aortic stenosis,
acute myocarditis, uncontrolled heart failure, severe pulmonary hypertension, and active
infective endocarditis. BLAKE WARREN ANG, MD (TOP 1 - AUG 2013 MED BOARDS;
TOPNOTCH MD) MIDTERM 2 EXAM - FEB 2014 568 The most common clinical indication
for Percutaneous Coronary Intervention a. Decompensated Heart Failure b. Ventricular
Tachyarrythmias c. Symptom – limiting Angina Pectoris d. Cardiogenic shock The most
common clinical indication for PCI is symptom-limiting angina pectoris, despite medical
therapy, accompanied by evidence of ischemia during a stress test. PCI is more effective than
medical therapy for the relief of angina. PCI improves outcomes in patients with unstable angina
or when used early in the course of myocardial infarction with and without cardiogenic shock.
BLAKE WARREN ANG, MD (TOP 1 - AUG 2013 MED BOARDS; TOPNOTCH MD)
MIDTERM 2 EXAM - FEB 2014 569 After verification of precocious pubertal development,
which of the following lab tests should first be measured? A. Serum LH and FSH B. Serum
testosterone C. Serum DHEA S D. Serum B-HCG Answer: To determine if it is gonadotropin
dependent or independent • Gonadotropin Dependent Gonadotropins increased in relation to
chronologic age CNS lesions, idiopathic central precocity • Gonadotropin Indenpendent High
testosterone/suppressed LH CAH, adrenal neoplasms, testicular neoplasms BLAKE WARREN
ANG, MD (TOP 1 - AUG 2013 MED BOARDS; TOPNOTCH MD) MIDTERM 2 EXAM -
FEB 2014 570 Most important step in the evaluation of male infertility A. Serum testosterone B.
HCG stimulation test C. Semen Analysis D. Testicular biopsy Answer: Most important step in
the evaluation of male infertility • Normal ejaculate Volume 2-6 mL Sperm count of
>20million/mL Motility >50% >15% normal morphology BLAKE WARREN ANG, MD (TOP 1
- AUG 2013 MED BOARDS; TOPNOTCH MD) MIDTERM 2 EXAM - FEB 2014 571 What
malignancy is associated with a mutated RET proto-oncogene? A. Follicular carcinoma B.
Medullary carcinoma C. Parathyroid carcinoma D. Papillary carcinoma BLAKE WARREN
ANG, MD (TOP 1 - AUG 2013 MED BOARDS; TOPNOTCH MD) MIDTERM 2 EXAM -
FEB 2014 572 Agents proven to reduce the risk of fractures in patients being treated with
glucocorticoids a. Bisphosphonates b. Selective estrogen response modulators (SERMs) c.
Calcium supplements d. Hormone replacement therapy Answer: Only bisphosphonates have
been demonstrated in large clinical trials to reduce the risk of fractures in patients being treated
with glucocorticoids. Risedronate prevents bone loss and reduces vertebral fracture risk by
~70%. BLAKE WARREN ANG, MD (TOP 1 - AUG 2013 MED BOARDS; TOPNOTCH MD)
MIDTERM 2 EXAM - FEB 2014 573 Substance that is produced by macrophages in chronic
granulomatous diseases (tuberculosis, sarcoidosis) that causes hypercalcemia a. PTHrP b.
1,25(OH)2D c. PTH d. 25(OH)D Answer: In patients with sarcoidosis and other granulomatous
diseases, such as tuberculosis and fungal infections, excess 1,25(OH)2D is synthesized in
macrophages or other cells in the granulomas. BLAKE WARREN ANG, MD (TOP 1 - AUG
2013 MED BOARDS; TOPNOTCH MD) MIDTERM 2 EXAM - FEB 2014 574 First line anti-
epileptic drug for petit mal and grand mal seizures (HPIM p2507 table 363-8) A. Phenytoin B.
Phenobarbital C. Levetiracetam D. Valproic acid ANSWER: first line drugs are as follows •
Primary Generalized Tonic-Clonic Valproic Acid Lamotrigine Topiramate • Absence Seizures
Valproic Acid Ethosuximide BLAKE WARREN ANG, MD (TOP 1 - AUG 2013 MED
BOARDS; TOPNOTCH MD) MIDTERM 2 EXAM - FEB 2014 TOPNOTCH MEDICAL
BOARD PREP INTERNAL MEDICINE SUPEREXAM For inquiries visit
www.topnotchboardprep.com.ph or email us at [email protected]
TOPNOTCH MEDICAL BOARD PREP INTERNAL MEDICINE SUPEREXAM Page 78 of 95
For inquiries visit www.topnotchboardprep.com.ph or email us at
[email protected] Item # QUESTION EXPLANATION AUTHOR
TOPNOTCH EXAM 575 Acute nephrotic syndrome associated with hepatitis B and C
infections: A. Mesangioproliferative GN B. Membranoproliferative GN C. IgA Nephropathy D.
MCD MGN and MPGN are causes of NEPHROTIC SYNDROME. IgA Neph causes SSx of
both. Mesangioproliferative GN is type II lupus nephritis. MGN and MPGN are both associated
with hep B and C. BLAKE WARREN ANG, MD (TOP 1 - AUG 2013 MED BOARDS;
TOPNOTCH MD) MIDTERM 2 EXAM - FEB 2014 576 The most common cause of nephrotic
syndrome in the elderly A. Focal segmental glomerulosclerosis B. Membranous
glomerulonephritis C. Diabetic nephropathy D. Minimal change disease BLAKE WARREN
ANG, MD (TOP 1 - AUG 2013 MED BOARDS; TOPNOTCH MD) MIDTERM 2 EXAM -
FEB 2014 577 Kimmelstiel-Wilson nodules are seen in: A. Fabry’s disease B. Focal segmental
glmerulosclerosis C. Diabetic Nephropathy D. Membranous Glomerulonephritis Answer: Some
patients (with Diabetic nephropathy) also develop eosinophilic, PAS+ nodules called nodular
glomerulosclerosis or Kimmelstiel-Wilson nodules. BLAKE WARREN ANG, MD (TOP 1 -
AUG 2013 MED BOARDS; TOPNOTCH MD) MIDTERM 2 EXAM - FEB 2014 578
Hematuria, thinning and splitting of the GBMs, mild proteinuria, chronic glomerulosclerosis
leading to renal failure and Sensorineural deafness is seen in: A. Anti-GBM disease B. Alport’s
Syndrome C. Thin Basement Membrane disease D. Nail-Patella Syndrome BLAKE WARREN
ANG, MD (TOP 1 - AUG 2013 MED BOARDS; TOPNOTCH MD) MIDTERM 2 EXAM -
FEB 2014 579 Sclerotherapy is a treatment option for which stage of hemorrhoidal disease? a.
Stage I – enlargement with bleeding b. Stage II – protrusion with spontaneous reduction c. Stage
III – protrusion requiring manual reduction d. Stage IV – irreducible protrusion III BLAKE
WARREN ANG, MD (TOP 1 - AUG 2013 MED BOARDS; TOPNOTCH MD) MIDTERM 2
EXAM - FEB 2014 580 After a person is infected with HBV, the first virologic marker
detectable in serum within 1-12 weeks is (H-18, C-304, P-2540): a) HBeAg b) HBsAg c) HBcAg
d) Anti-HBc IgM BLAKE WARREN ANG, MD (TOP 1 - AUG 2013 MED BOARDS;
TOPNOTCH MD) MIDTERM 2 EXAM - FEB 2014 581 Finkelstein's test is used to diagnose
what condition? A. De Quervain tenosynovitis B. Ulnar nerve palsy C. Compartment syndrome
D. Carpal tunnel syndrome E. Osteoarthritis of the hands Prayer test and Tinel test is for carpal
tunnel TIMOTHY TANG LEE SAY, MD (TOP 4 - AUG 2013 MED BOARDS; TOPNOTCH
MD) MIDTERM 1 EXAM - FEB 2014 582 A patient with meningitis suddenly went to coma.
The cause is? A. Sepsis B. Increased ICP C. Pulmonary embolism D. Hyperthermia E.
Subarachnoid hemorrhage One of the treatment for ICP increase is inducing a state of coma. The
most likely cause for a patient with meningitis is sepsis or bacterial seeding resulting in a
depressed brain. TIMOTHY TANG LEE SAY, MD (TOP 4 - AUG 2013 MED BOARDS;
TOPNOTCH MD) MIDTERM 1 EXAM - FEB 2014 583 Risk factors associated with the
development of osteoporosis include all the following EXCEPT? A. Excessive alcohol intake B.
Obesity C. Genetics D. Poor calcium intake during adolescence E. Postmenopausal state Obesity
prevents osteoporosis. (Pre test questions may appear in IM, no. 270, Pretest 7th ed) TIMOTHY
TANG LEE SAY, MD (TOP 4 - AUG 2013 MED BOARDS; TOPNOTCH MD) MIDTERM 1
EXAM - FEB 2014 584 Which antibodies are associated with neuropsychiatric lupus? A. Anti-
Sm B. Anti-Ro and Anti-La C. Anti-nRNP D. Anti-ribosomal E. Anti-centromere Anti-Sm -
specific for SLE Ro and La - neonatal lupus nRNP - mixed connective tissue disease anti-
centromere - scleroderma TIMOTHY TANG LEE SAY, MD (TOP 4 - AUG 2013 MED
BOARDS; TOPNOTCH MD) MIDTERM 1 EXAM - FEB 2014 585 What is the standard
immunization schedule for primary Hepatitis B vaccination among adults? A. 0, 1 and 6 months
from initial vaccination B. 0, 1 and 2 months (4 weeks interval) C. 0, 6 and 12 months (6 months
interval) D. 0, 1 and 2 months plus a booster dose after 1 year of the last dose E. 0, 2 months (2
doses 6-8 weeks apart) The schedule for initial vaccination in adults is similar to children. 0, 1
(or 2) and 6 months. TIMOTHY TANG LEE SAY, MD (TOP 4 - AUG 2013 MED BOARDS;
TOPNOTCH MD) MIDTERM 1 EXAM - FEB 2014 TOPNOTCH MEDICAL BOARD PREP
INTERNAL MEDICINE SUPEREXAM For inquiries visit www.topnotchboardprep.com.ph or
email us at [email protected] TOPNOTCH MEDICAL BOARD PREP
INTERNAL MEDICINE SUPEREXAM Page 79 of 95 For inquiries visit
www.topnotchboardprep.com.ph or email us at [email protected] Item #
QUESTION EXPLANATION AUTHOR TOPNOTCH EXAM 586 An 18 year old male
complains of polyuria. What is the most likely diagnosis? A. Diabetes Milletus B. Diabetes
Insipidus C. Primary polydipsia D. Renal tubular necrosis E. Idiopathic polyuria The most
common cause of polyuria in both adults and children is uncontrolled diabetes mellitus, causing
an osmotic diuresis. In the absence of diabetes mellitus, the most common causes are primary
polydipsia (excessive fluid drinking), central diabetes insipidus and nephrogenic diabetes
insipidus. TIMOTHY TANG LEE SAY, MD (TOP 4 - AUG 2013 MED BOARDS; TOPNOTCH
MD) MIDTERM 1 EXAM - FEB 2014 587 Uncontrolled hypertension not amenable to lifestyle
modifications is treated with? A. Calcium channel blocker B. Thiazide diuretic C. ACE Inhibitor
D. β-blocker E. Central acting adrenergic agents The first line for hypertension withour
compelling indications after lifestyle modification is a thiazide diuretic based on JNC 7.
TIMOTHY TANG LEE SAY, MD (TOP 4 - AUG 2013 MED BOARDS; TOPNOTCH MD)
MIDTERM 1 EXAM - FEB 2014 588 In the setting of myocardial infarction, what is the first
cardiac biomarker to be detected? A. Troponin I B. CK-MB C. Total CK D. LDH E. Myoglobin
The first biomarker to be elevated is myoglobin. The most specific for MI is troponin I or T.
TIMOTHY TANG LEE SAY, MD (TOP 4 - AUG 2013 MED BOARDS; TOPNOTCH MD)
MIDTERM 1 EXAM - FEB 2014 589 A systolic murmur is detected upon ausculatation of a
heart failure patient. Prominent findings include an enlarged and pulsatile liver. What is the heart
valve defect? A. Aortic stenosis B. Ventral septal defect C. Tricuspid insufficiecny D. Mitral
regurgitation E. Pulmonary stenosis An enlarged and pulsatile liver is virtually diagnostic of
Tricuspid Insufficiency. TIMOTHY TANG LEE SAY, MD (TOP 4 - AUG 2013 MED BOARDS;
TOPNOTCH MD) MIDTERM 1 EXAM - FEB 2014 590 The goal of BP reduction in a diabetic
patient is? A. 98% of patients during the course of disease; repeated negative tests suggest that
the diagnosis is not SLE, unless other autoantibodies are present. High-titer IgG antibodies to
double-stranded DNA and antibodies to the Sm antigen are both specific for SLE and, therefore,
favor the diagnosis in the presence of compatible clinical manifestations. RACHELLE
MENDOZA, MD (TOP 9 - AUG 2012 MED BOARDS; TOPNOTCH MD) FINAL EXAM -
FEB 2013 605 A 45-year old patient suddenly developed diffuse blisters and eventual skin
detachment after injection of a certain drug. Upon evaluation, the physician documented that the
25% of the total body surface area of the patient is affected. The most likely diagnosis is: A.
Stevens-Johnson Syndrome (SJS) B. Toxic Epidermal Necrolysis (TEN) C. SJS/TEN D. Serum
sickness E. Acute generalized exanthematous pustulosis The term SJS is now used to describe
cases with blisters developing on dusky or purpuric macules in which total body surface area
blistering and eventual detachment is 30% detachment. (Harrison's, 17th ed) RACHELLE
MENDOZA, MD (TOP 9 - AUG 2012 MED BOARDS; TOPNOTCH MD) FINAL EXAM -
FEB 2013 606 Which of the following is TRUE in anaphylaxis? A. Anaphylactic urticarial
eruptions are intensely pruritic and frequently persist beyond 48 h B. Atopy predisposes
individuals to anaphylaxis from penicillin therapy C. The diagnosis of an anaphylactic reaction
depends largely on an accurate history D. NSAIDS may produce IgE related acute airway
obstruction A characteristic feature is the eruption of wellcircumscribed, discrete cutaneous
wheals with erythematous, raised, serpiginous borders and blanched centers. These urticarial
eruptions are intensely pruritic and may be localized or disseminated. They may coalesce to form
giant hives, and they seldom persist beyond 48 h. The diagnosis of an anaphylactic reaction
depends on a history revealing the onset of the symptoms and signs within minutes after the
responsible material is encountered. Aspirin and other NSAIDs such as indomethacin,
aminopyrine, and mefenamic acid may precipitate a life-threatening episode of obstruction of
upper or lower airways, especially in patients with asthma, that is clinically indistinguishable
from anaphylaxis but is not associated with the presence of specific IgE or elevation of blood
tryptase. According to most studies, atopy does not predispose individuals to anaphylaxis from
penicillin therapy or venom of a stinging insect but is a risk factor for allergens in food or latex.
(Harrison's, 17th ed) RACHELLE MENDOZA, MD (TOP 9 - AUG 2012 MED BOARDS;
TOPNOTCH MD) FINAL EXAM - FEB 2013 607 What part of the immune system does the
nitroblue tetrazolium test (NBT) evaluate? A. B cell B. T cell C. Phagocytic D. Complement
system E. All of the above Deficiencies of oxidative metabolism are detected with either the
nitroblue tetrazolium (NBT) dye test or the dihydrorhodamine (DHR) oxidation test. These tests
are based on the ability of products of oxidative metabolism to alter the oxidation states of
reporter molecules so that they can be detected microscopically (NBT) or by flow cytometry
(DHR). Qualitative studies of superoxide and hydrogen peroxide production may further define
neutrophil oxidative function. (Harrison's 17th ed) RACHELLE MENDOZA, MD (TOP 9 -
AUG 2012 MED BOARDS; TOPNOTCH MD) FINAL EXAM - FEB 2013 608 Dementia with
Lewy bodies is associated with the following syndromes/symptoms, EXCEPT? A. Capgras'
syndrome B. Parkinsonism C. Visual hallucinations D. Memory loss E. Major depression in
DLB, the first symptoms include: Visual hallucinations, REM sleep disorder, delirium, Capgras'
syndrome, parkinsonism. The mental status affected is the drawing and frontal/executive part. It
spares memory, but is delirium prone. Patients with DLB also complain of visual hallucinations,
depression, sleep disorder, delusions. RACHELLE MENDOZA, MD (TOP 9 - AUG 2012 MED
BOARDS; TOPNOTCH MD) FINAL EXAM - FEB 2013 TOPNOTCH MEDICAL BOARD
PREP INTERNAL MEDICINE SUPEREXAM For inquiries visit
www.topnotchboardprep.com.ph or email us at [email protected]
TOPNOTCH MEDICAL BOARD PREP INTERNAL MEDICINE SUPEREXAM Page 82 of 95
For inquiries visit www.topnotchboardprep.com.ph or email us at
[email protected] Item # QUESTION EXPLANATION AUTHOR
TOPNOTCH EXAM 609 A 44 year old renal transplant patient consulted at the OPD for painful
blisters located along his right breast. Tzanck smear was done, revealing which of the follwoing
results? A. Hyphae and budding yeast B. Multinucleated giant cells C. Granules D. Sea-blue
histiocytes E. Hirano bodies The patient has herpes zoster along a dermatomal distribution.
Tzanck smear is done to confirm diagnosis. Pathognomonic of herpes zoster is the presence of
multinucleated giant cells. RACHELLE MENDOZA, MD (TOP 9 - AUG 2012 MED BOARDS;
TOPNOTCH MD) FINAL EXAM - FEB 2013 610 Urobilinogen is produced in the: A. Renal
tubules B. Red blood cell C. Liver D. Intestinal mucosa E. Spleen Following secretion into bile,
conjugated bilirubin reaches the duodenum and passes down the gastrointestinal tract without
reabsorption by the intestinal mucosa. An appreciable fraction is converted by bacterial
metabolism in the gut to the water-soluble colorless compound, urobilinogen. Urobilinogen
undergoes enterohepatic cycling. Urobilinogen not taken up by the liver reaches the systemic
circulation, from which some is cleared by the kidneys. RACHELLE MENDOZA, MD (TOP 9 -
AUG 2012 MED BOARDS; TOPNOTCH MD) FINAL EXAM - FEB 2013 611 A 62-year old
male was observed to be moving more slowly than usual. He has not been sleeping well and was
always anxious. He was brought to your clinic and you observed wide-based, shuffling gait, with
cogwheel rigidity and resting, pill-rolling tremors. What is the mechanism of action of the most
appropriate treatment for this condition? A. Dopamine precursor with peripheral carboxylase
inhbitor B. Catechol-O-methyltransferase (COMT) inhibitor C. Dopamine agonist that acts
directly on postsynaptic dopamine receptors (D2) D. A or B E. A or C Either a dopamine agonist
or levodopa/carbidopa can be used as initial treatment for parkinson's RACHELLE MENDOZA,
MD (TOP 9 - AUG 2012 MED BOARDS; TOPNOTCH MD) FINAL EXAM - FEB 2013 612
Which of the following statements is/are correct regarding the pathogenesis of Type 2 DM? A.
There is a strong genetic component for the development of the disease. B. The presence of islet
cell autoantibodies characterizes the initial stage of the disease C. Exposure to nitrosureas,
coxsackie and rubella viruses has been identified as putative triggers of the disease. D. All of the
above All the others describe Type 1 DM RACHELLE MENDOZA, MD (TOP 9 - AUG 2012
MED BOARDS; TOPNOTCH MD) FINAL EXAM - FEB 2013 613 A patient with complete
biliary obstruction developed bleeding tendencies. Which of the following parameters may be
prolonged? A. Partial thromboplastin time B. Prothrombin time C. Bleeding time D. All of the
above E. A and B Patients with complete biliary obstruction will have impaired absorption of fat-
soluble vitamins, one of which is vitamin K. It is expected in this patient that his PTT and PT
may be prolonged, due to inadequate amounts of factors 10, 9, 7 and 2. Factor 10 and 2 is part of
both intrnsic and extrinsic coagulation pathway (common pathway), such that both PTT and PT
may be affected. RACHELLE MENDOZA, MD (TOP 9 - AUG 2012 MED BOARDS;
TOPNOTCH MD) FINAL EXAM - FEB 2013 614 A 21-year old male developed fever and sore
throat. On PE, there was a note of lymphadenopathy. Blood exam revealed leukocytosis, with
predominance of atypical lymphocytes. Which of the following may be a complication of this
condition? A. Splenic rupture B. Guillain-Barré syndrome C. Autoimmune hemolytic anemia D.
All of the above E. A and B The patient is diagnosed with infectious mononucleosis. Most cases
are self-limited. Deaths are very rare and most often are due to central nervous system (CNS)
complications, splenic rupture, upper airway obstruction, or bacterial superinfection. Acute EBV
infection has also been associated with cranial nerve palsies (especially those involving cranial
nerve VII), Guillain-Barré syndrome, acute transverse myelitis, and peripheral neuritis.
Autoimmune hemolytic anemia occurs in ~2% of cases during the first 2 weeks. Hypertrophy of
lymphoid tissue in the tonsils or adenoids can result in upper airway obstruction, as can
inflammation and edema of the epiglottis, pharynx, or uvula. About 10% of patients with IM
develop streptococcal pharyngitis after their initial sore throat resolves. RACHELLE
MENDOZA, MD (TOP 9 - AUG 2012 MED BOARDS; TOPNOTCH MD) FINAL EXAM -
FEB 2013 615 For disorders along the wall of the gastrointestinal tract, such as ulcers,
diverticula, strictures, new growths and motility disorders, the most useful diagnostic test to
order is: A. Radiograph B. MRI C. CT scan D. Ultrasound E. Scintigraphy Radiograph studies,
such as barium swallow and barium enema, can provide an actual picture of motility, filling
defects and mucosal defects when taken in series. Intetsinal obstruction is also preferred to be
viewed in upright and supine radiographic studies. RACHELLE MENDOZA, MD (TOP 9 -
AUG 2012 MED BOARDS; TOPNOTCH MD) FINAL EXAM - FEB 2013 TOPNOTCH
MEDICAL BOARD PREP INTERNAL MEDICINE SUPEREXAM For inquiries visit
www.topnotchboardprep.com.ph or email us at [email protected]
TOPNOTCH MEDICAL BOARD PREP INTERNAL MEDICINE SUPEREXAM Page 83 of 95
For inquiries visit www.topnotchboardprep.com.ph or email us at
[email protected] Item # QUESTION EXPLANATION AUTHOR
TOPNOTCH EXAM 616 This syndrome results from defective hypothalamic gonadotropin-
releasing hormone (GnRH) synthesis and is associated with anosmia or hyposmia due to
olfactory bulb agenesis or hypoplasia: A. Bardet-Biedl Syndrome B. Prader-Willi Syndrome C.
Carney syndrome D. McCune-Albright syndrome E. Kallmann syndrome McCune-Albright
syndrome consists of polyostotic fibrous dysplasia, pigmented skin patches, and a variety of
endocrine disorders, including GHsecreting pituitary tumors, adrenal adenomas, and autonomous
ovarian function. Carney syndrome is characterized by spotty skin pigmentation, myxomas, and
endocrine tumors including testicular, adrenal, and pituitary adenomas. Acromegaly occurs in
about 20% of patients. Prader-Willi Syndrome is associated with hypogonadotropic
hypogonadism, hyperphagiaobesity, chronic muscle hypotonia, mental retardation, and adult-
onset diabetes mellitus. Bardet-Biedl Syndrome is a rare genetically heterogeneous disorder
characterized by mental retardation, renal abnormalities, obesity, and hexadactyly, brachydactyly,
or syndactyly. Central diabetes insipidus may or may not be associated. RACHELLE
MENDOZA, MD (TOP 9 - AUG 2012 MED BOARDS; TOPNOTCH MD) FINAL EXAM -
FEB 2013 617 A 29-year old male complains of progressive attacks of joint pains and stiffness
(hip and knee joints), back pain and darkening of urine. Physical exam revealed generalized
darkening of the ear, along with limited range of motion for both lower extremities due to pain.
The underlying defect is: A. Lack of homogentisic acid oxidase B. Abnormal overproduction of
melanin granules C. Reduced activity of cystathionine synthase D. Overactivity of tyrosinase E.
None of the above Alkaptonuria is a rare disorder of tyrosine catabolism in which deficiency of
homogentisate 1,2-dioxygenase (also known as homogentisic acid oxidase) leads to excretion of
large amounts of homogentisic acid in urine and accumulation of oxidized homogentisic acid
pigment in connective tissues (ochronosis). Alkaptonuria may go unrecognized until middle life,
when degenerative joint disease develops. RACHELLE MENDOZA, MD (TOP 9 - AUG 2012
MED BOARDS; TOPNOTCH MD) FINAL EXAM - FEB 2013 618 Which of the following is
NOT a major criterion included in the Framingham criteria of congestive heart failure? A. Rales
B. Cardiomegaly C. Dypnea on exertion D. S3 gallop E. Neck vein distention Framingham
Major Criteria include: paroxysmal nocturnal dyspnea, rales, incresed CVP, neck vein distention,
cardiomegaly, acute pulmonary edema, s3 gallop, hepatojugular reflux, weight loss. Dyspnea on
exertion is included in the minor criteria. RACHELLE MENDOZA, MD (TOP 9 - AUG 2012
MED BOARDS; TOPNOTCH MD) FINAL EXAM - FEB 2013 619 A 63-year old congestive
heart failure patient frequently complains of worsening orthopnea, requiring him to use 4 pillows
when sleeping. The most probable mechanism for this is: A. Worsening interstitial pulmonary
edema B. Redistribution of fluid from the splanchnic circulation and lower extremities into
central circulation C. Increased pressure in the bronchial arteries leading to airway compression
D. Transudation of fluid from the intravscular space into the alveoli E. Diminished sensitivity of
the respiratory canter to arterial PCO2 Orthopnea is due to redistribution of fluid from the
splanchnic circulation and lower extremities into the central circulation during recumbency. PNH
is due to increased pressure on bronchial arteries leading to airway compression, along with
interstitial edema. Cheyne-Stokes respiration is due to diminished sensitivty of respiratory center
to arterial PCO2. Crackles/rales is due to transudation of fluid from the intravscular space into
the alveoli. RACHELLE MENDOZA, MD (TOP 9 - AUG 2012 MED BOARDS; TOPNOTCH
MD) FINAL EXAM - FEB 2013 620 A 30-year old male sought consult due to presistent
headache and palpitations. He was pale, restless and sweating profusely during the interview. BP
was 220/120 mmHg, HR was 124 bpm, PE was otherwise unremarkable. CT scan was requested
revealing an adrenal mass. Surgery was planned. What is the most appropriate pre-oprative
treatment for this patient? A. Nitroprusside drip 12 hours prior to surgery B. Nicardipine IV
infusion 12 hours prior to surgery C. Propranolol 10mg TID-QID 10-14 days prior to surgery D.
Phenoxybenzamine 20-30mg TID 10-14 days prior to surgery E. Captopril 25mg TID-QID 10-14
days prior to surgery Complete tumor removal is the ultimate therapeutic goal. Preoperative
patient preparation is essential for safe surgery. -Adrenergic blockers (phenoxybenzamine)
should be initiated at relatively low doses (e.g., 5–10 mg orally three times per day) and
increased as tolerated every few days. Because patients are volume constricted, liberal salt intake
and hydration are necessary to avoid orthostasis. Adequate alpha blockade generally requires 10–
14 days, with a typical final dose of 20–30 mg phenoxybenzamine three times per day. Oral
prazosin or intravenous phentolamine can be used to manage paroxysms while awaiting adequate
alpha blockade. Before surgery, the blood pressure should be consistently below 160/90 mmHg,
with moderate orthostasis. Beta blockers (e.g., 10 mg propranolol three to four times per day)
can be added after starting alpha blockers, and increased as needed, if tachycardia persists.
Because beta blockers can induce a paradoxical increase in blood pressure in the absence of
alpha blockade, they should be administered only after effective alpha blockade. Other
antihypertensives, such as calcium-channel blockers or angiotensin-converting enzyme
inhibitors, have also been used when blood pressure is difficult to control with
phenoxybenzamine alone. RACHELLE MENDOZA, MD (TOP 9 - AUG 2012 MED BOARDS;
TOPNOTCH MD) FINAL EXAM - FEB 2013 TOPNOTCH MEDICAL BOARD PREP
INTERNAL MEDICINE SUPEREXAM For inquiries visit www.topnotchboardprep.com.ph or
email us at [email protected] TOPNOTCH MEDICAL BOARD PREP
INTERNAL MEDICINE SUPEREXAM Page 84 of 95 For inquiries visit
www.topnotchboardprep.com.ph or email us at [email protected] Item #
QUESTION EXPLANATION AUTHOR TOPNOTCH EXAM 621 A 36 year old man presents
to the clinic with complaints of a genital sore. The patient is sexually active, heterosexual
involved with 3 partners and practices unprotected intercourse. Four days ago he noted, a
painless sore on his penis. He is afebrile with a HR of 80bpm, BP of 120/80 mmHg. PE reveals a
solitary ulcerated lesion located on the lateral aspect of his penis. The lesion is non tender and is
associated with with bilateral inguinal lymphadenopathy. PE is otherwise normal. If left
untreated, this man is at increased risk for which of the following? A. Ascending aortic aneurysm
B. Coronary artery aneurysm C. Endocarditis D. Mitral valve stenosis E. Rupture of ventricular
free wall This case is a PRIMARY SYPHILIS. It may lead to tertiary syphilis which can cause
ascending aortic aneurysm VON ANDRE MEDINA, MD (TOP 4 - FEB 2012 MED BOARDS;
TOPNOTCH MD) DIAGNOSTIC EXAM - AUG 2012 622 A 65 year old woman presents to the
ER department complaining of dizziness. She is disoriented to the date and her location and it is
difficult to gather an accurate history. Her pulse is 48 bpm, BP of 80/60 mmHg, and RR of 12
cpm. On examination, her extremities are cool and clammy. her capplary refill time is 5 seconds.
What is the most appropriate therapy? A. Adenosine B. Amiodarone C. Atropine D. Isoproterenol
E. Metoprolol patient has symptomatic bradycardia. Atropine is the dug of choice. VON ANDRE
MEDINA, MD (TOP 4 - FEB 2012 MED BOARDS; TOPNOTCH MD) DIAGNOSTIC EXAM
- AUG 2012 623 A 25 year old man is recovering in the hospital froman open rapair of his
broken femur, which he suffered during an automobile accident. On postoperative day 3, he
develops sudden onset of shortness of breath and vague chest pain. His temperature is 37.6 C and
HR of 108 bpm, BP of 90/60 mmHg, RR of 42 cpm and O2 sat of is 89 % on room air. PE is
significant for jugular venous distention to 9 cm and an accentuated pulmonic component of S2.
Which of the following is most likely DECREASED? A. Airway resistance B. Alveolar dead
space C. Alveolar ventilation D. Pulmonary compliance E. Pulmonary vascular resistance This is
a case of Pulmonary Embolism in which the compliance of the lungs (volume change per unit
pressure change) decreases. VON ANDRE MEDINA, MD (TOP 4 - FEB 2012 MED BOARDS;
TOPNOTCH MD) DIAGNOSTIC EXAM - AUG 2012 624 A 48 year old man is brought to the
emergency department confused and disoriented. He is a diagnosed case of DM type 1. He
reports recent onset of nausea and has had several episodes of emesis in the past 4 days. On
further questioning, he also notes a metallic taste in his mouth, frequent hiccups, and pruritus. On
PE there is rough, velcrolike sound heard accross his precordium. Which of the following is the
most likely diagnosis? A. Addisonian crisis B. Fulminant hepatic failure C. Heroin withdrawal D.
Renal insufficiency E. Vit B 12 insufficiency As uremic toxins increases, patients will complain
of pruritus, nausea and vomiting, hiccups, and a metallic taste in her mouth. Pericardial friction
rub in cases of uremic pericarditis VON ANDRE MEDINA, MD (TOP 4 - FEB 2012 MED
BOARDS; TOPNOTCH MD) DIAGNOSTIC EXAM - AUG 2012 625 A 43 year old man
develops fever, headache, and altered mental status. His past medical history is notable only for a
motor vehicle accident 2 years ago, during which he sustained a splenic laceration requiring
splenectomy. Which of the following is the most likely causing this patient's syndrome? A.
Cryptococcus neoformans and Listeria monocytogenes B. Cryptococcus neoformans and
Streptococcus pneumonia C. Haemophilus influenza and Staphylococcus aureus D. N.
meningitides and S. Aureus E. N. meningitides and Streptococcus pneumoniae encapsulated
organism such as Neisseria, Strep pneumoniae and H influenza may cause infection in asplenic
patients, as they are unable to manufacture a new antibody immune response VON ANDRE
MEDINA, MD (TOP 4 - FEB 2012 MED BOARDS; TOPNOTCH MD) DIAGNOSTIC EXAM
- AUG 2012 TOPNOTCH MEDICAL BOARD PREP INTERNAL MEDICINE SUPEREXAM
For inquiries visit www.topnotchboardprep.com.ph or email us at
[email protected] TOPNOTCH MEDICAL BOARD PREP INTERNAL
MEDICINE SUPEREXAM Page 85 of 95 For inquiries visit www.topnotchboardprep.com.ph or
email us at [email protected] Item # QUESTION EXPLANATION
AUTHOR TOPNOTCH EXAM 626 A 34 year old man presents with fever and night sweats for
3 weeks and productive cough. A recent HIV test was negative. A PPD test performed on
admission shows 15 mm induration. His ESR is 97 mm/hr. Past medical history is significant for
relapse of alcoholism. rebiew of systems reveal generalized fatigue over the past month and a 3.2
kg weight loss. His temperature is 39.6 C, RR is 25 cpm, and O2 sat is 86 % on room air.
Bilateral pulmonary rales are noted on P.E and moderate sternal retractions are present. X ray of
the chest reveals reticulonodular infiltrates spread evenly throughout both lung fields. Which of
the following is the most likely diagnosis? A. Latent tuberculosis B. Miliary tuberculosis C.
Pneumocystus jiroveci pneumonia D. Primary tuberculosis E. Reactivation pulmonary
tuberculosis This is a case of Miliary TB. It refers to hematogenous spread of the bacteria
following reactivation of a latent infection. The important information here is is the classic
MILIARY PATTERN ON XRAY of the chest indicative of hematogenous spread. VON ANDRE
MEDINA, MD (TOP 4 - FEB 2012 MED BOARDS; TOPNOTCH MD) DIAGNOSTIC EXAM
- AUG 2012 627 A 54 year old man presents to his physician complaining of difficulty urinating.
He urinates more often during the day and often wakes at night to urinate. He also has difficulty
starting and maintaining a stream of urine. DRE reveals a fusely, large, rubbery prostate.
Urinalysis and urine culture reveal no hematuria or signs of infection. What is the BEST next
step in the diagnosis? A. Cystoscopy B. Measure creatinine levels C. Measure PSA level D. UTZ
guided transrectal prostate biopsy E. X ray of the chest and bone scan This is a cse of BPH.
Urinary obstructive symptoms (hesitancy, weak stream, intermittent stream and etc). Serum
creatinine is used to assess for renal insufficiency caused by obstructive uropathy 2ndary to BPH.
VON ANDRE MEDINA, MD (TOP 4 - FEB 2012 MED BOARDS; TOPNOTCH MD)
DIAGNOSTIC EXAM - AUG 2012 628 A 29 year old type 1 diabetic presented to the ER with
clouded sensorium. He is febrile and tachypneic, sweating profusely and pale. His BP is 90/60
mmHg, CR of 116 bpm and CBG of 300 mg/dL. Relatives reported that he has cough and yellow
sputum for about a week now and has not sought consult nor took antibiotics. he has decreased
appetite for the last 3 days. His breath has a characteristic fruity odor. These statements are
TRUE regarding the pathophysiology of the case EXCEPT: A. Increase hepatic gluconeogenesis
B. Insulin deficiency C. Low peripheral glucose utilization D. Blunted counter-regulatory
response E. Insulin resistance VON ANDRE MEDINA, MD (TOP 4 - FEB 2012 MED
BOARDS; TOPNOTCH MD) DIAGNOSTIC EXAM - AUG 2012 629 The MOST useful
physiologic marker of thyroid hormone synthesis is: A. Triiodithyronine B. TSH C. TRH D.
Thyroxine E. Thyroglobulin TSH is the most useful physiologic marker of thyroid hormone
synthesis VON ANDRE MEDINA, MD (TOP 4 - FEB 2012 MED BOARDS; TOPNOTCH MD)
DIAGNOSTIC EXAM - AUG 2012 630 The gold standard in the diagnosis of gastroesophageal
reflux disease: A. Esophagoscopy B. 24 hr pH monitoring C. Barium swallow D. Berstein test E.
Breath test 24 hr pH monitoring VON ANDRE MEDINA, MD (TOP 4 - FEB 2012 MED
BOARDS; TOPNOTCH MD) DIAGNOSTIC EXAM - AUG 2012 631 A 37 year old seaman
came to your clinic for a second opinion. In his pre-employment medical examination, he turned
(+) for HBs Ag. His industrial physician requested for a complete hepatitis serology. The
following are the results: anti-HBs (-), anti -HBc (-), HBeAg (+), anti-HBe (-), anti-HAV IgG
(+), anti-HCV (-). The CORRECT interpretation of the results is: A. Active Hepa B with high
infectivity and active Hepa A B. Active Hepa B with high infectivity and previous Hepa A C.
Active Hepa B in the window period and previous Hepa A D. Active Hepa B with low infectivity
and previous Hepa A E. None of the above Active Hepa B with high infectivity and previous
Hepa A VON ANDRE MEDINA, MD (TOP 4 - FEB 2012 MED BOARDS; TOPNOTCH MD)
DIAGNOSTIC EXAM - AUG 2012 TOPNOTCH MEDICAL BOARD PREP INTERNAL
MEDICINE SUPEREXAM For inquiries visit www.topnotchboardprep.com.ph or email us at
[email protected] TOPNOTCH MEDICAL BOARD PREP INTERNAL
MEDICINE SUPEREXAM Page 86 of 95 For inquiries visit www.topnotchboardprep.com.ph or
email us at [email protected] Item # QUESTION EXPLANATION
AUTHOR TOPNOTCH EXAM 632 A 42 year old female came in due to diarrhea for 6 months.
She has stopped all dairy and milk products for the past few months as advised by a physician
who told her that her symptoms were caused by lactase deficiency. The diarrhea occurs
throughout the day and she has not noticed blood or pus in the stools. Past medical history
revealed PUD for many years that has been relatively resistant to medical treatment. She takes
maximum doses of omeprazole and famotidine and still has symptoms. Her PE is unrevealing.
her lab results are normal except for mild hypercalcemia. What is the most likely diagnosis? A.
H. pylori associated PUD B. Zollinger-Ellison Syndrome C. Irritable Bowel syndrome D.
Autoimmune atrophic gastritis E. None of the above Zollinger-Ellison Syndrome VON ANDRE
MEDINA, MD (TOP 4 - FEB 2012 MED BOARDS; TOPNOTCH MD) DIAGNOSTIC EXAM
- AUG 2012 633 What is the cornerstone of the treatment for NONALCOHOLIC fatty liver
diasease? A. Thiazolidinediones B. Statins C. Fibrates D. Metformin E. Weight Loss weight loss
VON ANDRE MEDINA, MD (TOP 4 - FEB 2012 MED BOARDS; TOPNOTCH MD)
DIAGNOSTIC EXAM - AUG 2012 634 A 24 year old man is brought to the ER department after
suffering blunt trauma to his chest in a motor vehicle accident. His RR is 32 cpm, CR of 125
bpm, and BP is 80/40 mmHg, with a decrease to 60/40 mmHg on inspiration. PE reveals
decreased heart sounds and a pericardial friction rub. Which other symptom would the physician
expect to see as part of the patient's presentation? A. Bounding pulse B. Bradycardia C.
Holosystolic murmur D. Hypertension E. Jugular venous distention This is a cse of Cardiac
tamponande. Beck's triad: hypotension, muffled heart sounds and JUGULAR VENOUS
DISTENTION VON ANDRE MEDINA, MD (TOP 4 - FEB 2012 MED BOARDS;
TOPNOTCH MD) DIAGNOSTIC EXAM - AUG 2012 635 A 47 year old man with
hypertension is prescribed with hydochlorothiazide by his primary care physician. Which of the
following is a potential effect of this medication? A. Hypokalemia B. Dilutional Hyponatremia
C. Hyperuricemia D. Hyperlipidemia E. All of the above VON ANDRE MEDINA, MD (TOP 4 -
FEB 2012 MED BOARDS; TOPNOTCH MD) DIAGNOSTIC EXAM - AUG 2012 636 An 85
year old man with hypertension and cerebrovascular disease presents to his physician's office for
a routine check up. Review of systems is positive only for "periodic ringing in the ears". PE
reveals left facial paralysis and poof hearing on the left side. A stroke in which vascular territory
is MOST likely? A. Anterior cerebral artery B. Anterior inferior cerebellar artery C. Middle
cerebral artery D. Posterior cerebral artery E. Posterior inferior cerebellar artery Anterior inferior
cerebellar artery , distribution typically present with gaze palsy, DEAFNESS, TINNITUS and
ipsilateral facial weakness. VON ANDRE MEDINA, MD (TOP 4 - FEB 2012 MED BOARDS;
TOPNOTCH MD) DIAGNOSTIC EXAM - AUG 2012 637 Which of the following is associated
with development of primary central nervous system lymphoma in ppatients with HIV? A. EBV
B. JC virus C. Mycobacterium avium complex D. Previous radiation exposure E. Toxoplasma
gondii Virtually all cases of primary CNS lymphoma in patients with HIV are associated with
EBV VON ANDRE MEDINA, MD (TOP 4 - FEB 2012 MED BOARDS; TOPNOTCH MD)
DIAGNOSTIC EXAM - AUG 2012 638 What is the best treatment for anemia in patients with
chronic kidney disease? A. Thrice weekly erythropoietin B. Periodic blood transfusion C. Daily
intake of ferrous sulfate D. Bone marrow transplantation E. Any of the above choices VON
ANDRE MEDINA, MD (TOP 4 - FEB 2012 MED BOARDS; TOPNOTCH MD)
DIAGNOSTIC EXAM - AUG 2012 TOPNOTCH MEDICAL BOARD PREP INTERNAL
MEDICINE SUPEREXAM For inquiries visit www.topnotchboardprep.com.ph or email us at
[email protected] TOPNOTCH MEDICAL BOARD PREP INTERNAL
MEDICINE SUPEREXAM Page 87 of 95 For inquiries visit www.topnotchboardprep.com.ph or
email us at [email protected] Item # QUESTION EXPLANATION
AUTHOR TOPNOTCH EXAM 639 What is the gold standard in the diagnosis of pulmonary
embolism? A. V/Q scan B. CT Scan with IV contrast C. Pulmonary angiography D. MRI contrast
enhanced E. Chest Xray gold standard: pulmonary angiography; principal imaging technique: CT
scan with IV contrast VON ANDRE MEDINA, MD (TOP 4 - FEB 2012 MED BOARDS;
TOPNOTCH MD) DIAGNOSTIC EXAM - AUG 2012 640 A 69 year old male was diagnosed to
have COPD. What is/are the only intervention/s proven to improve survival in severe COPD
patients? A. Smoking cessation B. Oxygen therapy C. Cyclic antibiotics D. Inhaled
corticosteroids E. A and B smoking cessation and oxygen therapy VON ANDRE MEDINA, MD
(TOP 4 - FEB 2012 MED BOARDS; TOPNOTCH MD) DIAGNOSTIC EXAM - AUG 2012
641 Primary Syphilis is best treated with? A. Ampicillin B. Amikacin C. Penicillin G D.
Spectinomycin E. Ceftriaxone The DOC for syphilis is still Pen G LITO JAY MACARAIG, MD
(TOP 8 - FEB 2013 MED BOARDS; TOPNOTCH MD) DIAGNOSTIC EXAM - AUG 2013
642 The most common complication of Peptic Ulcer disease is? A. Perforation B. Bleeding C.
Fistula formation D. Hypertrophy of the pylorus E. Peritonitis LITO JAY MACARAIG, MD
(TOP 8 - FEB 2013 MED BOARDS; TOPNOTCH MD) DIAGNOSTIC EXAM - AUG 2013
643 A 24 year old male consulted you due to purulent penile discharge. Gram stain showed
garm(-) intracellular diplococci. What is/are the best treatment? A. Ceftriaxone 250mg IM B.
Metronodazole 500mg tablet, 4 tablets single dose C. Azithromycin 1 gram single dose D. A and
B E. A and C Ceftriaxone should definitely be given. However, gonorrhea is almost always
accompanied by Chlamydial infection, hence Azithromycin or Doxycycline should be given too.
LITO JAY MACARAIG, MD (TOP 8 - FEB 2013 MED BOARDS; TOPNOTCH MD)
DIAGNOSTIC EXAM - AUG 2013 644 A 32 year old cave explorer came to your clinic due to a
3 week history of cough that became associated with blood-streaked sputum. What is the best
differential for this case? A. Community Acquired Pneumonia B. Pulmonary tuberculosis C.
Paragonimus westermani infection D. Histoplasma capsulatum infection LITO JAY
MACARAIG, MD (TOP 8 - FEB 2013 MED BOARDS; TOPNOTCH MD) DIAGNOSTIC
EXAM - AUG 2013 645 The lowest grade of murmur that you can observe thrill is A. Grade I B.
Grade II C. Grade III D. Grade IV E. Grade V LITO JAY MACARAIG, MD (TOP 8 - FEB 2013
MED BOARDS; TOPNOTCH MD) DIAGNOSTIC EXAM - AUG 2013 646 A 42 year old
teacher came to your clinic to consult for lesions in her skin. Upon PE, you saw vesicular,
erythematous and crusted lesions on her extremities, chest and back. Some lesions have the
characteristic dew-drop on a rose petal morphology. You are sure this is A. Scabies B. Psoriasis
C. Impetigo D. Varicella zoster E. Herpes simplex LITO JAY MACARAIG, MD (TOP 8 - FEB
2013 MED BOARDS; TOPNOTCH MD) DIAGNOSTIC EXAM - AUG 2013 647 A 56 year
year old alcoholic male came to ER due to severe hematemesis. Bleeding is uncontrollable,
hence you considered rupture of distal esophagus due to retching which is called A. Mallory-
Weiss Syndrome B. Boerhaave Syndrome C. Plummer-Vinson Syndrome D. Zenker's
Diverticulum E. Traction Diverticulum LITO JAY MACARAIG, MD (TOP 8 - FEB 2013 MED
BOARDS; TOPNOTCH MD) DIAGNOSTIC EXAM - AUG 2013 TOPNOTCH MEDICAL
BOARD PREP INTERNAL MEDICINE SUPEREXAM For inquiries visit
www.topnotchboardprep.com.ph or email us at [email protected]
TOPNOTCH MEDICAL BOARD PREP INTERNAL MEDICINE SUPEREXAM Page 88 of 95
For inquiries visit www.topnotchboardprep.com.ph or email us at
[email protected] Item # QUESTION EXPLANATION AUTHOR
TOPNOTCH EXAM 648 The most common presenting symptom of Pulmonary
Thromboembolism is A. Syncope B. Tachycardia C. Dyspnea D. Hemoptysis E. "sense of
impending doom" LITO JAY MACARAIG, MD (TOP 8 - FEB 2013 MED BOARDS;
TOPNOTCH MD) DIAGNOSTIC EXAM - AUG 2013 649 A tumor in the descending colon
commonly presents as A. Fatigue and palpitations B. tenesmus C. hematochezia D.
Hypochromic, microcytic anemia E. Applecore lesion on radiograph LITO JAY MACARAIG,
MD (TOP 8 - FEB 2013 MED BOARDS; TOPNOTCH MD) DIAGNOSTIC EXAM - AUG
2013 650 During your internship, you saw a patient with psoriasis scraping off her lesions and
punctate hemorrhages appeared. This is called A. Koebner phenomenon B. Wickham striae C.
Auspitz sign D. Forscheimer phenomenon E. Nagayama spots LITO JAY MACARAIG, MD
(TOP 8 - FEB 2013 MED BOARDS; TOPNOTCH MD) DIAGNOSTIC EXAM - AUG 2013
651 A 27 year old male was noted to have BP of 150/90 mmHg. Which drug should you
prescribe the least? A. Amlodipine B. Losartan C. Captopril D. Propranolol E.
Hydrochlorothiazide Beta blockers can cause impotence LITO JAY MACARAIG, MD (TOP 8 -
FEB 2013 MED BOARDS; TOPNOTCH MD) DIAGNOSTIC EXAM - AUG 2013 652 You are
given a patient with thalassemia major who had hemochromatosis. This is also known as A.
Bronze diabetes B. Wilson's disease C. Ondine's curse D. Gustilo's syndrome E. Beckmann's
Disease LITO JAY MACARAIG, MD (TOP 8 - FEB 2013 MED BOARDS; TOPNOTCH MD)
DIAGNOSTIC EXAM - AUG 2013 653 The Philippine CPG for Community Acquired
Pneumonia requires ______ as basis for its diagnosis. A. History of cough of >2 weeks B.
Clinical findings alone C. Routine sputum gram stain D. Radiologic confirmation E. Blood
culture LITO JAY MACARAIG, MD (TOP 8 - FEB 2013 MED BOARDS; TOPNOTCH MD)
DIAGNOSTIC EXAM - AUG 2013 654 the Global Initiative Against Asthma (GINA)
recommends addition of long acting beta-agonist as controller medication for A. Mild
intermittent asthma B. Mild persistent asthma C. Moderate persistent asthma D. Severe acute
attacks E. Status asthmaticus LITO JAY MACARAIG, MD (TOP 8 - FEB 2013 MED BOARDS;
TOPNOTCH MD) DIAGNOSTIC EXAM - AUG 2013 655 In a patient with goiter on
hyperthyroid state, this medication is usually given to reduce vascularity and size of the tumor A.
Lugol's iodine B. Propanolol C. PTU D. Methimazole E. Bromocriptine LITO JAY
MACARAIG, MD (TOP 8 - FEB 2013 MED BOARDS; TOPNOTCH MD) DIAGNOSTIC
EXAM - AUG 2013 656 A 45 year old patient presented with bone pain. Radiographic findings
showed lytic bone lesions and serology showed monoclonal antibodies. This patient would
probably be suffering from? A. Chronic Myelogenous Leukemia B. Primary Myeloproliferative
disease C. Multiple Myeloma D. Multiple Sclerosis E. Reynaud's Disease LITO JAY
MACARAIG, MD (TOP 8 - FEB 2013 MED BOARDS; TOPNOTCH MD) DIAGNOSTIC
EXAM - AUG 2013 TOPNOTCH MEDICAL BOARD PREP INTERNAL MEDICINE
SUPEREXAM For inquiries visit www.topnotchboardprep.com.ph or email us at
[email protected] TOPNOTCH MEDICAL BOARD PREP INTERNAL
MEDICINE SUPEREXAM Page 89 of 95 For inquiries visit www.topnotchboardprep.com.ph or
email us at [email protected] Item # QUESTION EXPLANATION
AUTHOR TOPNOTCH EXAM 657 A 29 year old female came to ER due to severe flank pain
and decreased amount of urine. Urinalysis showed gross hematuria with leukocytes, few
squamous cells and coffin-lid crystals. You know this is caused be renal stones composed by A.
Magnesium Ammonium Phosphate B. Calcium oxalate C. Uric acid D. Calcium carbonate E.
None of the above LITO JAY MACARAIG, MD (TOP 8 - FEB 2013 MED BOARDS;
TOPNOTCH MD) DIAGNOSTIC EXAM - AUG 2013 658 This is the pathognomonic lesion for
Rheumatic Fever. A. Anitschkow cells B. Caterpillar cells C. Aschoff bodies D. McCullough
Plaques E. Vegetations LITO JAY MACARAIG, MD (TOP 8 - FEB 2013 MED BOARDS;
TOPNOTCH MD) DIAGNOSTIC EXAM - AUG 2013 659 The following are Type III
hypersensitivity reactions, EXCEPT? A. Systemic LupusErythematosus B. Arthus Reaction C.
PSGN D. Serum sickness E. Goodpasteur's Syndrome LITO JAY MACARAIG, MD (TOP 8 -
FEB 2013 MED BOARDS; TOPNOTCH MD) DIAGNOSTIC EXAM - AUG 2013 660 You are
presented with a 34 year old call center agent who developed severe coughing episodes
associated with high grade fever. Further history revealed previous diagnosis of HIV. You
requested for CD4 count and revealed 20 y/o? A. H. influenzae B. N. meningitidis C. S.
pneumoniae D. L. monocytogenes E. None The most common causes per age group are as
follows: a - infants, b - adolescents, c - adults >20, d - immunosuppressed and elderly HAZEL
KAREN RAZ, MD (TOP 6 - FEB 2013 MED BOARDS; TOPNOTCH MD) MIDTERM 2 -
AUG 2013 670 Used in diagnosis of diabetes insipidus? A. Metyrapone test B. Water deprivation
test C. Demeclocycline administration D. 24 hr urine osmolality E. None Metyrapone test is used
to diagnose excess cortisol production in ACTH, demeclocycline is used as treatment for
SIADH/ HAZEL KAREN RAZ, MD (TOP 6 - FEB 2013 MED BOARDS; TOPNOTCH MD)
MIDTERM 2 - AUG 2013 671 A 65 year old female came in due to joint pain invloving her
knees and hip joints. Her symptoms appeared gradually with morning stiffness lasting less than
30 minutes. Pain worsens with activity. On PE, there are nodes at the DIP and PIP joints of the
hands. what is the diagnosis? A. Rheumatoid arthritis B. Osteoarthritis C. Gouty arthritis D.
Septic arthritis E. none The case is a classic presentation of osteoarthritis which is due to
decreased synovial fluid and leading to"wear and tear" of the joint. Treatment includes lifestyle
modification, exercise and NSAIDS, and surgery id with severe debilitation. HAZEL KAREN
RAZ, MD (TOP 6 - FEB 2013 MED BOARDS; TOPNOTCH MD) MIDTERM 2 - AUG 2013
672 True of hereditary spherocytosis? A. A form of intravascular hemolysis B. Presents during
early adulthood C. Cells have decreased surface - volume ratio D. A and b E. None Hereditary
spherocytosis is an auto-hemolytic anemia characterized by the production of red blood cells that
are sphere-shaped rather than biconcave disk shaped (donut-shaped), and therefore more prone to
hemolysis. It is an example of extravascular hemolysis which can lead to heppsplenomegaly and
usually present during childhood. HAZEL KAREN RAZ, MD (TOP 6 - FEB 2013 MED
BOARDS; TOPNOTCH MD) MIDTERM 2 - AUG 2013 673 Mechanism of lead poisoning,
except? A. Inhibition of ferrochelatase B. Decreased heme synthesis C. Inhibition of ALA
dehydrogenase D. A and b only E. None One of the main causes for the pathology of lead is that
it interferes with the activity of an essential enzyme called ALA dehydratase, which is important
in the biosynthesis of heme, the cofactor found in hemoglobin.Lead also inhibits the enzyme
ferrochelatase, another enzyme involved in the formation of heme HAZEL KAREN RAZ, MD
(TOP 6 - FEB 2013 MED BOARDS; TOPNOTCH MD) MIDTERM 2 - AUG 2013 674 Type of
skin cancer associated with exposure to sunlight. Lesions are described to have rolled edges with
central ulceration and "palisading nuclei" on microscopy? A. Squamous cell carcinoma B.
Melanoma C. Basal Cell Carcinoma D. Actinic Keratosis E. Dysplastic Nevus Basal cell
carcinoma is the most common form of skin cancer, it is locally invasive and rarely metastasize.
SCC is mainly caused by cumulative UV exposure over the course of a lifetime HAZEL KAREN
RAZ, MD (TOP 6 - FEB 2013 MED BOARDS; TOPNOTCH MD) MIDTERM 2 - AUG 2013
675 Disease characterized by decreased production of neurotransmitters from the locus ceruleus,
ventral tegmentum and raphe nucleus? A. Anxiety B. Depression C. Huntington's D.
Schizophrenia E. None Depression is due to a decreased production of neurotransmitters, namely
NE, dopamine and 5- HT from locus ceruleus, tegmentum of septal nucleus and raphe nucleus
respectively. HAZEL KAREN RAZ, MD (TOP 6 - FEB 2013 MED BOARDS; TOPNOTCH
MD) MIDTERM 2 - AUG 2013 TOPNOTCH MEDICAL BOARD PREP INTERNAL
MEDICINE SUPEREXAM For inquiries visit www.topnotchboardprep.com.ph or email us at
[email protected] TOPNOTCH MEDICAL BOARD PREP INTERNAL
MEDICINE SUPEREXAM Page 91 of 95 For inquiries visit www.topnotchboardprep.com.ph or
email us at [email protected] Item # QUESTION EXPLANATION
AUTHOR TOPNOTCH EXAM 676 Lesion in this areal eads to development of fluent aphasia?
A. BA 22 B. BA 44, 45 C. BA 3, 1, 2 D. BA 19 E. Arcuate fasciculus Fluent aphasia, also known
as sensory aphasia/ Wernicke's aphasia, wherein articulation is normal, however, perception is
impaired. This involves BA 22 also known as the superior temporal gyrus, or Wernicke's area.
HAZEL KAREN RAZ, MD (TOP 6 - FEB 2013 MED BOARDS; TOPNOTCH MD)
MIDTERM 2 - AUG 2013 677 Most common location of berry aneurysms in the Circle of
WIllis? A. ICA B. Junction between the posterior cerebral and posterior communicating artery C.
Junction between anterior cerebral artery and anterior communicating artery D. basilar artery E.
none he most common sites include the anterior cerebral artery and anterior communicating
artery (30–35%), the bifurcation, division of two branches, of the internal carotid and posterior
communicating artery (30–35%), the bifurcation of the middle cerebral artery (20%), the
bifurcation of the basilar artery, and the remaining posterior circulation arteries (5%). HAZEL
KAREN RAZ, MD (TOP 6 - FEB 2013 MED BOARDS; TOPNOTCH MD) MIDTERM 2 -
AUG 2013 678 Side effects of anti - TB medications, except? A. INH : hepatotoxicity B.
Rifampicin : peripheral neuropathy C. PZA : hyperuricemia D. Ethambutol : optic neuropathy E.
Streptomycin : ototoxic Rifampicin causes orange discoloration of body fluids, hepatotoxicity,
pruritus. HAZEL KAREN RAZ, MD (TOP 6 - FEB 2013 MED BOARDS; TOPNOTCH MD)
MIDTERM 2 - AUG 2013 679 Drug of choice for the treatment of anaerobic infections above
the diaphragm? A. Amoxicillin B. Cloxacillin C. Co - amoxiclav D. Metronidazole E.
Clindamycin The first three antibiotics are treatment for penicillin - susceptible microorganisms.
Metronidazole is used for treatment of anaerobic infections below the diaphragm. HAZEL
KAREN RAZ, MD (TOP 6 - FEB 2013 MED BOARDS; TOPNOTCH MD) MIDTERM 2 -
AUG 2013 680 Lumbar tap showed increased lymphocytes, protein and decreased sugar in a
patient with nuchal rigidity and irritability. Cause? A. Fungal B. Bacterial C. Viral D. Protozoan
E. None of the above Bacterial - inc pressure, predominance of PMN, increased protein,
decreased sugar Viral - n/inc pressure, lymphocytes, n/inc protein, normal sugar HAZEL
KAREN RAZ, MD (TOP 6 - FEB 2013 MED BOARDS; TOPNOTCH MD) MIDTERM 2 -
AUG 2013 681 Component therapy is encouraged in transfusion medicine to avoid volume
overload most especially in elderly patients. In a male patient with a current hemoglobin level of
96 g/L, how many units of packed red blood cell (PRBC) is needed to be transfused to raise the
hematocrit level to 0.38? A. 2 units B. 3 units C. 4 units D. 5 units Answer: B. 3 units of PRBC
Notes: 1 unit of PRBC increases haemoglobin by 1 g/dL in adults (3 g/dL in children) and
hematocrit by 3% (0.03). MICHELLE JAY FRANCISCO, MD (TOP 9 - FEB 2013 MED
BOARDS; TOPNOTCH MD) MIDTERM 1 - AUG 2013 682 Which seronegative
spondyloarthropathies associated with Human Leukocyte Antigen B27 most likely presents in a
young man complaining of chronic lower back pain with morning stiffness for more than an hour
which improves with exercise? A. Rieter’s Syndrome B. MCTD C. Psoriatic Arthritis D.
Ankylosing Spondylitis Answer: D. Ankylosing Spondylitis MICHELLE JAY FRANCISCO,
MD (TOP 9 - FEB 2013 MED BOARDS; TOPNOTCH MD) MIDTERM 1 - AUG 2013 683 The
following miscellaneous cardiac “pathognomonisms” are correctly matched; except, A. Parvus et
Tardus pulses: aortic stenosis B. Continuous machinery murmur: patent ductus arteriosus C.
Pulsus paradoxus: cardiac tamponade D. Bifid pulse: atrial-septal defect Answer: D. Bifid pulse:
atrial-septal defect Notes: • Bifid pulse: hypertrophic cardiomyopathy (from midsystolic
obstruction) • Fixed, split S2 heart sound: atrial-septal defect • Opening snap: mitral stenosis •
Parvus only: low output cardiomyopathy MICHELLE JAY FRANCISCO, MD (TOP 9 - FEB
2013 MED BOARDS; TOPNOTCH MD) MIDTERM 1 - AUG 2013 684 A 20 yo male
complained of chest pains and difficulty in breathing. He was seen in the ED, and his vital signs
were as follows: T = 37OC; PR = 88 bpm, regular; BP = 110/70 mmHg, RR = 20 cpm; and O2
sat on room air = 94%. PE revealed a rather anxious patient with no pallor, icterus, or cyanosis.
JVP was normal. The apical pulse was in the 5th ICS MCL, and a murmur was heard that
extended throughout the cardiac cycle. The most likely cause for this murmur is which of the
following? A. Aortic Stenosis B. Mitral regurgitation C. Pericarditis D. Patent Ductus Arteriosus
Answer: D. Patent Ductus Arteriosus Notes: This patient has a continuous murmur that straddles
the entire spectrum of the cardiac cycle. Such murmur is called a “machinery murmur,” and is
observed in patients having PDA or an arteriovenous fistula. MICHELLE JAY FRANCISCO,
MD (TOP 9 - FEB 2013 MED BOARDS; TOPNOTCH MD) MIDTERM 1 - AUG 2013
TOPNOTCH MEDICAL BOARD PREP INTERNAL MEDICINE SUPEREXAM For inquiries
visit www.topnotchboardprep.com.ph or email us at [email protected]
TOPNOTCH MEDICAL BOARD PREP INTERNAL MEDICINE SUPEREXAM Page 92 of 95
For inquiries visit www.topnotchboardprep.com.ph or email us at
[email protected] Item # QUESTION EXPLANATION AUTHOR
TOPNOTCH EXAM 685 A 69 yo man with a history of coronary artery disease and
hypertension presents with acute onset of right facial weakness and numbness. On examination,
his speech and extremity strength are normal, but he has significant weakness of the right side of
the face, including the orbicularis oculi. In addition, he complains of roaring in the right ear, and
his taste sensation is absent on the right side of the anterior tongue. Sensation is normal to
prinprick. Which of the following would best explain these findings? A. Lacunar stroke of the
left internal capsule B. Brainstem glioma C. Bell’s palsy D. A stroke due to occlusion of the left
middle cerebral artery Answer: C. Bell’s Palsy Notes: Bell’s palsy is idiopathic and nearly
always acute. All of the patient’s deficits are referable to the peripheral nervous system,
including the loss of taste (chorda tympani branch of the facial nerve) and hyperacusis (branch of
the stapedius muscle of the ear). MICHELLE JAY FRANCISCO, MD (TOP 9 - FEB 2013 MED
BOARDS; TOPNOTCH MD) MIDTERM 1 - AUG 2013 686 A 34 yo man comes to your clinic
complaining of “crushing” chest discomfort for 1 hour. He has no siginifcant medical history.
The ECG is normal but you still give sublingual nitroglycerin which immediately improves the
pain. What is the next step in the management? A. Request for Troponin T B. Repeat the ECG C.
Do a barium swallow D. Do an endoscopy Answer: C. Do a barium swallow Notes: The most
likely diagnosis for the patient in this case is DES (Diffuse Esophageal Spasm). MICHELLE
JAY FRANCISCO, MD (TOP 9 - FEB 2013 MED BOARDS; TOPNOTCH MD) MIDTERM 1 -
AUG 2013 687 A 60 yo known diabetic patient was rushed to the ER for unresponsiveness. On
physical examination, she was noted to have dry skin and oral mucosa with BP 100/70mmHg,
HR 100 bpm, RR 28 cpm. RBS = 600mg/dL, urine – negative for ketones. What is the initial step
in the management of this patient? A. Give IV insulin as soon as possible B. Insert a line and
administer plain NSS C. Give IV bolus of sodium bicarbonate D. Intubate the patient Answer: B.
Insert a line and administer plain NSS MICHELLE JAY FRANCISCO, MD (TOP 9 - FEB 2013
MED BOARDS; TOPNOTCH MD) MIDTERM 1 - AUG 2013 688 A 30 yo female is brought to
the ER for lethargy. You noted dry skin and hair, periorbital edema and sparse hair on the armpit.
Serum TSH and FT4 levels are low. What is your most probable diagnosis? A. Primary
hyperthyroidism B. Primary hypothyroidism C. Secondary hyperthyroidism D. Secondary
hypothyroidism Answer: D. Secondary Hypothyroidism MICHELLE JAY FRANCISCO, MD
(TOP 9 - FEB 2013 MED BOARDS; TOPNOTCH MD) MIDTERM 1 - AUG 2013 689 A 35 yo
patient is brought for evaluation of confusion for a day. You find that she has an elevated BP,
decreased air entry on right lung base with dullness to percussion, and symmetrical joint swelling
of the wrists. Chemistry profile shows elevated creatinine with proteinuria on urinalysis. What is
your diagnosis? A. Rheumatoid arthritis B. Systemic lupus erythematosus C. Mixed connective
tissue disease D. Nephritic syndrome Answer: B. Systemic lupus erythematosus MICHELLE
JAY FRANCISCO, MD (TOP 9 - FEB 2013 MED BOARDS; TOPNOTCH MD) MIDTERM 1 -
AUG 2013 690 Characteristic of Obstructive Lung Disease: A. Increased FEV/FVC, normal RV,
increased VC B. Decreased FEV/FVC, normal RV, increased VC C. Decreased FEV/FVC,
increased RV, decreased VC D. Increased FEV/FVC, increased RV, decreased VC Answer: C.
Decreased FEV/FVC, increased RV, decreased VC MICHELLE JAY FRANCISCO, MD (TOP 9
- FEB 2013 MED BOARDS; TOPNOTCH MD) MIDTERM 1 - AUG 2013 691 Which of the
following is characteristic of a transient ischemic attack? A. Weakness, paralysis or dysarthria in
one side of the face or extremity B. Incontinence of bowel & bladder C. Bilateral paresthesias,
numbness or aphasia D. Symptoms lasting longer than 24 hrs Answer: B TIA completely
resolves within 24hrs, involves unilateral sensory or motor deficit. MICHELLE JAY
FRANCISCO, MD (TOP 9 - FEB 2013 MED BOARDS; TOPNOTCH MD) MIDTERM 1 -
AUG 2013 692 An 80 yr-old male previously diagnosed to have duodenal bulb ulcer was
admitted because of repeated bouts of vomiting previously ingested food of 2 months duration
associated with weight loss and weakness. On PE, he was noted to have poor skin turgor and
succusion splash. What is the probable diagnosis? A. Achalasia B. Peptic stricture C. Gastric
outlet obstruction D. Perforated duodenal ulcer Answer: C Achalasia is a motor disorder of the
esophagus characterized by dysphagia, nocturnal vomiting and regurgitation but w/out succusion
splash. Peptic stricture occurs at CEJ brought about by chronic reflux disease. It manifests w/
progressive dysphagia but w/out succusion splash.Perforated duodenal ulcer will present w/
sudden epigastric pain, pallor, hypotension, and tachycardia but still no succusion splash.
MICHELLE JAY FRANCISCO, MD (TOP 9 - FEB 2013 MED BOARDS; TOPNOTCH MD)
MIDTERM 1 - AUG 2013 TOPNOTCH MEDICAL BOARD PREP INTERNAL MEDICINE
SUPEREXAM For inquiries visit www.topnotchboardprep.com.ph or email us at
[email protected] TOPNOTCH MEDICAL BOARD PREP INTERNAL
MEDICINE SUPEREXAM Page 93 of 95 For inquiries visit www.topnotchboardprep.com.ph or
email us at [email protected] Item # QUESTION EXPLANATION
AUTHOR TOPNOTCH EXAM 693 A significant amount of pleural effusion accompanying
pneumonia is an indication for: A. CT Scan B. Thoracentesis C. Closed tube thoracostomy D.
Video-assisted thoracoscopy Answer: B Thoracentesis allows for analysis of pleural fluid and
microbiologic testing for etiologic diagnosis. CTT is not indicated unless there is empyema of
complicated effusion. VATS and CT Scan are usually not needed. MICHELLE JAY
FRANCISCO, MD (TOP 9 - FEB 2013 MED BOARDS; TOPNOTCH MD) MIDTERM 1 -
AUG 2013 694 A 60 yr-old female with known coronary heart disease was recently diagnosed to
have hyperthyroidism probably due to Grave’s. The best treatment option for this patient is: A.
Anti-thyroid drugs for 3 years B. RAI therapy C. Subtotal thyroidectomy D. Total thyroidectomy
Answer: B In patients with coexisting medical conditions particularly heart disease, best
treatment is RAI. MICHELLE JAY FRANCISCO, MD (TOP 9 - FEB 2013 MED BOARDS;
TOPNOTCH MD) MIDTERM 1 - AUG 2013 695 An acute myocardial infarction that is
transmural produces dramatic and characteristic electrocardiographic changes. The earliest
change on ECG with an acute transmural infarction is: A. Peaking of T waves B. ST segment
elevation C. Development of new Q waves D. T wave inversion Answer: A Usually T wave
peaking in the leads reflecting electrical activity from the necrosing area of the myocardial
infarction is the first sign. These are referred to as hyperacute T wave changes. MICHELLE JAY
FRANCISCO, MD (TOP 9 - FEB 2013 MED BOARDS; TOPNOTCH MD) MIDTERM 1 -
AUG 2013 696 The patient is a 43 yr-old male. He is anemic with a hemoglobin of 12.2 g/dl an
MCV of 120 fL. Which of the following is the least likely diagnosis? A. Acute bleeding B. Folate
deficiency C. Iron deficiency D. Vit. B2 deficiency E. Vit. K deficiency Answer: C Choice A,B
& D will reveal macrocytic RBCS but IDA will have microcytic RBC (Normal MVC = 80-
100femtoliters). Vit. K deficiency will have problems w/ coagulation but not with RBC
morphology. MICHELLE JAY FRANCISCO, MD (TOP 9 - FEB 2013 MED BOARDS;
TOPNOTCH MD) MIDTERM 1 - AUG 2013 697 The ECG of a 66 yr-old male with a history of
atherosclerotic heart disease reveals an irregular but rapid heart rate. The QRS complexes are
normal but no P waves can be seen. The most likely reason for this finding is: A. Atrial
fibrillation B. Paroxysmal ventricular tachycardia C. Right bundle branch block D. Sinus
tachycardia E. Wolf-Parkinson-White syndrome Answer: A A-fib is a continuous chaotic re-entry
of electrical impulses within the atrial myocardium that arises in a diseased or stretched L atrium.
The chaotic patterns of atrial excitation prevents P-waves from being seen in the ECG. He
ventricular response is rapid and irregular. In all other choices, p wave should be distinguishable.
MICHELLE JAY FRANCISCO, MD (TOP 9 - FEB 2013 MED BOARDS; TOPNOTCH MD)
MIDTERM 1 - AUG 2013 698 One of you diabetic patient has a blood glucose level of 200
mg/dl. Surprisingly, a dipstick test is negative for urinary glucose. How could this finding be
explained? A. Dipstick tests are more sensitive for reducing sugars other than glucose B. Patient
has defective tubular glucose transporters C. Patient has diabetes insipidus D. Patient has
significant renal damage E. Patient is in a state of antidiuresis Answer: D Glucose excretion by
the kidneys depends glomerular filtration and tubular reabsorption. \At a renal threshold of 180
mg/dl, excess glucose is spilled into the urine and is detected by dipstick in as little as 100 mg/dl.
MICHELLE JAY FRANCISCO, MD (TOP 9 - FEB 2013 MED BOARDS; TOPNOTCH MD)
MIDTERM 1 - AUG 2013 699 A 67 yr-old woman complains of gradually increasing fatigue. On
physical examination, she is found to be anemic and has peripheral neuropathy characterized by
loss of position and vibratory sense. Labs show macrocytic anemia and low WBC and platelet
counts. Your impression is: A. Folate defiency anemia B. Pernicious anemia C. Chronic blood
loss D. Diabetes mellitus E. Myelodysplastic sideroblastic anemia Answer: B The inability to
absorb Vit.B12 leads to a macrocytic pancytopenia and peripheral neuropathy MICHELLE JAY
FRANCISCO, MD (TOP 9 - FEB 2013 MED BOARDS; TOPNOTCH MD) MIDTERM 1 -
AUG 2013 700 Calcium and aluminium-containing antacids should not be given to patients
taking which of the following? A. amoxicillin B. erythromycin C. isoniazid D. minocycline E.
rifampin Answer: D Tetracyclines such as minocycline chelates cations (Ca & Al). MICHELLE
JAY FRANCISCO, MD (TOP 9 - FEB 2013 MED BOARDS; TOPNOTCH MD) MIDTERM 1 -
AUG 2013 TOPNOTCH MEDICAL BOARD PREP INTERNAL MEDICINE SUPEREXAM
For inquiries visit www.topnotchboardprep.com.ph or email us at
[email protected] TOPNOTCH MEDICAL BOARD PREP INTERNAL
MEDICINE SUPEREXAM Page 94 of 95 For inquiries visit www.topnotchboardprep.com.ph or
email us at [email protected] Item # ANSWER 1 D 2 B 3 A 4 E 5 A 6 A 7
C 8 D 9 B 10 C 11 B 12 A 13 E 14 C 15 A 16 C 17 D 18 A 19 B 20 A 21 A 22 C 23 D 24 E 25 D
26 B 27 A 28 C 29 B 30 A 31 B 32 D 33 E 34 C 35 B 36 D 37 B 38 D 39 A 40 A 41 C 42 B 43 A
44 C 45 B 46 D 47 B 48 A 49 E 50 D 51 A 52 A 53 A 54 D 55 B 56 A 57 C 58 C 59 A 60 E 61 B
62 E 63 A 64 B 65 C 66 C 67 D 68 B 69 A 70 E 71 B 72 B 73 C 74 A 75 D 76 D 77 B 78 C 79 E
80 A 81 D 82 E 83 A 84 D 85 A 86 E 87 C 88 C 89 B 90 E 91 B 92 C 93 D 94 A 95 B 96 D 97 A
98 E 99 B 100 B 101 B 102 C 103 C 104 D 105 A 106 E 107 A 108 D 109 C 110 B 111 B 112 A
113 B 114 B 115 C 116 D 117 E 118 A 119 E 120 D 121 C 122 A 123 A 124 B 125 D 126 A 127
C 128 A 129 D 130 D 131 A 132 E 133 E 134 C 135 D 136 C 137 B 138 A 139 A 140 D 141 A
142 A 143 B 144 A 145 B 146 C 147 D 148 C 149 A 150 B 151 D 152 B 153 D 154 C or E 155
C 156 A 157 B 158 C 159 B 160 BONUS 161 D 162 A 163 C 164 B 165 C 166 D 167 B 168 A
169 C 170 C 171 B 172 C 173 A 174 E 175 C 176 A 177 E 178 E, C 179 A 180 E 181 B 182 C
183 D 184 B 185 A 186 E 187 A 188 D 189 C 190 A 191 B 192 B 193 C 194 E 195 A 196 C 197
C 198 E 199 D 200 E 201 E 202 B 203 D 204 C 205 A 206 C 207 C 208 B 209 D 210 B 211 C
212 E 213 C 214 E 215 D 216 B 217 A 218 C 219 D 220 C 221 A 222 A 223 E 224 A 225 D 226
C 227 D 228 C 229 B 230 B 231 D 232 C 233 C 234 E 235 A 236 C 237 B 238 A 239 C 240 B
241 B 242 C 243 B 244 C 245 A 246 A 247 D 248 E 249 C 250 A 251 B 252 A 253 C 254 E 255
A 256 B 257 B 258 C 259 B 260 A 261 D 262 C 263 A 264 B 265 B 266 C 267 E 268 E 269 A
270 C 271 A 272 D 273 C 274 E 275 E 276 A 277 C 278 B 279 B 280 C 281 E 282 B 283 B 284
A 285 D 286 E 287 C 288 A 289 D 290 D 291 C 292 C 293 E 294 A 295 D 296 B 297 C 298 B
299 D 300 E 301 D 302 B 303 B 304 D 305 D 306 B 307 A 308 B 309 C 310 C 311 D 312 C 313
E 314 D 315 C 316 D 317 E 318 C 319 B 320 A 321 A 322 D 323 C 324 B 325 C 326 C 327 D
328 A 329 E 330 D 331 C 332 D 333 D 334 B 335 E 336 D 337 A 338 C 339 D 340 A 341 B 342
A 343 D 344 B 345 A 346 C 347 A 348 B 349 A 350 A 351 C 352 C 353 D 354 B 355 A 356 B
357 D 358 A 359 D 360 D 361 B 362 B 363 E 364 B 365 C 366 E 367 D 368 A 369 C 370 C 371
C 372 C 373 A 374 B 375 E 376 A 377 E 378 E 379 A 380 D 381 A 382 B 383 E 384 C 385 C
386 A 387 D 388 C 389 B 390 A 391 A 392 C 393 D 394 A 395 E 396 C 397 C 398 E 399 B 400
D 401 B 402 E 403 E 404 D 405 B 406 A 407 C 408 B 409 D 410 D 411 A 412 C 413 D 414 B
415 A 416 A 417 D 418 A 419 B 420 D 421 A 422 C 423 D 424 C 425 C 426 A 427 B 428 C 429
B 430 C 431 D 432 B 433 A 434 D 435 C 436 A 437 A 438 D 439 D TOPNOTCH MEDICAL
BOARD PREP INTERNAL MEDICINE SUPEREXAM For inquiries visit
www.topnotchboardprep.com.ph or email us at [email protected]
TOPNOTCH MEDICAL BOARD PREP INTERNAL MEDICINE SUPEREXAM Page 95 of 95
For inquiries visit www.topnotchboardprep.com.ph or email us at
[email protected] 440 B 441 A 442 B 443 C 444 B 445 B 446 C 447 D 448
B 449 C 450 D 451 C 452 B 453 D 454 D 455 C 456 E 457 C 458 A 459 B 460 C 461 E 462 A
463 D 464 E 465 A 466 D 467 C 468 A 469 C 470 B 471 B 472 C 473 D 474 C 475 B 476 D 477
B 478 C 479 E 480 A 481 B 482 D 483 C 484 C 485 A 486 C 487 C 488 D 489 A 490 B 491 C
492 A 493 D 494 A 495 C 496 A 497 A 498 B 499 D 500 A 501 A 502 D 503 B 504 C 505 E 506
D 507 A 508 B 509 C 510 B 511 A 512 B 513 A 514 C 515 E 516 E 517 A 518 A 519 B 520 D
521 B 522 D 523 C 524 D 525 A 526 C 527 D 528 B 529 D 530 A 531 A 532 D 533 C 534 A 535
A 536 D 537 B 538 B 539 B 540 D 541 D 542 C 543 D 544 E 545 A 546 D 547 E 548 D 549 D
550 D 551 D 552 D 553 D 554 D 555 C 556 D 557 D 558 B 559 C 560 D 561 D 562 A 563 D
564 C 565 D 566 C 567 B 568 C 569 A 570 C 571 B 572 A 573 B 574 D 575 B 576 A 577 C 578
B 579 A 580 B 581 A 582 A 583 B 584 D 585 A 586 A 587 B 588 E 589 C 590 C 591 D 592 A
593 C 594 B 595 B 596 B 597 B 598 D 599 C 600 A 601 D 602 D 603 C 604 E 605 C 606 C 607
C 608 D 609 B 610 D 611 E 612 A 613 E 614 D 615 A 616 E 617 A 618 C 619 B 620 D 621 A
622 C 623 D 624 D 625 E 626 B 627 B 628 E 629 B 630 B 631 B 632 B 633 E 634 E 635 E 636
B 637 A 638 A 639 C 640 E 641 C 642 B 643 E 644 D 645 D 646 D 647 B 648 C 649 E 650 C
651 D 652 A 653 D 654 C 655 A 656 C 657 A 658 C 659 E 660 D 661 C 662 D 663 A 664 A 665
A 666 D 667 C 668 E 669 C 670 B 671 B 672 C 673 C 674 C 675 B 676 A 677 C 678 B 679 E
680 A 681 B 682 D 683 D 684 D 685 C 686 C 687 B 688 D 689 B 690 C 691 B 692 C 693 B
694 B 695 A 696 C 697 A 698 D 699 B 700 D

You might also like